You are on page 1of 290

ADVANCED CALCULUS AND

COMPLEX ANALYSIS
(Common to all branches except Bio group)

As per SRM University Syllabus and Pattern

Dr.K.Ganesan
Dr.Sundarammal Kesavan
Prof.K.S.Ganapathy Subramanian
Dr.V.Srinivasan

DEPARTMENT OF MATHEMATICS
SRM UNIVERSITY

M/S Gamma Publications


Maraimalai Nagar
Kancheepuram Dist.
2014
Sixth Edition, January 2015

All rights reserved.

This book or part thereof cannot be translated or reproduced


in any form (except for review or criticism) without the written
permission of the Authors and the Publishers.

Price: Rs.355.00

Published by:
GAMMA PUBLICATIONS
Maraimalai Nagar
Kancheepuram District.

Printed by:
SivaSanka Graphics
Mandavelipakkam, Chennai - 600 028
Cell: 9444049567
Website: www.sivasanka.com
PREFACE

The main purpose of the book is to help bridge the gap which
separates many students from mathematics, by giving them a
bird’s eye view of those Mathematical topics which are indis-
pensable, in the study of the Engineering and Technology.

It has all along been a concern of Engineers and Physicists that


the usual course in advanced calculus viz., differentiation, inte-
gration, etc., plays insufficient emphasis on the art of formulating
physical problems in Mathematical terms.

This book is an outcome of a series of lectures offered by the


authors to students having a working knowledge of the elemen-
tary concepts in Mathematics. The main objective of this book
is to provide the practical utility of Mathematics and reasona-
ble effort has been made to select the topics as per the syllabus
prescribed.

The topics are covered exhaustively. The explanations are cle-


ar and lucid. Engineering applications are introduced and many
illustrative examples are given. this edition includes a lot of exer-
cises along with the answers.

-Authors

i
ACKNOWLEDGEMENT

We sincerely thank our revered Chancellor Dr. T. R. Pachamut-


hu, Chairman, SRM group of Educational Institution. Thiru. P.
Ravi, President, Prof. P. Sathyanarayanan, Vice President, Dr.
V. Shivakumar of our SRM University without whose permission
and patronage, this book would not have materialized.

We also sincerely thank our respected and beloved Vice Chancel-


lor Dr. M. Ponnavaikko, Pro Vice Chancellor (P.D) Dr.T.P.Ganesan,
Registrar Dr.N.Sethuramanan, Director (Engineering and Tech-
nology) Dr.C.Muthamizhchelvan, Director(OFD) Dr. P. A. K.
Muniswaran of our SRM University without whose enormous
support and encouragement, publication of this book would not
have been possible.

We thank the reviewers Dr. R. Sekar, Professor, Department


of Mathematics, Pondicherry Engineering College, Puducherry
and Dr. T. Venugopal, Reader in Mathematics, Head Science
and Humanities, SCSVMV University, Kancheepuram for their
valuable comments and suggestions for the improvement of this
book.

Our thanks are due to our department colleagues for their valua-
ble comments and suggestions and help at every stage of the pre-
paration of this book. We also thank M/S.SivaSanka Graphics,
Mandavelipakkam, Chennai - 600 028 for their full cooperation
in bringing out this book in the present form.

-Authors

ii
DEDICATED TO OUR PARENTS

&

BELOVED TEACHERS

iii
SYLLABUS
MA1002 Advanced Calculus & Complex Analysis L T P C
Prerequisite 3 2 0 4
Nil

Purpose: To impart analytical ability in solving mathematical


problems as applied to the respective branches of Engineering.

Instructional objectives:

• To have knowledge in multiple integrals.

• To improve their ability in Vector calculus.

• To equip themselves familiar with Laplace transform.

• To expose to the concept of Analytical function .

• To familiarize with Complex integration.

UNIT I MULTIPLE INTEGRALS


Double integration in Cartesian and polar coordinates - Change
of order of integration - Area as a double integral - Triple inte-
gration in Cartesian coordinates - Conversion from Cartesian to
polar - Volume as a Triple Integral.

UNIT II VECTOR CALCULUS


Gradient, divergence, curl - Solenoidal and irrotational fields -
Vector identities (without proof) - Directional derivatives - Li-
ne, surface and volume integrals -Green’s, Gauss divergence and
Stoke’s theorems (without proof) - Verification and applications
to cubes and parallelopipeds only.

iv
UNIT III LAPLACE TRANSFORMS
Transforms of simple functions - Basic operational properties -
Transforms of derivatives and integrals - Initial and final value
theorems - Inverse transforms - Convolution theorem - periodic
functions - Applications of Laplace transforms for solving linear
ordinary differential equations up to second order with constant
coefficients only.

UNIT IV ANALYTIC FUNCTIONS


Definition of Analytic Function - Cauchy Riemann equations -
Properties of analytic functions - Determination of harmonic con-
jugate - Milne-Thomson’s method - Conformal mappings: 1/z,
az , az+b and bilinear transformation.

UNIT V COMPLEX INTEGRATION


Line integral - Cauchy’s integral theorem (without proof) - Cauchy’s
integral formulae and its applications - Taylor’s and Laurent’s ex-
pansions (statements only) - Singularities - Poles and Residues
- Cauchy’s residue theorem - Contour integration - Unit circle
and semi circular contour.

TEXT BOOKS:

1. Kreyszig.E, Ädvanced Engineering Mathematics", John Wi-


ley and Sons. Singapore, 10th edition, 2012.

2. K.Ganesan, Sundarammal Kesavan, K.S.Ganapathy Sub-


ramanian and V.Srinivasan, Ädvanced Calculus and Com-
plex Analysis", Revised Edition, 2014

REFERENCES:

v
1. Grewal B.S, Higher Engineering Mathematics, Khanna Pu-
blications, 42nd Edition, 2012.

2. Veerajan. T, Engineering Mathematics I, Tata McGraw


Hill Publishing Co, New Delhi, 5th edition, 2006.

3. Kandasamy P etal. Engineering Mathematics, Vol.I (4th


revised edition), S.Chand and Co., New Delhi, 2000.

4. Narayanan S., Manicavachagom Pillay T.K., Ramanaiah


G., Advanced Mathematics for Engineering students, Vo-
lume I (2nd edition), S.Viswanathan Printers and Publis-
hers, 1992.

5. Venkataraman M.K., Engineering Mathematics - First Ye-


ar (2nd edition), National Publishing Co., Chennai, 2000.

6. David E.Penney and C.Henry Edwards, Single Variable


Calculus, Prentice Hall; 6th edition, 2002.

vi
Inhaltsverzeichnis

1 MULTIPLE INTEGRALS 1
1.1 Double and Triple Integrals . . . . . . . . . . . . 1
1.2 Change of Order of Integration . . . . . . . . . . 28
1.3 Plane Area as Double Integral . . . . . . . . . . . 42
1.4 Change of Variable from Cartesian to Polar Co-
ordinates . . . . . . . . . . . . . . . . . . . . . . . 52
1.5 Volume as a Triple Integral . . . . . . . . . . . . 62
2 VECTOR CALCULUS 69
2.1 Vector Differential Operator ∆(del) . . . . . . . . 69
2.2 Gradient of a scalar point function . . . . . . . . 69
2.3 Level Surface . . . . . . . . . . . . . . . . . . . . 70
2.4 Directional Derivatives of a Scalar Point Function 70
2.5 Divergence of a Vector Point Function . . . . . . 71
2.6 Solenoidal Vector . . . . . . . . . . . . . . . . . . 71
2.7 Curl of a vector Point Function . . . . . . . . . . 71
2.8 Irrotational Vector . . . . . . . . . . . . . . . . . 72
2.9 Integration of Vectors . . . . . . . . . . . . . . . 99
2.9.1 Surface Integral . . . . . . . . . . . . . . . 101
2.9.2 Volume Integral . . . . . . . . . . . . . . . 102
2.10 Integral Theorems . . . . . . . . . . . . . . . . . 122
2.10.1 Green’s Theorem in the Plane . . . . . . . 122
2.10.2 Stoke’s Theorem (Relation between Line
and Surface Integrals) . . . . . . . . . . . 123
2.10.3 Gauss Divergence Theorem (Relation bet-
ween Surface and Volume Integrals) . . . 123

vii
Inhaltsverzeichnis

3 LAPLACE TRANSFORM 155


3.1 Introduction . . . . . . . . . . . . . . . . . . . . . 155
3.2 Some basic operational properties of Laplace trans-
forms . . . . . . . . . . . . . . . . . . . . . . . . . 157
3.2.1 Laplace transform of Derivatives . . . . . 161
3.3 Initial value and Final value theorem . . . . . . . 164
3.4 Convolution Theorem . . . . . . . . . . . . . . . 165
3.5 Laplace transform of periodic functions . . . . . . 173
3.6 Inverse Laplace Transform . . . . . . . . . . . . . 178
3.7 Convolution Theorem . . . . . . . . . . . . . . . 182
3.8 Application of Laplace Transform to Solve Diffe-
rential Equations . . . . . . . . . . . . . . . . . . 196
4 COMPLEX INTEGRATION 212
4.1 Evaluation of line integral . . . . . . . . . . . . . 213
4.2 Cauchy’s Integral Theorem . . . . . . . . . . . . 217
4.2.1 Cauchy’s Theorem for Multiply Connected
Region . . . . . . . . . . . . . . . . . . . . 218
4.3 Taylor’s Series and Laurent’s Series . . . . . . . . 229
4.4 Poles and Residues . . . . . . . . . . . . . . . . . 243
4.5 Cauchy’s Residue Theorem . . . . . . . . . . . . 247

viii
1 MULTIPLE INTEGRALS

INTRODUCTION AND NOTATION

1.1 Double and Triple Integrals

Ry Rx
The symbol y12 x12 f (x, y)dxdy denotes a double integral. It
means that f (x, y) is to be integrated first with respect to x,
considering y temporarily, as a constant, between the limits x1
and x2 and the resulting expression is then to be integrated with
respect to y between the limits y1 and y2 . Here x1 and x2 may
be constants or functions of y alone.

(i.e.) x1 = f1 (y), x2 = f2 (y), y1 and y2 are constants.


Rz Ry Rx
Similarly the symbol z12 y12 x12 f (x, y, z)dxdydz denotes a tri-
ple integral. This is concluded as follows. First f (x, y, z) is inte-
grated with respect to x between the limits x1 and x2 , keeping
y and z are constants temporarily. The resulting function is in-
tegrated with respect to y between the limits y1 and y2 , keeping
z constant. The result there obtained is finally integrated with
respect to z between the limits z1 and z2 . In this integral, z1 and
z2 are constants, y1 and y2 are either constants or functions of z
alone and x1 , x2 are either constants or functions of y and z.

1
1 MULTIPLE INTEGRALS

RR
Note: In f (x, y)dxdy, if the right hand integral has limits
involving x, then the first integral is with respect to y.

Differentiation Formulas:

d
(i) (k) = 0
dx

d
(ii) [f (x) ± g(x)] = f ′ (x) ± g ′ (x)
dx

d
(iii) [k.f (x)] = k.f ′ (x)
dx

d
(iv) [f (x).g(x)] = f (x)g ′ (x) + g(x)f ′ (x)
dx
 
d f (x) g(x)f ′ (x) − f (x)g ′ (x)
(v) =
dx g(x) [g(x)]2

d
(vi) f (g(x)) = f ′ (g(x)).g ′ (x)
dx

d n
(vii) (x ) = nxn−1
dx

d x
(viii) (e ) = ex
dx

d x
(ix) (a ) = ax log a, a > 0
dx

d 1
(x) (loge x) = , x > 0
dx x

2
1 MULTIPLE INTEGRALS

d
(xi) (sin x) = cos x
dx

d
(xii) (cos x) = − sin x
dx

d
(xiii) (tan x) = sec2 x
dx

d
(xiv) (cot x) = − csc2 x
dx

d
(xv) (sec x) = sec x tan x
dx

d
(xvi) (csc x) = − csc x cot x
dx

d
(xvii) (sinh x) = cosh x
dx

d
(xviii) (cosh x) = sinh x
dx

d
(xix) (tanh x) = sec2 x
dx

d
(xx) (coth x) = − csc2 x
dx

d
(xxi) (sec hx) = − sec hx tanh x
dx

d
(xxii) (csc hx) = − csc hx coth x
dx

3
1 MULTIPLE INTEGRALS

d 1 d
(xxiii) (sin−1 x) = √ = − (cos−1 x)
dx 1−x 2 dx

d 1 d
(xxiv) (tan−1 x) = 2
= − (cot−1 x)
dx 1+x dx

d 1 d
(xxv) (sec−1 x) = √ = − (csc−1 x)
dx x x2 − 1 dx

d 1
(xxvi) (sinh−1 x) = √
dx 1 + x2

d 1
(xxvii) (cosh−1 x) = √
dx 2
x −1

d 1
(xxviii) (tan−1 x) = , |x| < 1
dx 1 − x2

d 1
(xxix) (coth−1 x) = − 2 , |x| > 1
dx x −1

d −1
(xxx) (sec h−1 x) = √
dx x 1 − x2

d 1
(xxxi) (csc h−1 x) = √
dx x x2 + 1

Integration Formulas:
R
1. dx = x + C

R xn+1
2. xn dx = + C, n 6= −1
n+1

4
1 MULTIPLE INTEGRALS

R 1
3. dx = log x + C
x
R eax
4. eax dx = +C
a
R
5. sin xdx = − cos x + C
R
6. cos xdx = sin x + C
R
7. sec2 xdx = tan x + C
R
8. csc2 xdx = − cot x + C
R
9. sec x tan xdx = sec x + C
R
10. csc x cot xdx = − csc x + C

R dx
11. √ = sin−1 x + C
1 − x2

R dx
12. = tan−1 x + C
1 + x2
R dx
13. √ = sec−1 x + C
x x2 − 1
R
14. sinh xdx = cosh x + C
R
15. cosh xdx = sinh x + C
R
16. sec h2 xdx = tanh x + C

5
1 MULTIPLE INTEGRALS

R
17. csc h2 xdx = − coth xdx + C
R
18. sec hx tanh xdx = − sec hx + C
R
19. csc hx coth xdx = − csc hx + C

R dx 1 x−a
20. = log +C
x2 − a2 2a x+a
R dx 1 a+x
21. = log +C
a2 −x 2 2a a−x
R dx 1 x
22. = tan−1 + C
x2 + a2 a a
R dx x
23. √ = sin−1 + C
a2−x 2 a

R dx x √
24. √ = sinh−1 + C (or) log(x + x2 + a2 )
x2+a 2 a

R dx x √
25. √ = cosh−1 + C (or) log(x + x2 − a2 )
x2−a 2 a
R
26. tan xdx = log sec x = − log cos x + C
R
27. cot xdx = log sin x + C
R
28. sec xdx = log(sec x + tan x) + C
R
29. csc xdx = log(csc x − cot x) + C

6
1 MULTIPLE INTEGRALS

R eax
30. eax sin bxdx = (a cos bx + b sin bx) + C
a 2 + b2
R eax
31. eax cos bxdx = (a sin bx − b cos bx) + C
a 2 + b2

R√ a2 x x√ 2
32. a2 − x2 dx = sin−1 + a − x2 + C
2 a 2

R√ a2 x x√ 2
33. a2 + x2 dx = sinh−1 + a + x2 + C
2 a 2

R√ a2 x x√ 2
34. x2 − a2 dx = − cosh−1 + x − a2 + C
2 a 2
R ax
35. ax dx = +C
log a
R2R1
Example 1.1. Evaluate 0 0 4xydxdy

R2R1
Solution: Let I = 0 0 4xydxdy
 1
2 
R2 x2 y2
=4 0 y
dy = 2 =4
0 2 2 0
RaRbRc
Example 1.2. Evaluate 0 0 0 (x + y + z)dxdydz

 x=c
RaRb x2
Solution: Let I = 0 0 + yx + zx dydz
2 x=0
 
RaRb c2
= 0 0 + yc + zc dydz
2

7
1 MULTIPLE INTEGRALS

 y=b
Ra c2 y y 2 c
= 0 + + zcy dz
2 2 y=0

 
Ra c 2 b b2 c
= 0 + + zcb dz
2 2
 z=a
c2 bz b2 cz z 2 cb abc
= + + (a + b + c)=
2 2 2
z=0 2
R 2π R π R a
Example 1.3. Evaluate 0 0 0 r4 sin φdrdφdθ

Solution: The given integral


 a
R 2π Rπ Ra R 2π Rπ r5
0 dθ 0 dφ 0 r4 sin φdr = 0 dθ 0 sin φdφ
5 0

a5 R 2π R π a5 R 2π
= dθ sin φdφ = (− cos φ)π0 dθ
5 0 0 5 0
2 R 2π
= a5 0 dθ as cos π = −1, cos 0 = 1
5
4
= πa5
5
R1Rx
Example 1.4. Evaluate 0 0 dxdy

Solution: Let I be the given integral.


R1
∴ I = 0 (y)x0 dx as right hand integral has a limit which is func-
tion of x.

8
1 MULTIPLE INTEGRALS

 1
R1 1 x2
I= 0 xdx = = .
0 2 2
R π R a sin θ
Example 1.5. Evaluate 0 0 rdθdr

 a sin θ
Rπ r2 1 Rπ 2 2
Solution: Let I = 0 = a sin θdθ
2 0 2 0
 
a2 R π 1 − cos 2θ
= dθ
2 0 2
 
a2 1 sin 2θ π
= . θ−
2 2 2 0

πa2
= as sin π = 0, sin 0 = 0
4
R 1 R √1+y2 dxdy
Example 1.6. Evaluate 0 0
1 + x2 + y 2

 
R 1 R √1+y2 1
Solution: The given integral = 0 0 dx dy
(1 + y 2 ) + x2

" !#x=√1+y2
R1 1 x
= 0
p tan−1 p dy
1+y 2 1 + y2 x=0
" #
R1 1
= 0
p (tan−1 (1) − tan−1 0) dy
1 + y2

π R1 dy
= 0
p
4 1 + y2

9
1 MULTIPLE INTEGRALS

πh p i1 π √
= log(y + 1 + y 2 ) = log(1 + 2)
4 0 4
R a R √a2 −y2 p
Example 1.7. Evaluate 0 0 a2 − x2 − y 2 dxdy

R a R √a2 −y2 p
Solution: Let I = 0 0 a2 − x2 − y 2 dxdy

"   !#√a2 −y2


Ra xp 2 a2 − y2 x
= 0 (a − y 2 ) − x2 + sin−1 p dy
2 2 a − y2
2
0
   a
Ra a2 − y 2
π π 2 y3 πa3
= 0 y= a y− =
2 2 4 3 0 6
R 1 R 1 R 1−x
Example 1.8. Evaluate 0 y2 0 xdzdxdy

R 1 R 1 R 1−x
Solution: Let I = 0 y2 0 xdzdxdy

R1R1 R1R1
= 0 y2 x(z)01−x dxdy = 0 y 2 (x − x2 )dxdy
   
R1 1 1 y4 y6
= 0 − − − dy
2 3 2 3
 
R1 1 y4 y6
= 0 − + dy
6 2 3
 1
1 1 y5 1 y7
= y− +
6 2 5 3 7 0

1 1 1 4
= − + =
6 10 21 35

10
1 MULTIPLE INTEGRALS

R 1 R √1−x2 R √1−x2 −y2 dxdydz


Example 1.9. Evaluate 0 0 0 p
1 − x2 − y 2 − z 2

Solution:
" !#√1−x2 −y2
R 1 R √1−x2 z
Let I = 0 0 sin−1 p dxdy
1 − x − y2 − z2
2
0

R 1 R √1−x2 π R 1 √1−x2 π R1√


= 0 0 dxdy = 0 (y)0 dx = 1 − x2 dx
2 2 0
 1
π x√ 2
1 −1 π2
= 1 − x + sin x =
2 2 2 0 8
R π R a cos θ √
Example 1.10. Evaluate 2
0 0 r a2 − r2 dθdr

 
R π 1  a cos θ
Solution: Let I = 0
2
− (a2 − r2 )3/2 0

3

1 R π2  2 2 2

3/2 − (a2 )3/2 a cos θ dθ
=− 0 (a − a cos θ) 0
3
1 R π2 3 3
=− (a sin θ − a3 )dθ
3 0

a3 R π2
= (1 − sin3 θ)dθ
3 0
   π
a3 1 1 2
= θ+ − cos 3θ + 3 cos θ
3 4 3 0

11
1 MULTIPLE INTEGRALS
 
a3 π 2
= −
3 2 3
R1Ry 2
Example 1.11. Evaluate 0 0 xye−x dxdy

2
!y
R1 e−x
Solution: Let I = 0 y dy
2
0

1 R 1 −y2
=− (ye − y)dy
2 0
" 2
#1
1 e−y y2
=− −
2 −2 2
0

1 1
= [(e−1 + 1) − (1)] =
4 4e
R a R a−x R a−x−y
Example 1.12. Evaluate 0 0 0 x2 dxdydz

R a R a−x
Solution: Let I = 0 0 [x2 z]a−x−y
0 dxdy
R a R a−x
= 0 0 x2 (a − x − y)dxdy
 
2 a−x
Ra y
= 0 x2 (a − x)y − x2 . dx
2 0

1 Ra 2
= x (a − x)2 dx
2 0

1 a5
= [a2 x3 − 2ax3 + x4 ]a0 =
2 60

12
1 MULTIPLE INTEGRALS

RaRxRy
Example 1.13. Evaluate 0 0 0 xyzdzdydx

 
R a R x xyz 2 y
Solution: Let I = 0 0 dydx
2 0
 x
1 RaRx 3 1 R a xy 4
= xy dydx = dx
2 0 0 2 0 4 0
 a
1 x6 a6
= =
8 6 0 48
R 1/2 R 1 x
Example 1.14. Evaluate 0 0
p dydx
1 − x2 y 2

 1
R 1/2 sin−1 xy
Solution: Let I = 0 x
x 0

R 1/2
= 0 sin−1 xdx

h √
i1/2
√ π 3
−1
= x sin x + 1 − = x2 + −1
0 12 2
R 1 R 1−z R 1−y−z
Example 1.15. Evaluate 0 0 0 xyzdxdydz

 1−y−z
R 1 R 1−z x2
Solution: Let I = 0 0 yz dydz
2 0

1 R 1 R 1−z
= yz(1 − y − z)2 dydz
2 0 0
1 R1
= yz[(1 − z)2 − 2(1 − z)y + y 2 ]dydz
2 0

13
1 MULTIPLE INTEGRALS

 1−z
1 R1 2 y2 y3 y4
= z(1 − z) − 2z(1 − z) + z dz
2 0 2 3 4 0
 
1 R1 1 4 2 4 1 4
= z(1 − z) − z(1 − z) + z(1 − z) dz
2 0 2 3 4
 
1 1 2 1 R1
= − + 0 z(1 − z)4 dz
2 2 3 4

1 R1
= [1 − (1 − z)](1 − z)4 dz
24 0
 
1 (1 − z)5 (1 − z)6
= +
24 −5 6
 
1 1 1 1
= − =
24 5 6 720
R log 2 R x R x−y
Example 1.16. Evaluate 0 0 0 ex+y+z dxdydz

R log 2 R x R x−y
Solution: Let I = 0 0 0 ex+y+z dxdydz
R log 2 Rx
= 0 dx 0 dyex+y (ez )z=x+y
z=0

R log 2 Rx 
= 0 dx 0 e2x+2y − ex+y dy
 2y
y=x
R log 2 e
= 0 e2x − ex ey dx
2 y=0

 
R log 2 e4x 3e2x x
= 0 − + e dx
2 2

14
1 MULTIPLE INTEGRALS

 log 2
1 4x 3e2x 5
= e − + ex =
8 4 0 8
RR
Example 1.17. Evaluate xydxdy over the region bounded
by the x-axis, ordinate at x = 2a and the parabola x2 = 4ay.

Solution:

x2
In the region y varies from 0 to and x varies from 0 to 2a.
4a
y

x 2 = 4ay

(0,0) y=0 x = 2a x

Fig. 1

 x2 /4a
R 2a R x2 /4a R 2a y2
∴I= 0 0 xydydx = 0 x
2 0

R 2a x x4 1 R 2a 5
= .
0 2 16a2 dx = x dx
32a2 0

15
1 MULTIPLE INTEGRALS

 6 2a
1 x a4
= = .
32a2 6 0 3
RR
Example 1.18. Evaluate ydxdy over the area bounded by
x = 0, y = x2 , x + y = 2 in the first quadrant.

Solution: We have to find the integral over the region OAB.

y
y = x2

B A = (1,1)

x+y=2

O x

Fig. 2

 
R 1 R 2−x R 1 y 2 2−x
I= 0 x2 ydydx = 0 dx
2 x2

1 R1
= [(2 − x)2 − x4 ]dx
2 0
1 R1
= [4 − 4x + x2 − x4 ]dx
2 0

16
1 MULTIPLE INTEGRALS

 1
1 2 x3 x5
= 4x − 2x + +
2 3 5 0
 
1 1 1 16
= 4−2+ − =
2 3 5 15
RR 2
Example 1.19. Evaluate (x + y 2 )dxdy over the area of the
triangle whose vertices are (0, 1), (1, 1), (1, 2).

Solution:

C = (1, 2)

A = (0,1) B = (1,1)

O x

Fig. 3

y−2 x−1
The equation of the line AC is =
1 1

(i.e.) y = x + 1.
 x+1
R1 R x+1 R1 y3
∴I= 0 dx 1 (x2 + y 2 )dy = 0 x2 y + dx
3 1

17
1 MULTIPLE INTEGRALS
   
R1 (x + 1)3 1
= 0 x2 (x + 1) + 2
− x + dx
3 3
 1
1 R1 3 2 1 4 3 3x2
= (4x + 3x + 3x)dx = x +x +
3 0 3 2 0
 
1 3 7
= 1+1+ =
3 2 6
R R y/x
Example 1.20. Evaluate e dxdy where D is the region
bounded by the straight lines y = x, y = 0 and x = 1.

Solution: The region D is a triangle as shown in the figure.

(1,1)

x =0 y=x
x =1

(0,0) y=0 (1,0) x

Fig. 4

In this region x varies from 0 to 1. For each fixed x, y varies from


0 to x.
R1Rx R1
∴I= 0 0 ey/x dydx = 0 [xey/x ]x0

18
1 MULTIPLE INTEGRALS

R1 1
= x(e − 1)dx = (e − 1)
0 2
RR 2 2
Example 1.21. Evaluate D x y dxdy where D is the circular
2 2
disc x + y ≤ 1.

Solution:
√ In √
D, x varies from -1 to 1. For a fixed x, y varies from
− 1 − x2 to 1 − x2 .

y = 1 - x2

(0,0) y=0

x 2 + y2 = 1
Fig. 4a

R R 1 R √1−x2
∴ D x2 y 2 dxdy = √
−1 − 1−x2 x2 y 2 dydx

R 1 R √1−x2
=4 0 0 x2 y 2 dydx

 √ 2
R 1 x2 y 3 1−x
=4 0
3 0

4 R1 2
= x (1 − x2 )3/2 dx
3 0

19
1 MULTIPLE INTEGRALS

4 R π/2 2
= sin θ cos4 θdθ put x = sin θ, dx = cos θdθ
3 0
 
4 1.3.1  π  π
= =
3 2.4.6 2 24
RR
Example 1.22. Evaluate I = D xydxdy where D is the region
2
bounded by the curve x = y , x = 2 − y, y = 0 and y = 1.

Solution: The given region bounded by the curves is shown in


the figure.

y2 = x

y =1

x =0 x =2- y
D1 D2

y=0

Fig. 5

In this region x varies from 0 to 2 when 0 ≤ x ≤ 1 for fixed x, y



varies from 0 to x. When 1 ≤ x ≤ 2, y varies from 0 to 2 − x.

∴ The region D can be subdivided into two regions D1 and D2


as shown in the figure.
RR RR RR
I= D xydxdy = D1 xydydx + D2 xydydx

20
1 MULTIPLE INTEGRALS

R 1 R √x R 2 R 2−x
= 0 0 xydydx + 1 0 xydydx

 √  
R 1 xy 2 x R 2 xy 2 2−x
= 0 + 1 dx
2 0 2 0

1 R1 2 1 R2
= x dx + x(2 − x)2 dx
2 0 2 1
1 R1 2 1 R2 3
= 0 x dx + (x − 4x2 + 4x)dx
2 2 1
 1  2
x3 1 2 x4 4x3 3
= + 2x + + =
6 0 2 4 3 1 8
RRR
Example 1.23. Evaluate I = D xyzdxdydz where D is the
region bounded by the positive octant of the sphere x2 + y 2 + z 2 =
a2 .

Solution:

x 2 + y2 + z2 = a 2

x =0
D

(0,0) y=0 (a,0)


Fig. 6

The projection of the given region in the xy plane (z = 0) is the

21
1 MULTIPLE INTEGRALS

region bounded by the circle x2 + y 2 = a2 and lying in the 1st


quadrant as shown in the figure.

In √
the given region x varies 0 to a. For a fixed x, ypvaries from 0
to a2 − x2 . For a fixed (x, y), z varies from 0 to a2 − x2 − y 2

R a R √a2 −x2 R √a2 −x2 −y2


∴I= 0 0 0 xyzdzdydx

  a2 −x2 −y2
R a R √a2 −x2 z2
= 0 0 xy dydx
2 0

1 R a R √a2 −x2
= xy(a2 − x2 − y 2 )dydx
2 0 0
1 R a R √a2 −x2
= x(a2 y − x2 y − y 3 )dydx
2 0 0
 2 2 4
√a2 −x2
R
1 a y y y
= x a2 − x2 −
2 0 2 2 4 0

1 Ra
= x(a2 − x2 )2 dx
8 0
1 Ra 4
= (a x − 2a2 x3 + x5 )dx
8 0
 a
1 4 x2 2 x4 x6 a6
= a − 2a + =
8 2 4 6 0 48
RRR
dxdydz
Example 1.24. Evaluate I = D where D
(1 + x + y + z)3
is the region bounded by the planes x = 0, y = 0, z = 0 and
x + y + z = 1.

22
1 MULTIPLE INTEGRALS

Solution: The given region is a tetrahedron. The projection of


the given region in the xy plane is the triangle bounded by the
lines x = 0, y = 0 and x + y = 1 as shown in the figure. In the
y
B(0,1)

x + y =1
x =0

O y=0 A(1,0) x

Fig. 7

given region x varies 0 to 1. For each fixed x, y varies from 0 to


1 − x. For each fixed (x, y), z varies from 0 to 1 − x − y.

R 1 R 1−x R 1−x−y dxdydz


∴I= 0 0 0 (1 + x + y + z)3

1 R 1 R 1−x
=− [(1 + x + y + z)−2 ]01−x−y
2 0 0
 
1 R 1 R 1−x 1 −2
=− 0 0 − (x + y + 1) dydx
2 4
 1−x
1 R1 1 −1
=− y + (x + y + 1) dx
2 0 4 0

23
1 MULTIPLE INTEGRALS
 
1 R1 1 1 −1
=− 0 (1 − x) + − (x + 1) dx
2 4 2
 
1 1 1 2 1
=− x − x + x − log(x + 1)
2 4 8 2

1 5
= log 2 −
2 16

EXERCISE
R π/2 R π
1. Evaluate 0 π/2 cos(x + y)dxdy [Ans: -2]
 
R1Rx 1
2. Evaluate 0 0 ex+y dydx Ans: (e − 1)2
2
 
R π R a(1+cos θ) 3
3. Evaluate 0 0 rdθdr Ans: πa2
4
R1 R2 R2
4. Evaluate 0 dx 0 dy 1 x2 yzdz [Ans: 1]

R 3 R 2 dxdy 3
5. Find the value of 2 1 xy [Ans: log 2 log 2 ]

R1R2
6. Evaluate 0 0 xy 2 dydx. [Ans: 4/3]

R π R π/2 R 1 h πi
7. Find I = 0 0 0 r2 sin θdrdθdφ Ans:
3
R2R3R2
8. Find the value of 0 1 1 xy 2 zdzdydx [Ans: 26]
R 1 R z R y+z
9. Evaluate 0 0 0 dzdydx [Ans: 1/2]

24
1 MULTIPLE INTEGRALS

RR
10. Find the limits of R f (x, y)dxdy, where R is the 1st
quadrant bounded by (a) x = 0, y = 0, x + y = 1 (b)
x = 0, x = y, y = 1.
h R 1 R 1−x R 1 R 1−y i
Ans:a) 0 0 f (x, y)dxdy b) 0 0 f (x, y)dydx

R a R √a2 −x2
11. Sketch the region by integration to 0 0 f (x, y)dxdy.

Ans:
y

x 2 + y2 = a 2

x =0

O y=0 x

R1Rx
12. Sketch the region by integration to 0 0 f (x, y)dxdy.

Ans:
y

y=x

x =1

O y=0 x

R1Rx
13. Evaluate 0 0 dxdy

R2Rx 1 h π i
14. Evaluate 1 0 dxdy Ans: log 2
x2 + y 2 4

25
1 MULTIPLE INTEGRALS
 
R π/2 R a sin θ R (a2 −r2 )/a 5
15. Evaluate 0 0 0 rdθdrdz Ans: πa3
64
 
R 2 R x R x+y e8 − 3 3 4
16. Evaluate 0 0 0 ex+y+z dxdydz Ans: − e + e2
8 4
RR
17. Evaluate (x2 − y 2 )xdxdy
 over the positive quadrant of
a 5
the circle x2 + y 2 = a2 Ans:
15
RR 2 xdxdy over the area bounded by the
18. Evaluate (x + y) 
x 2 y 2 πab 2
ellipse 2 + 2 = 1 Ans: 2
(a + b )
a b 4
RR
19. Evaluate dxdy
 throughout
 the area bounded by y =
10
x2 , x + y = 2 Ans: −
3
RR 2 2
20. Evaluate
h x y dxdy over the area of the circle x2 + y 2 =
πi
1 Ans:
24
RR 2
21. Evaluate (x + y 2 )dxdy over the area
 bounded  by the
344
parabola y 2 = 4x and its lactusrectum. Ans:
105

R 1 R 1 ydxdy
22. Evaluate 0 x 2 and also sketch the region of inte-
h xi + y 2
π
gration. Ans:
4
RR
23. Evaluate xydxdy over the region in the positive qua-

26
1 MULTIPLE INTEGRALS
 
3
drant bounded by the line 2x + 3y = 6 Ans:
2
RRR
24. Evaluate V (xy+yz+zx)dxdydz, where V is the region
of sphere bounded
 by x = 0, x = 1, y = 0, y = 2, z = 0, z =
33
3 Ans:
2
RRR
25. Evaluate D x2 yzdxdydz, where D is the tetrahedron
x y z
bounded by the planes x = 0, y = 0, z = 0 and + + =
  a b c
a 2 b2 c 2
1. Ans:
2520
 
R π/4 R a sin θ rdrdθ a(π − 3)
26. Evaluate 0 0
√ Ans:
a2 − r 2 6

R4Ry ydxdy
27. Evaluate 0 y 2 /4 and also sketch the region of in-
x2 + y 2
tegration. [Ans: 2 log 2]
RR 2
28. Evaluate R x dxdy, where R is the region bounded by
the hyperbola xy = 4, y = 0, x = 1 and x = 2. [Ans: 6]
RRR
29. Evaluate V dxdydz, where V is the finite region of
space formed by the planes x = 0, y = 0, z = 0 and
2x + 3y + 4z = 12. [Ans: 12]
RR
30. Evaluate R xydxdy, where R is the region bounded by
the parabola y 2 = x and the lines y = 0 and x + y = 2,
lying in the 1st quadrant. [Ans: 3/8]

27
1 MULTIPLE INTEGRALS

1.2 Change of Order of Integration

Sometimes a double integral may be conveniently evaluated by


changing the order of integration. When the limits of the integra-
tion are variables, the change of order of integration will involve
a change in the limit also. In such cases, it will be convenient to
draw a figure indicating the original region of integration.
R a R a xdxdy
Example 1.25. Evaluate 0 y 2 by changing the order
x + y2
of integration.

Solution: The region D of integration is bounded by the lines


x = y, x = a, y = 0 and y = a as shown in the figure.
y
y=a

x =0 x=y
x =a
D

(0,0) y=0 (1,0) x

Fig. 8

We have to change the order of integration as dydx.

Here in the region D, x varies from 0 to a and for fixed x, y varies


from 0 to x.
  y  x
R a R a xdxdy R a R x xdxdy Ra 1 −1
∴ 0 y 2 = 0 0 2 = 0 x tan
x + y2 x + y2 x x 0

28
1 MULTIPLE INTEGRALS

R a h −1  y ix Ra
= 0 x tan dx = 0 [tan−1 1 − tan−1 0]dx
x 0
Ra π Ra
= 0 [tan−1 1 − tan−1 0]dx = dx
4 0
π a πa
= [x] =
4 0 4
R ∞ R ∞ e−y
Example 1.26. Evaluate I = 0 x dydx by changing the
y
order of integration.

Solution: The region of integration is as shown. When are change


the order x varies from 0 to y and y varies from 0 to ∞.
y
y=¥

y=x

(0,0) y=0 x

Fig. 9

R ∞ R ∞ e−y R ∞ R y e−y
I= 0 x dydx = 0 0 dydx
y y
 
R ∞ e−y y R ∞ −y
= 0 x = 0 e dy
y 0

29
1 MULTIPLE INTEGRALS
 ∞  
e−y e−∞ − e−0 0−1
= =1 = =1
−1 0 −1 −1
RaRa 2
Example 1.27. Change the order of integration in 0 x (x +
y 2 )dxdy and hence evaluate the same.

Solution: The region of integration is OAB bounded by x =


0, x = y, y = a and x = a.

On changing the order of integration x = 0, x = y. Then the


strip slides from y = 0 to y = a.
y
y=a A(a,a)
B

x =0 x =a
y=x

O y=0 x

Fig. 10

RaRa 2
RaRy
∴ 0 x (x + y 2 )dydx = 0 0 (x2 + y 2 )dxdy
 y  
R a x3 2
R a y3 3
= 0 +y x = 0 + y dy
3 0 3

4 Ra 3 1 a4
= 0 y dy = [y 4 ]a0 =
3 3 3
Example 1.28. By changing the order of integration, prove that
R ∞ R y −y2 /x 1
0 0 ye dxdy =
2

30
1 MULTIPLE INTEGRALS

Solution: The region of integration is as shown:


y
y=¥

x =0
x=y

y=0 x

Fig. 11

It is bounded by x = 0, x = y, y = 0, y = ∞. On changing the


order of integration y ranges from y = x, y = ∞ and x ranges
from x = 0 to x = ∞.
R∞Ry 2 /x R∞ R∞ −y 2 /x dydx
0 0 ye−y dxdy = x=0 y=x ye

  
R∞ R∞ 2 y2
= x=0 0 e−y /x d dx
2
" #∞
R ∞ 1 e−y2 /x R ∞ −x
= 0 = 0 (0 − ex )dx
2 −1/x 2
x

1 R ∞ −x
= xe dx, on integrating by parts
2 0
  −x  ∞
1 e e−x 1 1
= x − 1. = (0 + 1) =
2 −1 1 0 2 2

31
1 MULTIPLE INTEGRALS

Example 1.29. Change the order of integration and evaluate


R 1 R √1−x2 ey
I= 0 0 p dxdy
(ey + 1) 1 − x2 − y 2

√ The integration is2 with


Solution:
2
respect to y between y =
0, y = 1 − x . (i.e.) y = 0, x + y = 1 and the next integration
2

is w.r.to x between 0 and 1. The region of integration is as shown


here as OAB.
y
B

x =0

O y=0 x

Fig. 12

We now change the p


order of integration. Integration I w.r.to x
between x = 0, x = 1 − y 2 . To cover OAB, y slides from 0 to
1.
R 1 R √1−x2 ey
I= 0 0
p dxdy
(ey + 1) 1 − x2 − y 2

R 1 R √1−x2 ey
= 0 0
p dydx
(ey + 1) 1 − x2 − y 2

R 1 R √1−y2 ey
= 0 0 p dxdy
(ey + 1) 1 − x2 − y 2

32
1 MULTIPLE INTEGRALS
" #
R 1 R √1−y2 ey dx
= 0 0 p dy
(ey + 1) (1 − y 2 ) − x2

" #√1−y2
R1 ey x R1 ey π
= 0 y
sin−1 p dy = 0 y
dy
(e + 1) 1 − y2 (e + 1) 2
0
 
π π e+1
= [log(ey + 1)]10 = log
2 2 2
Example
R 3 R 4−y 1.30. By changing the order of integration, evaluate
0 1 (x + y)dxdy.

Solution: The region D of integration is bounded by the line


y = 0, y = 3, x = 1 and by the parabola x2 = 4 − y as shown in
the figure.
y

(1,3) y=3

x =1 y = 4 - x2

(0,0) (1,0) (2,0) y=0 x


Fig. 13

In this region, x varies from 1 to 2 and y varies from 0 to 4 − x2 .


R 3 R √4−y R 2 R 4−x2
0 1 (x + y)dxdy = 1 0 (x + y)dydx

33
1 MULTIPLE INTEGRALS

 4−x2  
R2 y2 R2 (4 − x2 )2
= 1 xy + = 1 x(4 − x2 ) + dx
2 0 2
 
R 2 x4 3 2
= 1 − x − 4x + 4x + 8 dx
2
 2
x5 x4 x3 241
= − − 4 + 2x2 + 8x =
10 4 3 1 60
Example√1.31. Change the order of integration and evaluate
R4R2 x
I = 0 x2 /4 dydx

x2
Solution: The region of integration is bounded by y = ,y =
√ 4
2 2
2 x. (i.e) x = 4y and y = 4x which are parabolas and x = 0
and 4. Solving y 2 = 4x and x2 = 4y, we get (0, 0), (4, 4).

After changing the order of integration


R 4 R 2√x R 4 R 2√x
I= 0 x2 /4
dydx = 0 x2 /4
dxdy

R4 √
2 x R4 √ 2
= 0 [x]x2 /4 dy = 0 (2 y − (y /4))dy

 4
4 3/2 y 3 32 16 16
= y − = − =
3 12 0 3 3 3
Example 1.32. Change the order of integration and evaluate
R 1 R 1 R √2−x2 x
I = 0 dx 0 x p dy
x2 + y 2


Solution: The limits for y are x and 2 − x2 and those for
x are 0 and 1. We, therefore, draw the curve y = x which is

34
1 MULTIPLE INTEGRALS

A(4, 4) y=4
2
x = 4y

x =0
x=4

O y=0 x

Fig. 14


a straight line and y = 2 − x2 which is the upper half of the
circle x2 +y 2 = 2. The region of integration is OACD. Solving the
equation y = x and x2 + y 2 = 2, we get the points of intersection
A(1, 1) and B(−1, −1). If we consider the strip parallel to the
x-axis the region has to be divided into two points OAD and
ADC. In the region ODA, x varies from 0 to y and p y varies from
0 to 1. In the region
√ ADC, x varies from 0 to 2 − y 2 and y
varies from 1 to 2.
R1 R √2−x2 x
∴I= 0 dx x
p dy
x + y2
2

R1 Ry x R √2 R √2−y2 x
= 0 dy 0
p dx + 1 dy 0 p dx = I1 + I2
2
x +y 2 x + y2
2

35
1 MULTIPLE INTEGRALS

C y= 2
A
y =1 D
x 2 + y2 = 2
x =0

O y=0 x =1

Fig. 15

R1 p R1 √
Now I1 = 0 dy[ x2 + y 2 ]y0 = 0 ( 2y − y)dy

 1
√ y2 1 √
(i.e.) I1 = ( 2 − 1) = ( 2 − 1)
2 0 2

R √2 p √ R √2 √
2−y 2
I2 = 1 dy[ x2 + y 2 ] 0 = 1 ( 2y − y)dy

  √2
√ y2 3 √
= 2y − = − 2
2 1 2

1
I = I1 + I2 = 1 − √
2
Example
R 5 R1.33. Change the order of integration and evaluate
2+x
I = 0 dx 2−x dxdy

Solution: The I integration is w.r.to y, where y = 2 − x and

36
1 MULTIPLE INTEGRALS

y = 2 + x. The II integration is w.r.to x where x = 0 to 5. The


region integration is as shown in the figure as ABC. Split ABC
as ADC, ADB.

y y=7

C(5,7)
y=2+x

A(0, 2) D(5, 2)
x =0
x =5
y=2- x

y=3 B(5, - 3) x

Fig. 16

R2 R5 R7R5
I = I1 + I2 , where I1 = −3 2−y dxdy and I2 = 2 y−2 dxdy

R2 5
R2
Now I1 = −3 [x]2−y = −3 (3 + y)dy

 2
y2 9 25
= 3y + = 17 − =
2 −3 2 2
R7 R7
and I2 = 2 [x]5y−2 = 2 (7 − y)dy

 7
y2 49 25
= 7y − = 37 − =
2 2 2 2

25 25
∴ I = I1 + I2 = + = 25
2 2

37
1 MULTIPLE INTEGRALS

Example
R a R 2a−x 1.34. Change the order of integration and evaluate
0 x2 /a xydxdy

x2
Solution: The I integration is w.r.to y with limits y = and
2
y = 2a − x. The II integration is w.r.to x between the limits
x = 0, x = a.

The region of integration is as shown in the figure as OAC.


We split this into two regions OAB, ABC. I = I1 + I2 whe-
re I1 corresponds to OAB and I2 corresponds to ABC. I =
y

y = x2 / 2

C(0, 2a)
x =0
B A(a,a)
x
O y=0 y = 2a - x

Fig. 17

R a R √ay R 2a R 2a−y
0 0 xydxdy + a 0 xydxdy

 √ay  2a−y
Ra x2 R 2a x2
= 0 y dy + a y dy
2 0 2 0

38
1 MULTIPLE INTEGRALS

R a a2 2 1 R 2a 2 2 3
= 0 2 y dy + 2 a (4a y − 4ay + y )dy

 a  a
a y3 1 2 2 y3 y4
= + 2a y − 4a +
2 3 0 2 3 4 0

a4 5a4 3a4
= + =
6 24 8

EXERCISE

R a R √a2 −y2
1. Change the order of integration in −a 0 f (x, y)dxdy
h R a R √a2 −x2 i
Ans: 0 −

a2 −x2
f (x, y)dydx

RaRa x
2. Change the order of integration in 0 y dxdy
x2 + y2
 
RaRx x
Ans: 0 0 dydx
x + y2
2

R∞Ry
3. Change the order of integration in 0 0 f (x, y)dxdy
 R∞R∞ 
Ans: 0 x f (x, y)dxdy
R 1 R 2√x
4. Change the order of integration in 0 0 φ(x, y)dydx
h R2R1 i
Ans: 0 y 2 /4 φ(x, y)dxdy

R1Rx
5. Change the order of integration in 0 0 dydx

39
1 MULTIPLE INTEGRALS
h R1R1 i
Ans: 0 y dxdy

RaRx
6. Change the order of integration in 0 x2 /a f (x, y)dydx

h R a R √ay i
Ans: 0 y f (x, y)dxdy

R ∞ R 1/y
7. Change the order of integration in 0 0 f (x, y)dxdy
h R ∞ R 1/y i
Ans: 0 0 f (x, y)dxdy

R1Rx
8. Change the order of integration and show that 0 0 dydx =
1
.
2
R4R2 2+
9. Change the order of integration and hence evaluate 1

y (x
 
2 1006
y )dxdy Ans:
105

10. Change the order of integration and hence evaluate


 
R a R √ax 2a4
0 0 x2 dydx Ans:
7
R a R √1−x2 2
11. Evaluate 0 h0 y dydx by interchanging the order of
πi
integration. Ans:
16
R a R √a2 −y2
12. Change the order of integration in 0 a−y ydxdy and
 3

a
then evaluate it. Ans:
6

40
1 MULTIPLE INTEGRALS

R 2a R a
13. Change the order of integration in 0 x2 /4a (x + y)dydx
 
9a3
and hence evaluate it. Ans:
5
R 2 R 4/x
14. Evaluate 1 0 xydydx by changing the order of integra-
tion. [Ans:8 log 2]
R 1 R 2−y
15. Change the order of integration in 0 y xydxdy and
 
1
then evaluate it. Ans:
3

R 2 R x2 x 2
16. Change the order of integration in 1 1 dxdy and hence
  y
2
evaluate the same. Ans: log 2 − 7
9

RaRy ydydx
17. Change the order of integration in 0 y 2 /a
p )
h (a − x)( ax − y 2
πa i
Ans:
2
R 5 R 4/x
18. Evaluate 3 0 xydxdy  after reversing the order of inte-
5
gration. Ans:8 log
3
R∞R∞ 2 (1+t2 )
19. Change the order of integration
h in e−x xdxdt
πi
0 0
and then evaluate it. Ans:
4

20. Change the order of integration in each of the double in-


R 1 R 2 dxdy R 2 R 2 dxdy
tegrals 0 1 2 and 1 y x2 + y 2 and hence express
x + y2

41
1 MULTIPLE INTEGRALS
h π i
their sum as one double integral and evaluate it. Ans: log 2
4
R1R1
21. Evaluate 0 √x ex/y dxdy by interchanging the order of in-
tegration. [Ans: 1/2]

1.3 Plane Area as Double Integral

Plane area enclosed by one or more curves can be expressed as a


double integral. Let R be the plane region, the area of which is
required. Let us divide the area into a large number of elemental
areas ABCD, by dividing lines provided to the y-axis of intervals
∆x and the lines parallel to the x-axis, at intervals of ∆y.
y

D C

A B

O x

Fig. 18

Area of the rectangle = ∆x∆y


PP RR
Rectangle of area ∆ = lim∆x→0 [ ∆x∆y] = R dxdy
RR
Note: In polar system A = D rdrdθ

42
1 MULTIPLE INTEGRALS

Example 1.35. Show, by double integration, that the area bet-


16 2
ween the parabolas y 2 = 4ax and x2 = 4ay is a .
3

Solution: The region by integration is as shown.

A(4, 4)

x 2 = 4ay

O x

y 2 = 4ax

Fig. 19

Solving y 2 = 4ax and x2 = 4ay are set (0, 0), (4a, 4a). Take a
x2 √
strip parallel to y axis implies limits for y = and y = 2 ax
4a
and then x varies from 0 to 4a.
R 4a R 2√ax R 4a √
2 ax
∴ Area = 0 x2 /4a
dx = 0 [y]x2 /4a

 4a
 
R 4a √ 1/2 x2  √ x1/2 + 1 1 x3 
= 0 2 ax − dx = 2 a − 
4a 1 4a 3
+1
2 0

43
1 MULTIPLE INTEGRALS

 4a
4 √ 3/2 1 3 4√ 1
= ax − x = a(4a3/2 ) − (4a)3
3 12a 0 3 12a

4 1
= a2 (4 × 2) − × 64a3
3 12a
32 2 16 2 16 2
= a − a = a Sq.units.
3 3 3
Example 1.36. Find by double integration, the area enclosed by
x2 y 2
the ellipse 2 + 2 = 1
a b

Solution: The curve is symmetrical about both axes.


y
y = b 1 - x2 / a2

O
x 2 / a 2 + y2 / b2 = 1 y=0 x

Fig. 20

∴ Area = 4 area in I quadrant.


q
2
RR RaRb 1− x2
a
=4 A dydx =4 0 0 dydx
q
4b R a √ 2
2
Ra b 1− x2
=4 0 [y] 0
a
= a − x2 dx
a 0

44
1 MULTIPLE INTEGRALS
 a
4b x √ 2 2
a2 −1 x
= a −x + sin
a 2 2 a 0

4b a2 π
= × × = πab Sq.units.
a 2 2
Example 1.37. Find by double integration, smaller of the areas
bounded by the circle x2 + y 2 = 9 and x + y = 3.

√ The region of integration is as shown y varies from


Solution:
3 − x to 9 − x2 and x varies from 0 to 3.
y

(0,3)

x+y=3

O
(3,0) x

x 2 + y2 = 9

Fig. 21

RR
Area = dydx

R 3 R √9−x2 R3 √
= dydx = 2
0 3−x 0 [ 9 − x − (3 − x)]dx

 3  3
x√ 9 x x2
= 9 − x2 + sin−1 − 3[x]30 +
2 2 3 0 2 0

45
1 MULTIPLE INTEGRALS

9π 9 9π 9 9
= −9+ = − = (π − 2) sq.units.
22 2 4 2 4
Example 1.38. Find the area between the parabola y = 4x − x2
and the line y = x.

Solution:
y
y = 4x - x 2

A(3,3)
x=0

y=x

x
Fig. 22

The two curves intersect at points whose abscissa are given by


4x−x2 = x or x2 −3x = 0 (i.e.) x = 0.3. Using vertical strips, the
required area lies between x = 0, x = 3 and y = x, y = 4x − x2 .
R 3 R 4x−x2 R3 2
∴ Required area = 0 x dydx = 0 [y]4x−x
x dx
R3 R3
= 0 (4x − x2 − x)dx = 0 (3x − x2 )dx
 3
3x2 x3
= − = 4.5
2 3 0

46
1 MULTIPLE INTEGRALS

Example 1.39. Find by double integration, the area bounded by


the parabola y = x2 and the line y = 2x + 3.

Solution: The region of integration is as shown.


y

y = 2x + 3

y = x2

x
O

Fig. 23

Solving y = 2x + 3 we get x2 − 2x − 3 = 0. (i.e.) x = 3, −1.


RR
Required area = dydx where y varies from y = x2 and y =
2x + 3. Further x varies from -1 to 3.
R 3 R 2x+3 R3 2x+3
∴ Required area = −1 x2 dydx = −1 [y]x2 dx

  2 3
R3 2 )dx = 2 x x3
= (2x + 3 − x + 3x −
−1 2 3 −1

32 2
= = 10 Sq.units.
3 3
Example 1.40. Find the area of the circle using double integral.

RR
Solution: Area of the circle A = 4 D dxdy where D is the

47
1 MULTIPLE INTEGRALS

bounded region in the I quadrant of xy-plane.√In D, x varies


from 0 to r, and for fixed x, y varies from 0 to r2 − x2 .
y

(0, r)

(0,0) (r,0) x

Fig. 24

R r R √r2 −x2 Rr√


A=4 0 0 dydx = 4 0 r2 − x2 put x = r sin θ

R π/2 R π/2 1 + cos 2θ


= 4r2 0 cos2 θdθ = 4r2 0 dθ
2
 π/2
sin 2θ
= 2r2 θ+ = πr2 as sin π = 0, sin 0 = 0
2 0

Example 1.41. Find the area of the region D bounded by the


parabola y = x2 and x = y 2 .

Solution: The point of intersection of the curve are given by


(0, 0) and (1, 1). The required region D is as shown below.
RR R 1 R √x
The required area A = D dxdy = 0 x2
dydx

48
1 MULTIPLE INTEGRALS

y = x2
(1,1)
y2 = x

O x

Fig. 25

R1 √
x R1 √ 2
= 0 [y]x2 dx = 0 ( x − x )dx

" #1
x3/2 x3 1
= 2 − =
3 3 3
0
Example 1.42. Find by double integration, the area lying inside
the circle r = a sin θ and outside the coordinate r = a(1 − cos θ).

Solution: Eliminating r between the equations of two curves


sin θ = 1 − cos θ or sin θ + cos θ = 1.

Squaring 1 + sin 2θ = 1 or sin 2θ = 0 ∴ 2θ = 0 or π


π
(i.e.) θ = 0 or .
2

For the required area, r varies from a(1 − cos θ) to a sin θ and θ

49
1 MULTIPLE INTEGRALS

r = a(1 - cos q) q=p/2

q=0 x

Fig. 26

π
varies from 0 to .
2
R π/2 R a sin θ
Required area = 0 a(1−cos θ) rdrdθ

 
R π/2 r2 a sin θ
= 0
2 a(1−cos θ)

1 R π/2 2
= a [sin2 θ − (1 − cos θ)2 ]dθ
2 0

a2 R π/2  
= (2 cos θ − 2 cos 2 θ)dθ = a2 1 − π
2 0 4

EXERCISE

1. Find the smaller of the areas bounded the ellipse 4x2 +

50
1 MULTIPLE INTEGRALS
 
3
9y 2 = 36 and the straight line 2x+3y = 6. Ans: (π − 2)
2

2. Find by double integration, the area bounded by the para-


bolas y 2 = 4 − x and y 2 = 4 − 4x [Ans: 8]

3. Find the area bounded by the circle r = 2 sin θ and r =


4 sin θ [Ans: 3π]

4. Find the area enclosed by the curve r2 = a2 cos 2θ, by


double integration. [Ans: a2 ]

5. Find the area bounded by the parabola  y 2 = 4 − x and


16 √
y 2 = x by double integration. Ans: 2
3

6. Find the area common to the parabolay 2 = x and x2 +y 2 =


π 1
2 by double integration. Ans: +
2 3

7. Find the area that lies outside


 the circle
 r = a cos θ and
3 2
inside the circle r = 2a cos θ Ans: πa
4

8. Find the"area common to√the


!#two circles r = a and r =
2π 3
2a cos θ. Ans:a2 −
3 2

51
1 MULTIPLE INTEGRALS

1.4 Change of Variable from Cartesian to


Polar Co-ordinates

RR
Let R f (x, y)dxdy be the double integral. x = r cos θ, y =
r sin θ is the transformation from Cartesian to Polar coordinates.

∂(x, y)
Then dxdy = |J|drdθ where J = is the Jacobian of the
∂(r, θ)
transformation and

∂x ∂x

∂r ∂θ
cos θ −r sin θ
J = =
∂y ∂y sin θ r cos θ


∂r ∂θ

= r(cos2 θ + sin2 θ) = r
RR RR
∴ R f (x, y)dxdy = R f (r, θ)drdθ
RaRa
Example 1.43. Change to polar coordinates and evaluate 0 y xdxdy

Solution: The region of integration is x = y, x = a, y = 0, y = a.

(i.e.) The triangle OAB putting x = r cos θ, y = r sin θ, the line


x = y becomes r cos θ = r sin θ
π
(i.e.) tan θ = 1 ⇒ θ =
4
 a/ cos θ
R π R a/ cos θ R π r3
Hence in polar form I = 0
4
0 r2 cos θdrdθ = 0
4
cos θ dθ
3 0

52
1 MULTIPLE INTEGRALS

a3 R π4
sec2 θdθ
3 0

a3 π a3
= [tan θ]04 =
3 3
y

x=y

A
O
x =a
x

Fig. 27

Example 1.44. Change to polar coordinates and evaluate


R a R √a2 −x2 2 +y 2 )
0 0 e−(x dxdy

Solution: Putting x = r cos θ, y = r sin θ, the given limit y 2 =


a2 − x2 .

(i.e) The circle x2 + y 2 = a2 changes to r = a and y = 0.

(i.e.) The x-axis changes to initial line θ = 0. Hence, in the given

53
1 MULTIPLE INTEGRALS

π
region r changes from 0 to a and θ changes from 0 to .
2
R π Ra 2
I= 0
2
0 e−r rdrdθ
 a
R π 1 2
= 2
0 − e−r dθ
2 0

1 R π2 −a2 π 2
=− (e − 1)dθ = (1 − e−a )
2 0 4
y

p r =a
q=
2

O q=0 x

Fig. 28

R 4a R y
Example 1.45. Evaluate 0 y 2 /4a dydx by changing to polar
coordinates.

Solution: The region of integration is bounded by the parabola


x = y 2 /4a.

54
1 MULTIPLE INTEGRALS

(i.e.) y 2 = 4ax and the line x = y.

By putting x = r cos θ, y = r sin θ, the parabola becomes r2 sin2 θ =


4ar cos θ.

4a cos θ
(i.e.) r = and the line becomes x cos θ = r sin θ
sin2 θ
π
(i.e.) θ =
4
y

A
y 2 = 4ax

x=y

O x

Fig. 29

4a cos θ π π
Hence r varies from 0 to 2 and θ varies from to .
sin θ 4 2
R π R 4a cos θ
sin2 θ
I= π
2
0 rdθdr
4

55
1 MULTIPLE INTEGRALS

  4a cos θ
R π r2 2
sin θ
= π
2
4 2 0
 
1 R π2 16a2 cos2 θ
= π dθ
2 4 sin4 θ
R π
= 8a2 π
2
cot2 θ csc2 θdθ
4

  π2
− cot3 θ 8a2 8a2
= 8a2 =− [0 − 1] =
3 π 3 3
4

Example 1.46. Express the following integral in polar coordi-


R a R √a2 −x2 dxdy
nates and evaluate 0 √ax−x2 p
ax − x2 − y 2

√ √
Solution: The limits of y are ax − x2 and a2 − x2 . (i.e.)

O a x =a x
2

Fig. 30

56
1 MULTIPLE INTEGRALS

 a 2
Upper half of the circles i) x2 +y 2 −ax = 0; x − +(y−0)2 =
 a 2 2
and ii) x2 + y 2 = a2
2

To change the given integral to polar coordinates, we put x =


r cos θ, y = r sin θ and dxdy = rdrdθ.

The equations of the circles now become

i) r2 − ar cos θ = 0 (i.e.) r = a cos θ

ii) r2 = a2 (i.e.) r = a

Hence r changes from r = a cos θ to a and θ changes from 0 to


π
2
R π Ra rdrdθ
∴I= 0
2
a cos θ

a2 − r 2
R π √
= 2
0 [− a2 − r2 ]aa cos θ dθ
R π π
= 2
0 a sin θdθ = [a cos θ]02 = a
xydxdy RR
Example 1.47. Evaluate p where R is the region
R
x2 + y 2
in the first quadrant enclosed by the circles x2 + y 2 = a2 and
x2 + y 2 = 4a2 , by converting into polar coordinates.

Solution: We put x = r cos θ, y = r sin θ ∴ dxdy = rdrdθ r


π
varies from r = a to r = 2a and θ varies from 0 to
2

57
1 MULTIPLE INTEGRALS

O x

x 2 + y2 = a 2
x 2 + y 2 = 4a 2

Fig. 31

R 2a r3 sin θ cos θdrdθ


R π
Hence I = 0 a
2

r2
R  1 R π 
2a
= a r2 dr 2
sin 2θdθ
2 0
 2a  π
1 r3 − cos 2θ 2
=
2 3 0 2 0

1
=− (8a3 − a3 )(cos π − cos 0)
12
1 7
=− (7a3 )(−2) = a3
12 6
R∞R∞ 2 2
Example 1.48. Evaluate 0 0 e−(x +y ) dxdy by using polar
R ∞ −(x2 )
coordinates and hence evaluate 0 e dx.

58
1 MULTIPLE INTEGRALS

R∞R∞ 2 +y 2 ) RR −(x2 +y 2 ) dA,


Solution: 0 0 e−(x dxdy = Ae where A is

x
Fig. 32

the positive quadrant.

Put x = r cos θ, y = r sin θ ∴ dxdy = rdrdθ

The positive quadrant given the limits for r varies from 0 to ∞


π
and θ varies from 0 to
2
R∞R∞ 2 +y 2 ) R π R∞ 2
I= 0 0 e−(x dxdy = 2
0 0 e−(r ) rdθdr

Put r2 = R ⇒ 2rdr = dR
 
R π2 1 e−R ∞ 1 R π2
∴I= 0 dθ = dθ
2 −1 0 2 0

1 π π
= × =
2 2 4

59
1 MULTIPLE INTEGRALS

R∞R∞ 2 +y 2 ) R∞ 2 R∞ 2
Since 0 0 e−(x dxdy = 0 e−x dx + 0 e−y dy

= I × I = I2
π
I2 =
4

R∞ 2 π
∴I= 0 e−x dx =
2
R a R √a2 −x2
Example 1.49. Evaluate −a 0 (x2 +y 2 )dxdy by changing
to polar coordinates.

Solution: Put x = r cos θ, y = r sin θ ∴ dxdy = rdrdθ


R a R √a2 −x2
I= −a 0 (x2 + y 2 )dxdy
RπRa
= 0 0 r2 .rdrdθ
 a
r4 πa4
= [θ]π0 =
4 0 4
R 2a R √2ax−x2
Example 1.50. Evaluate 0 0 (x2 + y 2 )dydx.


Solution: Here y varies from 0 to 2ax − x2

(i.e.) y = 0 to x2 + y 2 = 2ax which is a circle centre (a, 0) and


radius a. Also the circle passes through (0, 0). Since x varies from
0 to 2a, the area of integration is the area above the x-axis and
inside the circle. The polar equation of the circle is x = 2a cos θ.
Changing to polar coordinates x = r cos θ, y = r sin θ

60
1 MULTIPLE INTEGRALS

r =a

q=p r =0 q=0

Fig. 33

y
r = a cos q

(a,0) (2a,0) x

Fig. 34

∴ dxdy = rdrdθ
R π/2 R 2a cos θ
I= 0 0 r2 rdrdθ

 2a cos θ
R π/2 r4
= 0 dθ
4 0

R π/2
= 4a4 0 cos4 θdθ

61
1 MULTIPLE INTEGRALS

3 1 π 3
= 4a4 . . = πa4
4 2 2 4

EXERCISE

Changing to polar coordinates evaluate the following:


RR h π i
1. x2 y 2 dxdy over the circle x2 + y 2 = a2 Ans: a6
24
R a R a xdxdy h πa i
2. 0 y x2 + y 2 Ans:
4
 
R a R x x3 dxdy a4 √
3. 0 0 p Ans: log(1 + 2)
x2 + y 2 4

R a R √a2 −x2 p
4. 0 0 y2  y dxdy over
x2 + 2
 the region bounded by
1
y = 0, x = y, x = 1 Ans: πa5
20

RR dxdy
5. over the region bounded by y = 0, x =
(x2 +h y 2 )3/2 i
π
y, x = 1 Ans:
12

1.5 Volume as a Triple Integral

Triple integrals can be used to evaluate volume V of a finite


region D in space.
RRR
The volume V = D dxdydz

62
1 MULTIPLE INTEGRALS

x2 y 2 z 2
Example 1.51. Find the volume of the ellipsoid 2 + 2 + 2 =
a b c
1 by triple integrals.

Solution:

Volume = 8× volume in the first octant.

Rcq
q
2
RaR 1− x2
a x2 y2
V =8× 0 0 0 1− a2
− b2
dzdydx

RaR
q
1− x2
2 q
x2 y2
=8× 0 0
a
[z]c0 1− a2
− b2

q "r #
x2 y 2
2
RaR 1− x2
a
= 8c 0 0 1− 2 − 2
a b
 
x2 2
Put r2 = 1− 2 b
a

8c R a R r p 2
= r − y 2 dydx
b 0 0
 r
8c R a r2 −1 y yp 2 2
= sin + r −y
b 0 2 r 2 0

2cπ R a 2
= r dx
b 0
 
2cπ R a x2 2
= 1 − 2 b dx
b 0 a

63
1 MULTIPLE INTEGRALS
 a
x3 4π
= 2cbπ x − 2 = abc
3a 0 3

Example 1.52. Find the volume bounded by the cylinder x2 +


y 2 = 4 and the planes y + z = 4 and z = 0.

Solution: z varies from z = 0 to z = 4 − y and x, y varies over

x
Fig. 35

all points of the circle x2 + y 2 = 4.


R 2 R √4−x2 R 4−y
Volume V = √
−2 − 4−x2 0 dzdydx

R 2 R √4−x2 4−y
= −2 − 4−x2 [z]0 dydx

R 2 R √4−x2
= −2 − 4−x2 (4
√ − y)dydx

64
1 MULTIPLE INTEGRALS

 √4−x2
R2 y2
= 4y − dx
−2 2 √
− 4−x2

R2√
=8×2 0 4 − x2 dx
 
x√ 2
4 −1 x
V = 16 4 − x + sin = 16π
2 2 2
Example 1.53. Find the volume of the tetrahedron bounded by
x y z
the coordinate planes and + + = 1
a b c

RRR
Solution: Volume required = dxdydz with limits.

x
Fig. 36

R a R b(1− xa ) R c(1− xa − yb )
= 0 0 0 dzdydx

65
1 MULTIPLE INTEGRALS

R a R b(1− xa )  x y
=c 0 0 1− − dydx
a b
 b 1− x
Ra  x y2 ( a )
=c 0 1− y− dx
a 2b 0
  
Ra x  2 b2  x 2
=c 0 b 1− − 1− dx
a 2b a

bc R a  x 2
= 1 − dx
2 0 a
 
x 2  
bc  1 −
=  a × −a  
2 3 1

abc
=
6
Example 1.54. Find the volume of the sphere x2 + y 2 + z 2 = a2
using the triple integrals.

Solution:

Since the sphere x2 + y 2 + z 2 = a2 is symmetric about the coor-


dinate planes, the volume of the sphere = 8 x volume of the first
octant.
RRR
=8× V dxdydz

" p #√a2 −x2


2 2 2 2
R a y (a − x ) − y a − x 2 y
=8 0 . sin−1 √ dx
2 2 a − x2 0
2

66
1 MULTIPLE INTEGRALS

Ra
=4 0 [(a2 − x2 )(sin−1 1 − sin−1 0)]dx

R a R √a2 −x2 R √a2 −x2 −y2


=8 0 0 0 dzdydx

R a R √a2 −x2 √
a2 −x2 −y 2
=8 0 0 [z]0 dydx

R a R √a2 −x2 p
=8 0 0 (a2 − x2 ) − y 2 dydx
Ra π
=4 0 (a2 − x2 ) dx
2
Ra
= 2π 0 (a2 − x2 )dx
 a
x3
= 2π a2 x −
3 0

2π 4π 3
= (3a3 − a3 ) = a
3 3

EXERCISE

1. Find the volume bounded by xy-plane, 2


 the cylinder x +
1
y 2 = 4 and the plane x + y + z = 3 Ans: (9π − 4)

2. Find the volume in the positive octant bounded  by the 


16
plane x + 2y + 3z = 4 and the coordinate planes. Ans:
9
RRR
3. Evaluate V dxdydz, where V is the volume enclosed
by the cylinder x2 + y 2 = 1 and the planes z = 0, z = 2 − x

67
1 MULTIPLE INTEGRALS
 
4
Ans:2π −
3

68
2 VECTOR CALCULUS

INTRODUCTION

A physical quantity, which is a function of the position of a point


in space, is called a scalar point function or a vector point
function according as the quantity is a scalar or vector. Tempe-
rature, velocity are respectively scalar and vector point function.
When a point function is defined at every point of certain region
of space, then that region is called field.

2.1 Vector Differential Operator ∆(del)

 
→∂
− →∂
− −
→∂
The operator ∆ is defined as ∆ = i + j + k
∂x ∂y ∂z

2.2 Gradient of a scalar point function

Let f (x, y, z) be defined and differentiable at each point (x, y, z)


in a certain region of space. Then the gradient of f , written as
∆f or gradf is defined as

69
2 VECTOR CALCULUS
 
→∂
− − ∂
→ →∂

∆f = i + j + k f
∂x ∂y ∂z
 
− ∂f −
→ → ∂f − → ∂f
= i + j + k
∂x ∂y ∂z

If f is a scalar point function, then ∆f is a vector point function.

2.3 Level Surface

Let f (x, y, z) be a scalar field over a region R. The points satis-


fying an equation of the type f (x, y, z) = c(arbitrary constant)
constitute a family of surfaces in three dimensional space. The
surfaces of this family are called level surfaces.

2.4 Directional Derivatives of a Scalar


Point Function

Definition: Let f (x, y, z) define a scalar field in a region R and


let P be any point in this region. Suppose Q is a point in this
region in the neighborhood of P in the direction of a given unit
f (Q) − f (P )
vector b
a. Then limQ→P , if it exits is called the di-
PQ
rectional derivative of f (x, y, z) at P in the direction of b a.

Result 1: The directional derivative of a scalar field f at a point


P (x, y, z) in the direction of a unit vector b
a is given by ∆f.b a.
The directional derivative is maximum in the direction of gradf
and its maximum value is |gradf |.

70
2 VECTOR CALCULUS

∆f
Result 2: Unit normal to a surface f = c is given by
|∆f |

Result 3: Angle between two surfaces φ1 = c, φ2 = c is given


∆φ1 .∆φ2
by cos θ =
|∆φ1 ||∆φ2 |

2.5 Divergence of a Vector Point Function

Definition: Let V be any given differentiable vector point func-


tion. Then the divergence
 of V written as ∆.V
 or divV is defined
→∂
− →∂
− −
→∂ P−→ ∂V
as divV = ∇V = i + j + k .V = i.
∂x ∂y ∂z ∂x


− →
− →
− ∂v1 ∂v2 ∂v3
If V = v1 i + v2 j + v3 k then divV = + + .
∂x ∂y ∂z

2.6 Solenoidal Vector

A vector V is said to be solenoidal if divV = 0.

2.7 Curl of a vector Point Function

Definition: Let F be a vector point function. Then curlF writ-


P− → ∂F
ten as ∇ × F is defined as curlF = i.
∂x

71
2 VECTOR CALCULUS

→ −
→ −

i j k


− →
− −
→ ∂ ∂ ∂
If F = F1 i + F2 j + F3 k then curlF =

∂x ∂y ∂z
F1 F2 F3

2.8 Irrotational Vector

A vector F is said to be irrotational if ∇ × F = 0. In this case


we can represent F = ∇φ, where φ is called scalar potential.

Note: 1 Physical meaning of ∇.V .



− →
− →

If V = v1 i + v2 j + v3 k is a vector point function representing
the instantaneous velocity of a moving fluid at the point (x, y, z),
then ∇.V represents rate of loss of fluid per unit volume at that
point.

Note: 2 Physical meaning of curlF .

If F represents the linear velocity of the point (x, y, z) of a ri-


gid body that rotates about a fixed axis with constant angular
velocity w, then curlF at that point represents 2w.
Example 2.1. If φ(x, y, z) = x2 y + y 2 x + z 2 , find ∇φ at the
point (1, 1, 1).

→ ∂φ P −
P− → 2
Solution: Let ∇φ = i. = i (x y + y 2 x + z 2 )
∂x

− →
− −

= i (2xy + y 2 ) + j (x2 + 2yx) + k (2z)

72
2 VECTOR CALCULUS


− →
− −→
∴ ∇φ(1,1,1) = i (2 + 1) + j (1 + 2) + k (2)


− →
− →

=3 i +3j +2k
Example 2.2. Find a unit normal vector to the level surface
x2 y + 2xz = 4 at the point (2, −2, 3).

∇φ
Solution: The unit normal vector n
b= , where φ = x2 y +
|∇φ|
2xz − 4.

P−
→ ∂φ P −→ 2
Now ∇φ = i. = i (x y + 2xz − 4)
∂x

− →
− →

= i (2xy + 2z) + j (x2 ) + k (2x)

− →
− −

∴ ∇φ(2,−2,3) = i (−8 + 6) + j (4) + k (4)


− →
− →

= −2 i + 4 j + 4 k

⇒ |∇φ| = 4 + 16 + 16 = 6

1−→ 2− → 2−→
∴n
b=− i + j + k
3 3 3
Example 2.3. Find the directional derivative of f (x, y, z) =
x2 yz + 4xz 2 at the point (1, −2, 1) in the direction of the vector
→ −
− → →

2 i − j −2k.

Solution: The required directional derivative is ∇f.b


a. Here a =
→ −
− → →

2 i − j −2k.

73
2 VECTOR CALCULUS


→ − → −

√ a 2 i − j −2k
So |a| = 4 + 1 + 4 = 3 and b
a= = .
|a| 3

P−
→ ∂f P−
→ ∂
Now gradf = ∇f = i. = i. = (x2 yz + 4xz 2 )
∂x ∂x

− →
− →

= i (2xyz + 4z 2 ) + j (x2 z) + k (x2 y + 8xz)

− →
− −

Hence (∇f )(1,−2,1) = i (8) + j (−1) + k (−10)


→ −
→ −


→ − → −
→ 2 i − j −2k
∴ The directional derivative = (8 i − j −10 k ). =
3
16 1 20 37
+ + =
3 3 3 3
Example 2.4. Find the directional derivative of f (x, y, z) =


x2 − 2y 2 + 4z 2 at the point (1, 1, −1) in the direction of 2 i +
→ −
− →
j − k.

Solution: The required directional derivative is ∇f.b


a.
→ →
− − −→ −
→ −→ − →
2i + j − k 1 2i + j − k
Here b
a= √ =√ (
4+1+1 6 )

→ ∂
− → ∂

Now ∇f = i . (x2 − 2y 2 + 4z 2 ) + j . (x2 − 2y 2 + 4z 2 ) +
∂x ∂y
→ ∂ 2

k . (x − 2y 2 + 4z 2 )
∂z

− →
− →

= i (2x) + j (−4y) + k (8z)

− →
− −

and (∇f )(1,1,−1) = 2 i − 4 j − 8 k

74
2 VECTOR CALCULUS

− → −
→ − →

→ −
→ −
→ 1 2i + j − k
Directional derivative = (2 i − 4 j − 8 k ). √ (
6 )

4−4+8 8
= √ =√
6 6
Example 2.5. Find the directional derivative of f (x, y, z) =
x2 − y 2 + 2z 2 at the point P(1, 2, 3) in the direction of the line
PQ, where Q is the point (5, 0, 4).

Solution: Given f (x, y, z) = x2 − y 2 + 2z 2

∂f −
→ ∂ −→ ∂ −→ −
→ −
→ −

Hence gradf = i + j + k = 2x i − 2y j + 4z k =
∂x ∂y ∂z

− →
− →

2 i − 4 j + 12 k at the point (1, 2, 3).

Also P Q = Position of Q - Position vector of P.



− →
− →
− →
− →
− −

= (5 i + 0 j + 4 k ) − ( i + 2 j + 3 k )
−−→
If b
a be the unit vector in the direction of P Q then

− → →
− − −
→ → −
− →
4 i −2j + k 4 i −2j + k
a= √
b = √
16 + 4 + 1 21

∴ The required directional derivative



→ → −
− →

→ −
→ −
→ 4 i −2j + k 28
= (gradf ).b
a = (2 i − 4 j + 12 k ). √ = √ =
21 21
28 4√
= 21
21 3
Example 2.6. In what direction from the point (1, 1, −1) is the

75
2 VECTOR CALCULUS

directional derivative of f (x, y, z) = x2 − 2y 2 + 4z 2 a maximum?


Also find the value of this maximum directional derivative.


− −
→ −
→ −
→ − → − →
Solution: We have gradf = 2x i −4y j +8z k = 2 i −4 j −8 k
at the point (1, 1, −1).

The directional derivative of f is a maximum in the direction of



− →
− →

gradf = 2 i − 4 j − 8 k

The maximum value of this directional derivative



− →
− →
− √ √
= |gradf | = |2 i − 4 j − 8 k | = 4 + 16 + 64 = 2 21

Example 2.7. Show that the vector field F given by F = (x2 −



− →
− −

yz) i + (y 2 − zx) j + (z 2 − xy) k is irrotational. Find a scalar
φ such that F = ∇φ.

Solution: We have

− →
− −

i j k

∂ ∂ ∂
curlF =

∂x ∂y ∂z
x2 − yz y 2 − zx z 2 − xy


− →
− −

= (−x + x) i − (−y + y) j + (−z + z) k = 0

∴ The vector field F is irrotational.

Let F = ∇φ (or)


− →
− −
→ ∂φ − → ∂φ − → ∂φ − →
(x2 − yz) i + (y 2 − zx) j + (z 2 − xy) k = i + j + k
∂x ∂y ∂z

76
2 VECTOR CALCULUS

∂φ x3
Then = x2 − yz whence φ = − xyz + f1 (y, z)
∂x 3

∂φ y3
= y 2 − zx whence φ = − xyz + f2 (x, z)
∂y 3

∂φ z3
= z 2 − xy whence φ = − xyz + f3 (x, y)
∂z 3

We choose φ satisfying all the above requirements.

x3 + y 3 + z 3
⇒φ= − xyz + C.
3


Example 2.8. Prove that the vector (y 2 cos x+z 3 ) i +(2y sin x−

− →

4) j + (3xz 2 + 2) k is irrotational and find its scalar potential.


→ −

Solution: Let F = (y 2 cos x + z 3 ) i + (2y sin x − 4) j + (3xz 2 +


2) k .

− −
→ −

i j k

∂ ∂ ∂
Then curlF =

∂x ∂y ∂z
y 2 cos x + z 3 2y sin x − 4 3xz 2 + 2

 

→ ∂ ∂
= i (3xz 2 + 2) − (2y sin x − 4)
∂y ∂z
 

− ∂ ∂ 2
−j (3xz 2 + 2) − (y cos x + z 3 )
∂x ∂z
 

→ ∂ ∂ 2
+k (2y sin x − 4) − (y cos x + z 3 )
∂x ∂y

77
2 VECTOR CALCULUS


− →
− →

= i (0 − 0) − j (3z 2 − 3z 2 ) + k (2y cos x − 2y cos x) = 0

⇒ F is irrotational.

If φ is scalar potential let F = ∇φ

P−
→ ∂φ
∴F = i
∂x


− −
→ −
→ ∂φ − →
⇒ (y 2 cos x + z 3 ) i + (2y sin x − 4) j + (3xz 2 + 2) k = i +
∂x
∂φ →
− ∂φ −→
j + k
∂y ∂z

∂φ ∂φ ∂φ
⇒ = y 2 cos x + z 3 ; = 2y sin x − 4; = 3xz 2 + 2
∂x ∂y ∂z

Integrating partially, we get

φ = y 2 sin x + z 3 x + f1 (y, z)

φ = y 2 sin x − 4y + f2 (x, z)

φ = xz 3 + 2z + f3 (x, y)

∴ φ = y 2 sin x + z 3 x − 4y + 2z + C
Example 2.9. Find the angle between the surfaces x2 + yz = 2
and x + 2y − z = 2 at (1, 1, 1).

Solution: The first surface is φ1 = x2 + yz − 2 = 0


 
− ∂
→ →∂
− →∂
− P−
→ ∂V 2
Then ∇φ1 = i + j + k .V = i. (x + yz −
∂x ∂y ∂z ∂x

78
2 VECTOR CALCULUS

2)

→∂ 2
− →∂ 2
− −
→∂ 2
= i (x + yz − 2) + j (x + yz − 2) + k (x + yz − 2)
∂x ∂y ∂z

− →
− →

= i (2x) + j (z) + k (y)
→ −
− → − →
= 2 i + j + k at (1, 1, 1).

The second surface φ2 = x + 2y − z − 2 = 0.


 
→∂
− →∂
− −
→∂ P−
→ ∂V
Then ∇φ2 = i + j + k .V = i. (x + 2y −
∂x ∂y ∂z ∂x
z − 2)

→∂
− →∂
− −
→∂
= i (x+2y−z−2)+ j (x+2y−z−2)+ k (x+2y−z−2)
∂x ∂y ∂z

− →
− →

= i (1) + j (2) + k (−1)

− → −
− →
= i + 2 j − k at (1, 1, 1).

Let ′ θ′ be the angle between the surfaces φ1 and φ2 .

∆φ1 .∆φ2
∴ cos θ =
|∆φ1 ||∆φ2 |
→ −
− → − → − → → −
− →
(2 i + j + k )( i + 2 j − k ) 3 1
= √ √ = =
6 6 6 2
 
1 π
⇒θ= cos−1 =
2 3

79
2 VECTOR CALCULUS

π
∴θ=
3
Example 2.10. Find a and b so that the surfaces ax2 − byz =
(a + 2)x and 4x2 y + z 3 = 4 cut orthogonally at (1, −1, 2).

Solution:

Here φ1 = ax2 − byz − (a + 2)x = 0 (2.1)

and φ2 = 4x2 y + z 3 − 4 = 0 (2.2)


→ ∂φ1 −
− → ∂φ1 −→ ∂φ1
Now ∇φ1 = i + j + k
∂x ∂y ∂z

− →
− −

= (2ax − a − 2) i + (−bz) j + (−by) k

→ −
→ −

⇒ (∇φ1 ) at (1, −1, 2) = (2a − a − 2) i − 2b j + b k

→ ∂φ2 −
− → ∂φ2 −→ ∂φ2 −
→ −
→ −

and ∇φ2 = i + j + k = 8xy i + 4x2 j + 3z 2 k
∂x ∂y ∂z

− −
→ −

⇒ (∇φ2 ) at (1, −1, 2) = −8 i + 4 j + 12 k

Since ∇φ1 .∇φ2 = 0, [∵ surfaces intersect orthogonally]



− →
− →
− −
→ −
→ −

we get [(a − 2) i − 2b j + b k ].(−8 i + 4 j + 12 k )

(i.e.) −8(a − 2) − 8b + 12b = 0 (or)

−2a + 4 + b = 0 (2.3)

Also the point (1, −1, 2) lies on φ1 = 0

80
2 VECTOR CALCULUS

Putting x = 1, y = −1, z = 2 in 2.1, we get

a + 2b − (a + 2) = 0 ⇒ b = 1 (2.4)
5
Putting b = 1 in 2.3, we get −2a + 5 = 0 ⇒ a =
2
Example 2.11. Find the angle between the surfaces z = x2 +
y 2 − 3 and x2 + y 2 + z 2 = 9 at the point (2, −1, 2).

Solution: Let ∇φ1 = x2 + y 2 − z − 3 and ∇φ2 = x2 + y 2 + z 2 − 9.

→ ∂φ1 −
− → ∂φ1 −→ ∂φ1
Now ∇φ1 = i + j + k
∂x ∂y ∂z

− →
− →

= i (2x) + j (2y) − k

− → −
− →
⇒ (∇φ1 ) at (2, −1, 2) = 4 i − 2 j − k

→ ∂φ2 −
− → ∂φ2 −→ ∂φ2 −
→ −
→ −

and ∇φ2 = i + j + k = 2x i + 2y j + 2z k
∂x ∂y ∂z

− →
− −

⇒ (∇φ2 )(2,−1,2) = 4 i − 2 j + 4 k

If ′ θ′ be the angle between the surfaces φ1 and φ2 then cos θ =


∆φ1 .∆φ2
|∆φ1 ||∆φ2 |

− − −
→ → − → −
→ −

(4 i − 2 j − k )(4 i − 2 j + 4 k )
cos θ = √ √
16 + 4 + 1 16 + 4 + 16

16 + 4 − 4 8
= √ √ = √
21 36 3 21

81
2 VECTOR CALCULUS
 
8
∴θ= cos−1 √
3 21
Example 2.12. Find the equation of the tangent plane to the
surface xz 2 + x2 y = z − 1 at the point (1, −3, 2).

Solution: Let φ = xz 2 + x2 y − z + 1 = 0

P−
→ ∂φ −→ −
→ −

∴ ∇φ = i = i (z 2 + 2xy) + j (x2 ) + k (2xz − 1)
∂x
→ −
− → →

(∇φ)(1,−3,2) = −2 i + j + 3 k

Direction ratios of the normal to the plane is (−2, 1, 3).

Required tangent plane passes through (1, −3, 2).

⇒ Its equation is −2(x − 1) + (y + 3) + 3(z − 2) = 0.

(or) −2x + y + 3z + 2 + 3 − 6 = 0

(or) 2x − y − 3z + 1 = 0

Example 2.13. Find the constants a, b, c so that F = (x + 2y +



− →
− −

az) i + (bx − 3y − z) j + (4x + cy + 2z) k may be irrotational.
For these values of a, b, c, find its scalar potential.

Solution: We know that F is irrotational if curlF = 0.



− −
→ −

i j k

∂ ∂ ∂
∇ × F = 0 ⇒ =0

∂x ∂y ∂z
x + 2y + az bx − 3y − z 4x + cy + 2z

82
2 VECTOR CALCULUS
 

→ ∂ ∂
⇒ i (4x + cy + 2z) − (bx − 3y − z)
∂y ∂z
 

→ ∂ ∂
−j (4x + cy + 2z) − (x + 2y + az)
∂x ∂z
 

− ∂ ∂
+k (bx − 3y − z) − (x + 2y + az) = 0
∂x ∂y

− →
− →

⇒ i (c + 1) − j (4 − a) + k (b − 2) = 0

(i.e.) c + 1 = 0, 4 − a = 0, b − 2 = 0

(i.e.) a = 4, b = 2 and c = −1

Using these values in F , we get



− −
→ −

F = (x + 2y + 4z) i + (2x − 3y − z) j + (4x − y + 2z) k

Let φ be the scalar potential. We consider F = ∇φ.


− −
→ → P−
− → ∂φ
∴ (x + 2y + 4z) i + (2x − 3y − z) j + (4x − y + 2z) k = i
∂x

→ ∂φ
− → ∂φ
− →
− ∂φ
⇒ i = x + 2y + 4z; j = 2x − 3y − z; k = 4x − y + 2z
∂x ∂y ∂z

x2 y2
Integrating, φ = + 2yzx + 4zx + f1 (y, z), φ = 2xy − 3 −
2 2
yz + f2 (x, z) and φ = 4xz − yz + z 2 + f3 (x, y)

We must choose φ, satisfying the above condition.

83
2 VECTOR CALCULUS

x2 y2
∴φ= − 3 + z 2 + 2xy + 4zx − yz + C
2 2

− −
→ −

Example 2.14. If F = 3x2 i +5xy 2 j +xyz 3 k , find ∇F , ∇(∇F ), ∇×
F , ∇(∇ × F ) and ∇ × (∇ × F ) at the point (1, 2, 3).

∂ ∂ ∂
Solution: Let ∇.F = (F1 ) + (F2 ) + (F3 )
∂x ∂2 ∂z

Here F1 = 3x2 , F2 = 5xy 2 , F3 = xyz 3

∂ ∂
∴ ∇F = df rac∂∂x(3x2 ) + (5xy 2 ) + (xyz 3 )
∂2 ∂z

= 6x + 10xy + 3xyz 2

⇒ (∇F )(1,2,3) = 6 + 20 + 54 = 80.

P−
→∂
Also ∇(∇F ) = i (∇F )
∂x
P−
→∂
= i (6x + 10xy + 3xyz 2 )
∂x

− →
− −

= i (6 + 10y + 3yz 2 ) + j (10x + 3xz 2 ) + k (6xyz)

− −
→ −

⇒ ∇(∇.F )(1,2,3) = i (6 + 20 + 54) + j (10 + 27) + k (36)


− →
− →

= 80 i + 37 j + 36 k

84
2 VECTOR CALCULUS

→ −
→ −

i j k

∂ ∂ ∂
Now ∇ × F = curlF =

∂x ∂y ∂z
3x2 5xy 2 xyz 3


− →
− →

= i (xz 3 ) − j (yz 3 ) + k (5y 2 )

− →
− −

(∇ × F )(1,2,3) = 27 i − 54 j + 20 k

∂ ∂ ∂
Also ∇(∇ × F ) = divcurlF = (xz 3 ) + (−yz 3 ) + (5y 2 )
∂x ∂y ∂z

= z3 − z3 = 0

→ −
→ −

i j k

∂ ∂ ∂
and ∇ × (∇ × F ) = curlcurlF ) =

∂x ∂y ∂z
xz 3 −yz 3 5y 2


− →
− −

= i (10y + 3z 2 y) − j (−3xz 2 ) + k (0)

→ −
→ −
→ −

(∇ × (∇ × F ))(1,2,3) = (20 + 54) i − (−27) j = 74 i + 27 j

Example 2.15. If − →r is the position vector of the point (x, y, z)


with respect to the origin, prove that div −

r = 3, curl−

r = 0, ∇(rn ) =
nr n−2 →

r


− −
→ −

Solution: Let →

r =x i +y j +zk

∂ ∂ ∂
Then div −

r = (x) + (y) + (z)
∂x ∂y ∂z

85
2 VECTOR CALCULUS

→ −
→ −

i j k


− →
− ∂ ∂ ∂
Also curl r = ∇ × r =

∂x ∂y ∂z
x y z

   

→ ∂ ∂ → ∂
− ∂
= i (z) − (y) − j (z) − (x)
∂y ∂z ∂x ∂z
 

→ ∂ ∂
+k (y) − (z) = 0
∂x ∂y

P→
− ∂ n P−
→ n−1 ∂r
and ∇(rn ) = i (r ) = i nr
∂x ∂x

− −
→ →
− p
Here →

r = x i + y j + z k ⇒ r = x2 + y 2 + z 2 = | −

r|

(i.e.) r2 = x2 + y 2 + z 2 .

Differentiating partially w.r.to x, we get

∂r ∂r x
2r = 2x ⇒ = .
∂x ∂x r

∂r y ∂r z
Similarly, = , =
∂y r ∂z r
P−
→ n−1 x
Now grad(rn ) = ∇(rn ) = i nr
r

nrn−1 −→ −
→ →

= (x i + y j + z k ) = nrn−2 −

r
r
Example 2.16. Prove that i) curl gradφ = 0 ii) div curlF =
∇(∇ × F ) = 0 iii) div(φu) = φdivu + ∇φ.u

86
2 VECTOR CALCULUS

→ ∂φ −
− → ∂φ −→ ∂φ
Solution: (i) We know that gradφ = ∇φ = i +j +k
∂x ∂y ∂z

→ −
→ −

i j k

∂ ∂ ∂

∴ curl gradφ = ∇ × (∇φ) = ∂x ∂y ∂z


∂φ ∂φ ∂φ

∂x ∂y ∂z

     

→ ∂2φ ∂2φ → ∂2φ
− ∂2φ → ∂2φ
− ∂2φ
= i − −j − +k − =
∂y∂z ∂z∂y ∂x∂z ∂z∂x ∂x∂y ∂x∂y
0

− →
− →

(ii) Let F = F1 i + F2 j + F3 k

→ −
→ −

i j k

∂ ∂ ∂
∴ curl F = (∇ × F ) =

∂x ∂y ∂z
F1 F2 F3

     
− ∂F3 ∂F2
→ → ∂F3 ∂F1
− −
→ ∂F2 ∂F1
= i − − j − + k −
∂y ∂z ∂x ∂z ∂x ∂y

Now div curl F = ∇.(∇ × F )


     
∂ ∂F3 ∂F2 ∂ ∂F1 ∂F3 ∂ ∂F2 ∂F1
= − + − + −
∂x ∂y ∂z ∂y ∂x ∂z ∂z ∂x ∂y

∂ 2 F3 ∂ 2 F2 ∂ 2 F1 ∂ 2 F3 ∂ 2 F2 ∂ 2 F1
= − + − + −
∂x∂y ∂x∂z ∂y∂z ∂y∂x ∂z∂x ∂z∂y

=0

87
2 VECTOR CALCULUS

P−
→ ∂F
(iii) We know that divF = i.
∂x
P−→ ∂
⇒ (φu) = i . (φu|)
∂x
 
P−
→ ∂φ ∂u
= i. u +φ
∂x ∂x
     
P →
− ∂φ P −
→ ∂u
= i. .u + i . φ.
∂x ∂x
   
P−
→ ∂φ P−
→ ∂u
= i .u + φ i
∂x ∂x

= ∇φ.u + φ div u
Example 2.17. Find the value of n, if rn r is both solenoidal

− −
→ −

and irrotational, when r = x i + y j + z k

Solution:

Consider div(rn r) = ∇rn .r + rn divr (2.5)

[∵ div(φu) = φdivu + ∇φ.u]

We know that divr = 3, ∇rn = nrn−2 r

From (2.5), div rn r = (nrn−2 r.r) + rn (3)

= nrn−2 r2 + 3rn = nrn + 3rn = (n + 3)rn

88
2 VECTOR CALCULUS

⇒ rn r is solenoidal means div rn r = 0, which is possible only


when n = −3.

We know that curl(φu) = ∇φ × u + φcurlu

∴ curl(rn r) = ∇rn × r + rn curlr

= nrn−2 (r × r) + rn .0 = 0 (as r × r = 0, curlr = 0)

⇒ rn r is irrotational if curl(rn r) = 0, (i.e.) whatever be the


value of n.

Hence rn r is solenoidal as well as irrotational when n = −3.


Example 2.18. Prove that f (r)r is an irrotational vector.

Solution:

Consider curl(f (r)r) = ∇f (r)r × r + f (r)curlr (2.6)


P−
→∂ P−
→ ′ ∂r
∴ ∇f (r) = i f (r) = i f (r)
∂x ∂x

P−
→ ′ x f ′ (r) −→ −
→ −
→ f ′ (r)
= i f (r) = (x i + y j + z k ) = r
r r r

Further curlr = 0.

f ′ (r)
From (2.6), curl(f (r)r) = (r × r) + f (r).0 = 0 + 0 = 0.
r

⇒ f (r)r is an irrotational vector.


Example 2.19. Prove that ∇2 rn = n(n + 1)rn−2 where r =

− −
→ →

x i +y j +zk.

89
2 VECTOR CALCULUS

Solution: We know that ∇2 φ = ∇(∇φ).

Hence ∇2 rn = ∇(∇rn ) = ∇(nrn−2 r).

Since ∇(φu) = ∇φ.u + φ div u

We have ∇(nrn−2 r) = ∇nrn−2 .r + nrn−2 div r (2.7)


P−→∂ P−
→ ∂r
Now ∇(nrn−2 ) = i nrn−2 = i n(n − 2)rn−3
∂x ∂x
X−
→ x X −

= i n(n − 2)rn−3 = n(n − 2)rn−4 x i (2.8)
r
Now put (2.8) in (2.7), we have

∇(nrn−2 ) = n(n − 2)rn−4 r.r + nrn−2 .3

= nrn−2 (n − 2 + 3) = n(n + 1)rn−2


Example 2.20. Find the angle between the normals to the sur-
face x2 = yz at the points (1, 1, 1) and (2, 4, 1).

Solution: Let n1 = unit normal to x2 − yz = 0 at (2, 4, 1).

∇φ
(i.e.) n1 = .
|∇φ|

P−
→∂ 2
∇φ = i (x − yz) as φ = x2 − yz
∂x

− →
− →

= i (2x) + j (−z) + k (−y)
→ −
− → −

at (2, 4, 1) ⇒ n1 = 4 i − j − 4 k

90
2 VECTOR CALCULUS

P−
→∂ 2
i (x − yz)
Similarly n2 = ∂x at (1, 1, 1).
|∇φ|
→ −
− → − →
∴ n2 = 2 i − j − k

If ′ θ′ is the required angle then


→ −
− → −
→ −
→ − → − →
n1 .n2 (4 i − j − 4 k ) (2 i − j − k )
cos θ = = √ . √
|n1 ||n2 | 16 + 1 + 16 4+1+1

8+1+4 13
= √ √ =√
33 6 198
 
13
∴ θ = cos−1 √
3 22

− −
→ −

Example 2.21. Prove that A = (2x + yz) i + (4y + zx) j −


(6z − xy) k is solenoidal as well as irrotational. Also find the


scalar potential of A .

Solution: We know that


→ ∂ ∂ ∂
div A = (2x + yz) + (4y + zx) + (−6z + xy)
∂x ∂y ∂z

=2+4−6=0


∴ A is solenoidal.

91
2 VECTOR CALCULUS

− →
− −

i j k


− ∂ ∂ ∂
curl A ) =

∂x ∂y ∂z
2x + yz 4y + zx −6z + xy

 

→ ∂ ∂
= i (−6z + xy) − (4y + zx)
∂y ∂z
 

→ ∂ ∂
−j (−6z + xy) − (2x + yz)
∂x ∂z
 

− ∂ ∂
+k (4y + zx) − (2x + yz)
∂x ∂y

− →
− →

= i (x − x) − j (y − y) + k (z − z) = 0


∴ A is irrotational.

→ −

Let φ(x, y, z) be the scalar potential of A . Then A = ∇φ =
→ ∂φ −
− → ∂φ → − ∂φ
i + j + k
∂x ∂y ∂z

→ ∂φ −
− → ∂φ −→ ∂φ −
→ −

Hence i +j +k = (2x + yz) i + (4y + zx) j − (6z −
∂x ∂y ∂z


xy) k
→ →
− − − →
Equating the components by i , j , k
∂φ
= 2x + yz (2.9)
∂x
∂φ
= 4y + zx (2.10)
∂y

92
2 VECTOR CALCULUS

∂φ
= −6z + xy (2.11)
∂z
Partially integrating (2.9), (2.10) and (2.11) w.r.to x, y, z respec-
tively, we get

φ = x2 + xyz + a constant independent of x

φ = 2y 2 + xyz + a constant independent of y

φ = −3z 2 + xyz + a constant independent of z

Hence a possible form of φ is φ = x2 + 2y 2 − 3z 2 + xyz + a.


Example 2.22. Prove that the directional derivative of φ =

→ − → − →
x3 y 2 z at (1, 2, 3) is maximum along the direction 9 i +3 j + k .
Also find the maximum directional derivative.

Solution: Given φ = x3 y 2 z
 
− ∂
→ →∂
− →∂

Hence ∇φ = i + j + k (x3 y 2 z)
∂x ∂y ∂z

→∂ 3 2
− →∂ 3 2
− −
→∂ 3 2
= i (x y z) + j (x y z) + k (x y z)
∂x ∂y ∂z

− →
− −

= (3x2 y 2 z) i + (2x3 yz) j + (x3 y 2 ) k

− →
− →
− →
− → −
− →
= 36 i + 12 j + 4 k = 4( i + 3 j + k )

We know that the directional derivative of φ is maximum along


the direction ∇φ.

→ → −
− →
Hence it is maximum along the direction 4( i + 3 j + k ). (i.e.)

93
2 VECTOR CALCULUS


− → −
− →
along ( i + 3 j + k ).
√ √
Magnitude of this vector = 4 92 + 32 + 12 = 4 91 and this is
the maximum directional derivative.

EXERCISE

1. Find the unit normal " to the surface x2 +#2y 2 + z 2 = 7 at



→ −
→ −

i −2j +3k
the point (1, -1, 2). Ans:
3


− →
− −

2. If ∇φ = yz i + zx j + xy k , find φ. [Ans : φ = xyz + c]

3. If φ(x, y, z) = x2 y − 2y 2 z 3 , find ∇φ at the point (1, -1, 2).



− →
− →

[Ans : −2 i + 33 j − 24 k ]

→ −

4. Show that the vector →

v = (x + 3y) i + (y − 3z) j + (x −


2z) k is solenoidal.

− −
→ −

5. Show that the vector A = 3x2 y i + (x3 − 2yz 2 ) j + (3z 2 −


2y 2 z) k is irrotational.

6. If a is a constant, find div(r × a). [Ans: 0]

7. Determine the constant ′ a′ so that the vector − →


v = (x +

− →
− −

3y) i + (y − 2z) j + (x + az) k is solenoidal. [Ans: a = -2]

→ −
→ −

8. Evaluate divf where f = 2x2 z i − xy 2 z j + 3y 2 x k [Ans :
2xz(2 − y)]

94
2 VECTOR CALCULUS

9. If u = x2 − y 2 + 4z, show that ∇u = 0.



− −
→ →
− −
→ −

10. If F = x2 y i + xz j + 2yz k , verify that div curl F = 0.

11. Find the equation of the tangent plane to the surface z =


x2 + y 2 at the point (2, -1, 5). [Ans : 4x − 2y − z = 5]

→ → −
− → −
→ −

12. If F = (x + y + 1) i + j − (x + y) k , show that F is


perpendicular to curl F

13. Find the maximum directional


√ derivative of φ = x3 y 2 z at
the point (1, 1, 1) [Ans : 14]

− −
→ −

14. Find the value of a, if F = (axy − z 3 ) i + (a − 2)x2 j +


(1 − a)xz 2 k is irrotational.

15. If u and v are irrotational, prove that (u × v) is solenoidal.



− −
→ −
→ −

16. Show that F = (sin y + z) i + (x cos y − z) j + (x − y) k
is irrotational.

17. When is a vector said to be (i) solenoidal, (ii) irrotational?

18. Find the unit normal to 4 2


 the surface x − 3xyz + z + 1 = 0
1 − → → −
− →
at the point (1, 1, 1). Ans: √ ( i − 3 j − k )
11

→ −
→ −

19. If r = |−→
r |, where →

r = x i + y j + z k , prove that


r
∇ log |−

r|= 2
r

20. Find grad f , where f is given by f = x3 − y 3 + xz 2 at the

95
2 VECTOR CALCULUS


− −
→ −

point (1, -1, 2). [Ans : 7 i − 3 j + 4 k ]

21. What is the greatest rate of increase of u = xyz 2 at the


point (1, 0, 3)? [Ans: 9]

22. Prove that grad f (u) = f ′ (u) gradu

23. Find the directional derivative of the function φ = xy 2 +


yz 3 at the point (2, -1, 1) in the direction of the nor-
mal to the surface 2
  x log z − y + 4 at the point (-1, 2, 1).
15
Ans: √ units
17

24. Find the constants a and b, so that the surfaces 5x2 −2yz −
9x = 0 and ax2 y + bz 3 = 4 may cut orthogonally at the
point (1, -1, 2). [Ans: a=4, b=1]

25. Find the angle between the surfaces x2 − y 2 − z 2 = 11 and


xy + yz − zx = 18 at the point (6, 4, 3).
  
−24
Ans: θ = cos−1 √
5246

→ −

26. Show that − →
u = (2x2 +8xy 2 z) i +(3x3 y−3xy) j −(4y 2 z 2 +


v = xyz 2 −
2x3 z) k is not solenoidal, but −
→ →
u is solenoidal.

27. Find the directional derivative of φ = 2xy + z2 at the point



→ −
→ −
→ 14
(1, -1, 3) in the direction of i + 2 j + 2 k Ans:
3

→ →
− −

28. If ∇φ = 2xyz 3 i + x2 z 3 j + 3x2 yz 2 k , find φ(x, y, z) given
that φ(1, −2, 2) = 4. [Ans : φ = x2 yz 3 + 20]

96
2 VECTOR CALCULUS

29. If r = |− →r |, where −

r is the position vector of the point
(x, y, z) with respect to the origin, prove that ∇2 f (r) =
2
f ′′ (r) + f ′ (r)
r

30. Find the directional derivative of φ = xy + yz +


 zx in the


− →
− −
→ 10
direction of vector i + 2 j + 2 k at (1, 2, 0). Ans:
3

31. Find the angle of intersection at (4, -3, 2) of spheres x2 +


y 2 + z 2 = 29pand x2 + y 2 + z 2 + 4x − 6y − 8z − 47 = 0.
[Ans : cos−1 19/29]

2 +y 2 +z 2 )1/2
32. Find ∇φ and |∇φ| when φ = (x2 + y 2 + z 2 )e−(x
[Ans : (2 − r)e−r −

r , (2 − r)e−r r]

33. If u = 3x2 y and v = xz 2 −2y, then find grad[(gradu).(gradv)]



− −
→ −

[Ans : (6yz 2 − 4x) i + 6xz 2 j + 12xyz k ]

− →
− −

34. Find div f and curl f where f = grad(x3 + y 3 + z 3 −

− −

3xyz) [Ans : div f = 6(x + y + z).curl f = 0]

35. Find the directional derivative of φ = xy 2 +yz 3 at the point


 1) in thedirection of PQ where Q is the point (3,
P (2, −1,
11
1, 3). Ans: −
3

36. Find the values of λ and µ if the surfaces λx2 − µyz =


(λ + 2)x and 2 3
 4x y + z = 4 cut orthogonally at the point
5
(1, -1, 2). Ans:λ = , µ = 1
2

97
2 VECTOR CALCULUS


− −
→ −

37. Prove that yz 2 i +(xz 2 −1) j +2(xyz −1) k is irrotational
and find its scalar potential. [Ans : xy 2 − y − z + c]

38. Find the angle between the surfaces 2 and


 x log z = y − 1
1
x2 y = 2 − z at the point (1, 1, 1). Ans: cos−1 √
30

39. Find the directional derivative of φ = 2x2 + 3y 2 2


 + z at (2,

→ −
→ 4
1, 3) in the direction of the vector i − 2 k . Ans: − √
5

− −
→ −

40. Prove that ex [(2y + 3z) i + 2 j + 3 k ] is irrotational and
find its scalar potential. [Ans : ex (2y + 3z) + c]


41. Given the vector field −→
v = (x2 − y 2 + 2xz) i + (xz −

− →

xy + yz) j + (z 2 + x2 ) k ]. Find curl− →v . Show that the


vectors given by curl v at P0 (1, 2, −3) and P1 (2, 3, 12) are
orthogonal.

→ −

42. A fluid motion is given by −
→v = (y + z) i + (z + x) j + (x +


y) k . Is this motion irrotational? If so, find the velocity
potential. [Ans: yes, φ = xy + yz + zx]

43. In what direction from (3, 1, -2) is the directional deriva-


tive φ = x2 y 2 z 4 is maximum?
√ Find the magnitude of this
maximum. [Ans : 96 19]

→ −
→ −

44. Establish the relation: curl curl f = grad div f − ∇2 f .

98
2 VECTOR CALCULUS

2.9 Integration of Vectors

Line Integral


Let F (x, y, z) be a vector function and a curve AB. Line integral

T
B P

O X
Fig.1



of a vector function F along the curve AB is defined as the


integral component of F along the tangent to the curve AB.

− −

Component of F along a tagnent PT at P = Dot product of F
→ dr dr

and unit vector along P T = F . [ is a unit vector along
ds ds

99
2 VECTOR CALCULUS

tangent PT]

P−→ d−
→r
Line integral = F. from A to B along the curve.
ds
 
R → d−
− →r R −→ →
∴ Line integral = C F . ds = C F .d−
r
ds

− −
→ →
− −
→ −
→ −

Since →

r = x i + y j + z k , therefore d−

r = dx i + dy j + dz k
→ →
− →
− →
− →
− −
→ −
→ −

∴ F .d−
r = (F1 i + F2 j + F3 k ).(dx i + dy j + dz k )

= F1 dx + F2 dy + F3 dz

Therefore in components from the above line integral is written


as
R →− − → R
C F .d r = C (F1 dx + F2 dy + F3 dz)

R R −→
Note 1: C φdr, where φ is a scalar point functions and C F ×
d−

r are also line integral.


Note 2: If F represents the variable force acting on a particle
RB−→ →
along arc AB, then the total work done = A F .d− r


− H −→ →
Note 3: If F represents the velocity of a liquid then C F .d−
r


is called the circulation of F round the curve C.
H −
→ →
Note 4: If C F .d−
r = 0, then the field F is called consecutive.


(i.e.) ∇ × F = 0

100
2 VECTOR CALCULUS

RB→− − →
Note 5: If A F .d r is said to be independent of path then


F = ∇φ.

2.9.1 Surface Integral



Let vector F be a vector point function and S be the given


surface. Surface integral of a vector point function F over the


surface S is defined as the integral of the component F along


the normal to the surface. Component of F along the normal

n
F

ds

Fig.2



= F .n where n is the unit normal vector to an element dS and
grad f dxdy
n
b= , dS =
|grad f | (b
n.k)

− −
→ RR − →
Surface integral of F over S = F .b
n= S ( F .b
n)ds

101
2 VECTOR CALCULUS

2.9.2 Volume Integral



Let vector F be a vector point function and volume V enclosed
by a closed surface.
RRR → −
The volume integral = V F dv
R −→ → −
→ −
→ −

Example 2.23. Evaluate C F .d− r , where F = x2 i + y 3 j
and curve C is the arc of the parabola y = x2 in the x - y plane
from (0, 0) to (1, 1).


→ −

Solution: In the x - y plane, we have −

r =x i +y j

− −

∴ d−

r = dx i + dy j
→ →
− →
− →
− →
− −
→ R−
→ → R
∴ F .d−
r = (x2 i +y 3 j ).(dx i +dy j ) ⇒ F .d−
r = C (x2 dx+
3
y dy)

Now along the C, y = x2 ⇒ dy = 2xdx


R →− − → R1 2 6
R1 2 7
∴ C F .d r = 0 [x dx + x (2x)dx] = 0 (x + 2x )dx

 1
x3 2x8 7
= + = .
3 8 0 12

− −
→ −
→ −

Example 2.24. If F = (3x2 + 6y) i − 14yz j + 20xz 2 k , eva-
R −→ →
luate C F .d−
r , where C is the straight line joining (0, 0, 0) to
(1, 1, 1).

Solution: The equation of the straight line joining (0, 0, 0) and


(1, 1, 1) are

102
2 VECTOR CALCULUS

x−0 y−0 z−0


= = = t (say)
1−0 1−0 1−0

Then along C, x = t, y = t, z = t

− −
→ → →
− −
→ − → − →
Also →

r = x i + y j + z k d−
r = ( i + j + k )dt

− →
− −
→ −

Also along C, F = (3t2 + 6t) i − 14t2 j + 20t3 k )

At (0, 0, 0), t = 0 and at (1, 1, 1), t = 1


R →− − → R t=1 2 2 3
∴ C F .d r = t=0 [(3t + 6t) − 14t + 20t ]dt

 1
t3 t2 t3 t4 14 13
= 3 + 6 − 14 + 20 =1+3− +5=
3 2 3 4 0 3 3
Example 2.25. Find the total work done in moving a particle

− −
→ −
→ −

in a force field given by F = 3xy i − 5z j + 10x k along the
curve x = t2 + 1, y = 2t2 , z = t3 from t = 1 to t = 2.

Solution: Let C denote the arc of the given curve from t = 1 to


t = 2.
R →− − →
Then the total work done C F .d r

R −
→ →
− →
− −
→ −
→ −

= C (3xy i − 5z j + 10x k ).(dx i + dy j + dz k )
R
= C (3xydx − 5zdy + 10xzdz)
 
R2 dx dy dz
= 1 3xy − 5z + 10x dt
dt dt dt

103
2 VECTOR CALCULUS

R2
= 1 [3(t2 + 1)(2t)2 (2t) − (5t3 )(4t) + 10(t2 + 1)(3t2 )]dt
R2
= 1 (12t5 + 12t3 − 20t4 + 30t4 + 30t2 )dt
R2
= 1 (12t5 + 10t4 + 12t3 + 30t2 )dt
 2  2  2  2
t6 t5 t4 t3
= 12 + 10 + 12 + 30
6 1 5 1 4 1 3 1

= 303

− −
→ −
→ −

Example 2.26. Show that F = (2xy + z 3 ) i − x2 j + 3xz 2 k
is a conservative force field. Find the scalar potential. Find also
the work done in moving an object in this field from (1, -2, 1) to
(3, 1, 4).



Solution: The field F will be conservative if ∇ × F = 0.

− −
→ →

i j k


− ∂ ∂ ∂
We have ∇ × F =
∂x ∂y ∂z
2xy + z 3 x 2 3xz 2
   

→ ∂ 2 ∂ 2 → ∂
− 2 ∂ 3
= i (3xz ) − (x ) − j (3xz ) − (2xy + z )
∂y ∂z ∂x ∂z
 

→ ∂ 2 ∂
+k (x ) − (2xy + z 3 )
∂x ∂y

− →
− →

= i (0 − 0) − j (3z 2 − 3z 2 ) + k (2x − 2x) = 0


Therefore F is conservative force field.

104
2 VECTOR CALCULUS


− →
− −
→ −

Let F = ∇φ ⇒ (2xy + z 3 ) i − x2 j + 3xz 2 k

∂φ −
→ ∂φ − → ∂φ − →
= i + j + k
∂x ∂y ∂z

∂φ
Then = 2xy + z 3 whence φ = x2 y + z 3 x + f1 (y, z) (2.12)
∂x
∂φ
= x2 whence φ = x2 y + f2 (x, z) (2.13)
∂y
∂φ
= 3xz 2 whence φ = xz 3 + f3 (x, y) (2.14)
∂z
Then (2.12), (2.13) and (2.14) each represents φ. These agree if
we choose

f1 (y, z) = 0, f2 (x, z) = xz 3 , f3 (x, y) = x2 y

∴ φ = x2 y + xz 3 + c
R (3,1,4) →
− − → R (3,1,4) (3,1,4)
Work done = (1,−2,1) F .d r = (1,−2,1) dφ = [φ](1,−2,1)

(3,1,4)
(i.e.) Work done = [x2 y+xz 3 ](1,−2,1) = (9+192)−(−2+1) = 202.

→ −

Example 2.27. A vector field is given by F = sin y i + x(1 +


cos y) j . Evaluate the line integral over the circular path given
by x2 + y 2 = a2 , z = 0.

Solution: The parametric equation of the circular path are x =


a cos t, y = a sin t and z = 0 where t varies from 0 to 2π. Since
the particle moves in the xy plane (z = 0), we can take − →r =

− −
→ →
− →
− −

x i + y j , so that d r = dx i + dy j .

105
2 VECTOR CALCULUS

H →− − → H −
→ −
→ −
→ −

C F .d r = C [sin y i + x(1 + cos y) j ].[dx i + dy j ]
H
= C [sin ydx + x(1 + cos y)dy]
H H H
= C [(sin ydx + x cos ydy) + xdy] = C d(x sin y) + C xdy
R 2π R 2π
= 0 d[a cos t sin(a sin t)] + 0 a cos ta cos tdt
R 2π
= [a cos t sin(a sin t)]2π
0 +a
2
0 cos2 tdt
 
a2 R 2π a2 sin 2t 2π
= (1 + cos 2t)dt = t+ = πa2
2 0 2 2 0

− −
→ −
→ −
→ H −

Example 2.28. If F = 2y i − z j + x k , evaluate C F × d− →
r
along the curve x = cos t, y = sin t, z = 2 cos t from t = 0 to
π
t=
2

− −
→ → −→
i j k

− →

Solution: Let F × d r = 2y −z x

dx dy dz


− −
→ −

= (−zdx − xdy) i + (xdx − 2ydz) j + (2ydz + zdx) k

In terms of t,

→ −

F × d−

r = [−2 cos t(−2 sin t)dt − cos t(cos t)dt] i


+[cos t(− sin t) − 2 sin t(−2 sin t)dt] j


+[2 sin t(cos t)dt + 2 cos t(− sin t)dt] k

106
2 VECTOR CALCULUS


− −

= [(4 cos t sin t − cos2 t) i + (4 sin2 t − cos t sin t) j dt]
H →− → R x/2
− 2 −→ 2 −

C F ×d r = 0 [(4 cos t sin t−cos t) i +(4 sin t−cos t sin t) j dt]
     
1 1π → − 1 π 1 − → h πi−
→ 1 − →
= 4. − i + 4. . − j = 2− i + π− j
2 22 2 2 2 4 2


Example 2.29. Find the work done when a force F = (x2 −

− →

y 2 + x) i − (2xy + y) j moves particle in the xy plane from (0, 0)
along the parabola y 2 = x. Is the work done different when the
path is the straight line y = x?


− −
→ −

Solution: As F = (x2 − y 2 + x) i − (2xy + y) j , in the xy plane

− −

z = 0, −

r =x i +y j .

− −
→ → →

∴ d−

r = dx i +dy j and F .d−
r = (x2 −y 2 +x)dx−(2xy +y)dy.

(i) Along the parabola x = y 2 , dx = 2ydy.


R →− − →
∴ Work done = W = C F .d r

R1
= 0 (y 4 − y 2 + y 2 )(2ydy) − (2y 3 + y)dy
 1
y6 y4 y2 1 1 1 2
= 2 −2 − = − − =−
6 4 2 0 3 2 2 3

(ii) Along the straight line y = x, dy = dx


R →− − →
Work done = C F .d r

R1
= 0 (y 2 − y 2 + y)dy − (2y 2 + y)dy

107
2 VECTOR CALCULUS

 1
y2 y3 y2 2
= −2 − =−
2 3 2 0 3

We observe that the work done is the same.


R −→ → −
→ −
→ − →
Example 2.30. Evaluate C F .d− r , given that F = z i +x j +

→ −
→ −
→ −

y k and C being the arc of the curve −→r = cos t i + sin t j + t k
from t = 0 to t = 2π.


− −
→ → d−
− →r −

Solution: Let →

r = cos t i + sin t j + t k , = − sin t i +
→ −
− → dt
cos t j + k .
 
R →− → H → d−
− →r
Now C F .d−
r = C F. dt
dt
R 2π →
− →
− →
− −
→ → −
− →
= 0 [(t i + cos t j + sin t k ).(− sin t i + cos t j + k ]dt
R 2π
= 0 (−t sin t + cos2 t + sin t)dt
R 2π R 2π R 2π
= 0 (−t sin t)dt + 0 cos2 tdt + 0 sin tdt
 
R 2π R 2π 1 + cos 2t R 2π
=− 0 td(− cos t) + 0 dt + 0 sin tdt
2

h i  2π
R 2π 1 sin 2t
=− (−t cos t)2π
0 + 0 cos tdt + t + − (cos t)2π
0
2 4 0

= 2π + π = 3π as sin 2π = 0, cos 2π = 1, sin 0 = 0, cos 0 = 1.




Example 2.31. Find the work done by the force F = (x2 +

− →
− −

y 2 ) i + (x2 + z 2 ) j + y k when it moves a particle along the

108
2 VECTOR CALCULUS

upper half of the circle x2 + y 2 = 1 from the point (−1, 0) to


(1, 0).

Solution: As the circle is in the xy plane z = 0. We use x =


cos t, y = sin t, 0 < t < π.

dx dy −
→ −

= − sin t, = cos t, d−

r =x i +y j
dt dt

→ →
− →
− −
→ −
→ →
F = (x2 + y 2 ) i + (x2 + z 2 ) j + y k and F .d−
r = (x2 + y 2 )dx +
2
x dy
R →− − → Rπ 2
Work done C F .d r = 0 (− sin t + cos t cos t)dt

" #
Rπ ∵ cos 3A = 4 cos3 A − 3 cos A
2
= 0 (− sin t + cos t)dt 3 1
∴ cos3 A = cos A + cos 3A
4 4
 
Rπ 3 1
= 0 − sin t + cos t + cos 3t dt
4 4
 π
3 1
= cos t + sin t + sin 3t = −1 − 1 = −2
4 12 0

∴ Work done = −2.


Example 2.32. If φ = 2xyz 2 and C is the curve x =R t2 , y =
2t, z = t3 from t = 0 to t = 1, evaluate the line integral C φd−

r

Solution: Let φ = 2xyz 2 , x = t2 , y = 2t, z = t3 so that φ =


2xyz 2 = 2t2 .2t.t6 = 4t9
R R 9 )(2tdt)−
→ −
→ −

∴ C φd−

r = C (4t i + (2dt) j + (3t2 dt) k

109
2 VECTOR CALCULUS

R1 →
− →
− →

= 0 (8t10 i + 8t9 j + 12t11 k )dt
 1
t11 −
→ t10 −
→ t12 −
→ 8−→ 4− → − →
= 8 i +8 j + 12 k = i + j + k
11 10 12 0 11 5

− −
→ −

Example 2.33. If F = (2x + y) i + (3y − x) j , evaluate
R −→ − →
C F .d r , where C is the curve in the xy plane consisting of
the straight lines from (0, 0) to (2, 0) and then to (3, 2).

Solution: The path of integration C has been shown in the fi-


gure. It consists of the straight lines OA and AB, we have

B(3, 2)

O(0, 0) A(2, 0) X
Fig.3

R →− − → R →
− −
→ −
→ −

C F .d r = C [(2x + y) i + (3y − x) j ].[dx i + dy j ]
R
= C [(2x + y)dx + (3y − x)dy]

Now along the straight line OA, y = 0, dy = 0 and x varies from

110
2 VECTOR CALCULUS

0 to 2.

The equation of the straight line AB is

2−0
y−0= (x − 2) (i.e.) y = 2x − 4
3−2

Along AB, y = 2x − 4, dy = 2dx and x varies from 2 to 3.


R →− − → R2 R3
C F .d r = 0 [(2x + 0)dx + 0] + 2 [(2x + 2x − 4)dx + (6x − 12 −
x)2dx]
 3
R3 (x − 2)2
= [x2 ]20 + 2 (14x − 28)dx = 4 + 14 = 4 + 7 = 11
2 2


Example 2.34. Find the work done by the force F = (ex z −

− →
− →

2xy) i + (1 − x2 ) j + (ex + z) k when it moves a particle from
(0, 1, −1) to (2, 3, 0) along any path.

Solution: Since the path of integration is not given we may


assume that the work done depends on end points only, for which

− →

curl F = 0, F = ∇φ.


∴ We first check if F is conservative.

− →
− −

i j k


− ∂ ∂ ∂

∴∇× F =
∂x ∂y ∂z
ex z − 2xy 1 − x2 ex + z

   

→ ∂ x ∂ 2 → ∂ x
− ∂ x
= i (e + z) − (1 − x ) − j (e + z) − (e z − 2xy)
∂y ∂z ∂x ∂z

111
2 VECTOR CALCULUS
 

→ ∂ ∂ x
+k (1 − x2 ) − (e z − 2xy)
∂x ∂y

− →
− −

= (0 − 0) i − (ex − ex ) j + (−2x + 2x) k = 0


⇒ F is conservative force.


Let F = ∇φ


− →
− → P−
− → ∂φ
(i.e.) (ex z − 2xy) i + (1 − x2 ) j + (ex + z) k = i
∂x

∂φ ∂φ ∂φ
⇒ = ex z − 2xy; = 1 − x2 ; = ex + z
∂x ∂y ∂z

Integrating, φ = ex z − x2 y + f1 (y, z); φ = y − xy + f2 (x, z); φ =


z2
ex z + + f3 (x, y)
2

z2
We choose φ = ex z − x2 y + y + +c
2
R →− − → (2,3,0)
Now work done W = C F .d r = [φ](0,1,−1)

 (2,3,0)  
z2 1
= ex z − x2 y +y+ = (0 − 12 + 3) − −1 + 1 +
2 (0,1,−1) 2

1 19
= −9 − =−
2 2

→ −
→ − →
Example 2.35. Find the work done by the force F = x i −z j +


2y k in the displacement along the closed path C, consisting of
segments C1 , C2 and C3 , where

112
2 VECTOR CALCULUS

on C1 , 0 ≤ x ≤ 1, y = x, z = 0
on C2 , 0 ≤ z ≤ 1, x = 1, y = 1
on C3 , 1 ≥ x ≥ 0, y = z = x.

H −→ − →
Solution: Total work done = C F .d r

H −
→ →
− −
→ →
− −
→ −

= C (x i − z j + 2y k )(dx i + dy j + dz k )
H
= C (xdx − zdy + 2ydz)

on C1 , 0 ≤ x ≤ 1, y = x, z = 0, dz = 0

R1 1
∴ W1 = 0 xdx =
2

on C2 , 0 ≤ z ≤ 1, x = 1, y = 1, dx = 0, dy = 0
R1
∴ W2 = 2 0 dz = 2

on C3 , 1 ≥ x ≥ 0, y = z = x, dy = dz = dx
R1 R1
∴ W3 = 0 (xdx − xdx + 2xdx) = 2 0 xdx = 1

1 3
Total work done = W1 + W2 + W3 = +2+1=
2 2
RR − → −
→ −

Example 2.36. Evaluate nds, where A = (x + y 2 ) i −
S A .b

− →

2x j + 2yz k and S is the surface of the plane 2x + y + 2z = 6
in the first octant.

Solution: A unit vector normal to the surface S is given by


∇φ
n
b= where φ = 2x + y + 2z − 6
|∇φ|

113
2 VECTOR CALCULUS


− →

2−
→+ j +2k 2−→ 1−→ 2−→
b= √
⇒n = i + j + k
4+1+4 3 3 3
 

→ →
− 2−→ 1− → 2−→ 2
k .b
n= k. i + j + k =
3 3 3 3

RR → − RR −→ dxdy
∴ S A .b
ndS = R A .b
n. −
→ where R is the projection of S
| k .b
n|
on the xy plane. The region R is bounded by x axis, y axis, line
2x + y = 6, z = 0.

O
B Y
A

X
Fig.4

 

− →
− →
− −
→ 2− → 1− → 2−→
n = [(x + y 2 ) i − 2x j + 2yz k ].
Now A .b i + j + k
3 3 3
 
2 2 4 2 4 6 − 2x − y
= (x + y 2 ) − x + yz = y 2 + y
3 3 3 3 3 2

4
= y(3 − x)
3

114
2 VECTOR CALCULUS

RR →− RR −→ dxdy RR 4 3
Hence S A .b
ndS = R A .b
n. −
→ = R 3 y(3 − x). 2 dxdy
| k .b
n|
R 3 R 6−2x
= 0 0 2y(3 − x)dydx
 6−2x
R3 y2
= 0 2(3 − x)
2 0

R3 2
R3
= 0 (3 − x)(6 − 2x) dx = 4 0 (3 − x)3 dx
 3
(3 − x)4
=4 = −(0 − 81) = 81
4(−1) 0
RR − → −
→ −
→ −

Example 2.37. Evaluate nds, where A = z i + x j −
S A .b


3y 2 z k and S is the surface of the cylinder x2 + y 2 = 16 included
in the first octant between z = 0 and z = 5.

Solution: A vector normal to the surface S is given by ∇(x2 +



− −

y 2 ) = 2x i + 2y j .

b = a unit vector normal to surface.


∴n

− −
→ −
→ −

2x i + 2y j x i +y j
=p = p
(2x)2 + (2y)2 x2 + y 2

− −

x i +y j
= as x2 + y 2 = 16
4
RR −→
Let R be the projection of S on yz plane, then S A .b
nds =
RR − → dxdy
R A .b
n. −

| k .b
n|

115
2 VECTOR CALCULUS

A' O' B'

4
O
A B
X Y
Fig.5

The region R is OBB ′ O′ enclosed by y = 0 to y = 4 and z = 0


to z = 5.
 

− → 1 −
− → 1− → 1
Now i .bn= i xi + j = x
4 4 4
 

→ →
− →
− →
− 1 −→ 1− →
n = (z i + x j − 3y 2 z k ).
A .b xi + j
4 4

1 1 1
= xz + xy = x(y + z)
4 4 4
RR →− RR →− dxdy RR 1 dydz
Hence S A .b
n ds = R A .b
n . −
→ = R 4 x(y + z) 1
| k .b
n| x
4
 4
R5R4 R 5 y2
= 0 0 (y + z)dydz = 0 + zy
2 0

116
2 VECTOR CALCULUS

 5
R5 z2
= 0 (8 + 4z)dz = 8z + 4 = 40 + 50 = 90
2 0
RR − → −→ −
→ −
→ −

Example 2.38. Evaluate S F .d s , where F = yz i +zx j +


xy k and S is the part of the sphere x2 + y 2 + z 2 = 1 that lies in
the first octant.

RR →−
Solution: Let I = n.d−
S F .b

s

∇φ
n
b= where φ = x2 + y 2 + z 2 = 1
|∇φ|

− −
→ →

2x i + 2y j + 2z k −
→ −
→ −

⇒n b= p = x i + y j + z k as (x, y, z) lies
4(x2 + y 2 + z 2 )
on S.
RR −
→ →
− −
→ −
→ → −
− → RR
∴I= S (yz i + zx j + xy k ).(x i + y j + z k ) = S 3xyzds

RR dxdy
= R 3xyz

− where R is the projection in the xy - plane
| k .b
n|
bounded by the circle x2 +y 2 = 1 and lying in the first quadrant.

√  2  1−y2
RR dxdy R 1 R 1−y2 R1 x
I= R 3xyz z = 0 0 3xydxdy = 0 3y dy
2 0
 1
3 R1 2 3 y2 y4 3
= 0 y(1 − y )dy = − = .
2 2 2 4 0 8
RRR 2
Example 2.39. Evaluate V φdv, where φ = 45x y and V is
the closed origin bounded by the planes 4x2y + z = 8, x = 0, y =
0, z = 0.

117
2 VECTOR CALCULUS

O
B Y
A

X
Fig.6

RRR R2 R 4−2x R 8−4x−2y


Solution: Let V φdv = x=0 y=0 z=0 45x2 ydzdydx

R2 R 4−2x 2 8−4x−2y
= 45 x=0 y=0 x y[z]0 dydx

R2 R 4−2x
= 45 x=0 y=0 x2 y(8 − 4x − 2y)dydx

 4−2x
R2 y2 y3
= 45 0 x2 (8 − 4x) − 2x2 dx
2 3 0

R 2 x2 3
= 45 0 3 (4 − 2x) dx = 128


→ −
→ −
→ −

Example 2.40. If F = (2x2 − 3z) i + 2xy j − 4x k , then
RRR →

evaluate V ∇ F .dv, where V is bounded by the planes x =
0, y = 0, z = 0 and 2x + 2y + z = 4.


− ∂ ∂ ∂
Solution: Let ∇ F = (2x2 −3z)+ (−2xy)+ (−4x) = 2x
∂x ∂y ∂z

118
2 VECTOR CALCULUS

RRR →
− RRR
Hence V ∇ F .dv = V 2xdxdydz

R 2 R 2−x R 2 R 2−x
= 0 0 2x[z]4−2x−2y
0 dydx = 0 0 2x(4 − 2x − 2y)dydx
R 2 R 2−x
= 0 0 [4x(2 − x) − 4xy]dydx
R2
= 0 [4x(2 − x)y − 2xy 2 ]02−x dx
R2
= 0 [4x(2 − x)2 − 2x(2 − x)2 ]dx
R2 R2
= 0 2x(2 − x)2 dx = 2 0 (4x − 4x2 + x3 )dx
 2  
x3 x4 32 8
=2 2x2 −4 + =2 8− +4 =
3 4 0 4 3

EXERCISE


− →
− 2 −2t)−
→ 2 +3t3 )−
→ R1 − →
1. If
 f ( t = t i +(t j +(3t k , find the 0 f ( t )dt
1−→ 2− → 7− →
Ans: i − j + k
2 3 4

→ = t−
− → 2− → →
− −
→ −

2. If
R 2 r− i +t j +(t−1) k and −

s = 2t2 i +6t k , evaluate
→−→
0 r . s dt. [Ans: 12]


− −
→ →
− R −→ →
3. If F = 3xy i −y 2 j , evaluate C F .d−
r , where
 C is the arc

2 7
of the parabola y = 2x from (0, 0) to (1, 2). Ans: −
6

− →
− −
→ −

4. If F = (4xy − 3x2 z 2 ) i + 2x2 j − 2x3 z k , show that
R −→ − →
C F .d r is independent of the path C.

119
2 VECTOR CALCULUS

5. Define the line integral of a vector point function.

6. State the necessary and sufficient condition for the line


RB−
→ →
integral A F .d−r to be independent of the path of inte-
gration.

7. What is meant by conservative vector field?


R
8. Evaluate C xdy − ydx around the circle x2 + y 2 = 1. [Ans:
2π]

− −

9. Find the circulation of F around the curve C, where F =

→ →
− →

y i + z j + x k and C is the circle x2 + y 2 = 1, z = 0 [Ans:
−π]

− →
− −
→ R−→ →
10. If A = (5xy − 6x2 ) i + (2y − 4x) j , evaluate A .d−
r
where C is the curve y = x3 in the xy- plane from the
point (1, 1) to (2, 8). [Ans: 35]

− →
− −
→ −

11. F = (3x2 + 6y) i − 14yz j + 20xz 3 k , then evaluate
R − → − →
C F .d r from (0, 0, 0) to (1, 1, 1) along the curve x =
t, y = t , z = t3 . [Ans: 5]
2

R −→ → −
→ −
→ −

12. Evaluate C F .d− r where F = (x2 + y 2 ) i − 2xy j where
C is the rectangle in the xy - plane bounded by y = 0, x =
a, y = b and x = 0. [Ans: −2ab2 ]

13. Find the work done in moving a particle once around a


circle C in the xy - plane, if the circle has centre at the


origin and radius 2 and if the force field F is given by

− →
− −
→ −

F = (2x − y + 2z) i + (x + y − z) j + (3x − 2y − 5z) k .

120
2 VECTOR CALCULUS

[Ans: 8π]
R
14. Evaluate the line integrals C [(x2 + xy)dx + (x2 + y 2 )dy]
where C is the square formed by the lines y = ±1 and
x = ±1. [Ans: 0]
R
15. Evaluate C [(y + 3z)dx + (2z + x)dy + (3x + 2y)dz],where
2aθ
C is the arc of helix x = a cos θ, y = a sin theta, z =
π
between the points (a, 0, 0) and (0, a, 0) [Ans: 2a]

− −
→ →
− →
− R −→
16. If F = xy i − z j + x2 k , evaluate C F × d− →
r , where C
2
is the curve x = t , y = 2t,z = t from (0, 0, 0) to (1, 2, 1).

9→− 2−→ 7− →
Ans: − i − j + k
10 3 5

→ −

17. Find the work done by the force F = (y 2 cos x + z 3 ) i +

− −

(2y sin x − 4) j +(3xz 2 + 2)
 k , when it moves a particle
π
from (0, 1, −1) to , −1, 2 along any path. [Ans: 4π + 5]
2
R
18. Show that the line integral C [(2xy + 3)dx + (x2 − 4z)dy −
4ydz], where C is any path joining (0, 0, 0) to (1, −1, 3) does
not depend on the path C and evaluate the line integral.
[Ans: 14]

19. Find the work done in moving a particle in the force field

− →
− →
− −

F = 3x2 i + (2xz − y) j + z k along the curve defined by
x2 = 4y, 3x3 = 8z from x = 0 to x = 2. [Ans: 16]

RR − → 3 −
→ −
→ −

20. Show that S F .b
nds = , where F = 4xz i − y 2 j +

− 2
yz k and S is the surface of the cube bounded by planes

121
2 VECTOR CALCULUS

x = 0, x = 1, y = 0, y = 1, z = 0 and z = 1.
RR − → −
→ −
→ −
→ −

21. Evaluate nds, where F = 4x i − 2y 2 j + z 2 k and
S F .b
S is the surface bounding the region x2 + y 2 = 4, z = 0 and
z = 3. [Ans: 84π]
RR − → −
→ −
→ −
→ −

22. Evaluate nds, where F = xy i − x2 j + (x + z) k
S F .b
and S is partof the plane
 2x + 2y + z = 6 included in the
27
first octant. Ans:
4

→ →
− →
− −
→ RRR − →
23. If F = 2z i − x j + y k , evaluate V F .dv, where V
is the region boundedby the surfaces x =  0, y = 0, x =
2 32 −→ −

2, y = 4, z = x , z = 2 Ans: (3 i + 5 j )
15

− →
− −
→ −
→ RRR
24. If F = (2x2 −3z) i −2xy j −4x k , then evaluate V ∇×


F , where V is the closed region bounded by the planes 
8 −→ − →
x = 0, y = 0, z = 0 and 2x + 2y + z = 4. Ans: ( j − k )
3

2.10 Integral Theorems

2.10.1 Green’s Theorem in the Plane

Statement: If M (x, y) and N (x, y) be continuous functions of


∂M ∂N
x and y having continuous partial derivatives and in
∂y ∂x
a region R of the xy - plane bounded by a closed curve C, then

122
2 VECTOR CALCULUS
 
H RR ∂N ∂M
C (M dx + N dy) = R − dxdy, where C is traver-
∂x ∂y
sed in the counter clockwise direction.

2.10.2 Stoke’s Theorem (Relation between Line and


Surface Integrals)

Statement: If S be an open surface bounded by a closed curve C



− →
− →
− →

and F = F1 i +F2 j +F3 k be any vector point function having
H −→
continuous first order partial derivatives, then C F × d−→r =
RR →

nds, where n
C curl F .b b is a unit normal vector at any point
of S drawn in the sense in which a right bounded screw would
advance when rotated in the sense of description of C.

2.10.3 Gauss Divergence Theorem (Relation between


Surface and Volume Integrals)



Statement: If F is a vector point function having continuous
first order derivatives in the region V bounded by a closed surface
RRR − → − → RR − →
S, then V F .d v = nds, where n
S F .b b is the outwards
drawn unit normal to the surface S.
H
Example 2.41. Verify Green’s theorem in the plane for C (3x2 −
8y 2 )dx + (4y − 6xy)dy, where C is the boundary of the region
defined by x = 0, y = 0, x + y = 1.

Solution: Here M = 3x2 − 8y 2 , N = 4y − 6xy

∂M ∂N
= −16y; = −6y
∂y ∂x

123
2 VECTOR CALCULUS

∂N ∂M
∴ − = 10y
∂x ∂y
y
B(0,1)

x + y =1
x =0

O y=0 A(1,0) x

Fig. 7

 
RR ∂N ∂M R 1 R 1−x R1
∴ R − dxdy = 0 0 10ydydx = 0 5[y 2 ]01−x dx
∂x ∂y
 1
R1 (1 − x)3 5 5
=5 0 (1 − x)2 dx =5 = − (0 − 1) =
−3 0 3 3
Z Z  
∂N ∂M 5
∴ − dxdy = (2.15)
R ∂x ∂y 3

Along OA, y = 0 ∴ dy = 0 and the limits of x are from 0 to 1.


R1
∴ Line integral along OA = 0 3x2 dx = [x3 ]10 = 1

Along AB, y = 1 − x. ∴ dy = −dx and the limits of x are from


1 to 0.

124
2 VECTOR CALCULUS

R0
∴ Line integral along AB = 1 [3x2 − 8(1 − x)2 ]dx + [4(1 − x) −
6x(1 − x)](−dx)
R0
= 1 (3x2 − 8 + 16x − 8x2 − 4 + 4x + 6x − 6x2 )dx
 0
R0 11
= 1 (−12 + 26x − 11x2 )dx = −12x + 13x2 − x3
3 1
 
11 8
= − −12 + 13 − =
3 3
R0
∴ Line integral along BO = 1 4ydy as along BO.

x = 0, dx = 0 and the limits of y are from 1 to 0.


R0
Along BO: 1 4ydy = [2y 2 ]01 = −2

8 5
∴ Line integral along C ((i.e.) along OABO) = 1 + −2=
3 3
I
5
(i.e.) (M dx + N dy) = (2.16)
C 3

The equality of (2.15) and (2.16), verify Green’s theorem in the


plane.
H
Example 2.42. Verify Green’s theorem in the plane for C (3x2 −
8y 2 )dx + (4y − 6xy)dy, where C is the closed curve of the region
bounded by y = x and y = x2 .

Solution: By Green’s theorem in plane, we have

125
2 VECTOR CALCULUS
 
RR ∂N ∂M H
R − dxdy = C (M dx + N dy)
∂x ∂y

Here M = xy + y 2 ; N = x2

Y
(1,1)

y=x
y = x2
R

O X
Fig.8

The curves y = x and y = x2 intersect at (0, 0) and (1, 1). The


positive direction in traversing C is as shown in the figure.
   
RR ∂N ∂M RR ∂ 2 ∂M 2
∴ R − dxdy = R (x ) − (xy + y ) dxdy
∂x ∂y ∂x ∂y
RR RR
= R (2x − x − 2y)dxdy = R (x − 2y)dxdy
R1Rx R1
= 0 y=x2 (x − 2y)dydx = 0 [xy − y 2 ]xx2 dx

R1 2 2 3 4
R1
= 0 (x − x − x + x )dx = 0 (x4 − x3 )dx

126
2 VECTOR CALCULUS

 1
x5 x4 1 1 1
= − = − =
5 4 0 5 4 20

Now let us evaluate the line integral along C. Along y = x2 , dy =


2xdx.

∴ Along y = x2 , the line integral equals

R1 R1 19
0 [[x(x2 ) + x4 ]dx + x2 (2x)dx] = 0 [3x3 + x4 ]dx =
20

Along y = x, dy = dx. Therefore along y = x, the line integral


equals
 0
R0 R0 x3
= 1 [(x(x) + x2 )dx + x2 dx] = 1 3x2 dx = = −1
3 1

19 1
Therefore the required line integral = −1=−
20 20

Hence the theorem is verified.


R
Example 2.43. Verify Green’s theorem in the plane for C (x2 +
2xy)dx + (y 2 + x3 y)dy where C is a square with vertices P (0, 0),
Q(1, 0), R(1, 1) and S(0, 1).

Solution: The Green’s theorem in plane is


Z Z Z  
∂N ∂M
(M dx + N dy) = − dxdy (2.17)
C R ∂x ∂y

∂M ∂N
Here M = x2 + 2xy; N = y 2 + x3 y, = 2x; = 3x2 y
∂y ∂x

127
2 VECTOR CALCULUS
 
RR ∂N ∂M R1R1
∴ R − dxdy = 0 0 (3x2 y − 2x)dxdy
∂x ∂y
"  1  1 #
R1 x3 x2 R1
= 0 3y −2 dy = 0 (y − 1)dy
3 0 2 0

Z Z    1
∂N ∂M y2 1 1
− dxdy = −y = −1= (2.18)
R ∂x ∂y 2 0 2 2

Y
R(1,1)
S(0,1)

P(0, 0) Q(1, 0)

Fig.9

Now along PQ, y = 0, dy = 0 and x varies from 0 to 1.


 1
R 2
R1 y3 1
∴ P Q x dx = 0 x2 dx = =
3 0 3

Along QR, x = 1 ⇒ dx = 0, y = 0, 1.

128
2 VECTOR CALCULUS

 1
R R1 y3 y2 1 1 5
∴ QR (y 2 + y)dy = 0 (y 2 + y)dy = + = + =
3 2 0 3 2 6

Along RS, y = 1 ⇒ dy = 0, x = 1, 0.
R 2
R0
∴ RS (x + x)dx = 1 (x2 + 2x)dx
 0
x3 2x2 1 4
= + =− −1=−
3 2 1 3 3

Along SP, x = 0, dx = 0, y = 1, 0.
 0
R R0 y3 1
SP y 2 dy = 1 y 2 dy = =−
3 1 3

Adding the four line integrals, we have


Z
1 1 1 1 1 1
(M dx + N dy) = + + − − 1 − = − (2.19)
C 3 3 2 3 3 2
From (2.18 and (2.19, Green’s theorem stands verified.
R
Example 2.44. Apply Green’s theorem in the plane to C [(2x2 −
y 2 )dx + (x2 + y 2 )dy] where C is the boundary of the surface en-
closed by the x - axis and the semicircle y = (1 − x2 )1/2 .

∂M ∂N
Solution: Here M = 2x2 − y 2 ; N = x2 + y 2 , = −2y; =
∂y ∂x
2x

∴ By Green’s theorem,
 
R RR ∂N ∂M
C [(2x2 − y 2 )dx + (x2 + y 2 )dy] = R − dxdy
∂x ∂y

129
2 VECTOR CALCULUS

R1 R √1−x2
= x=−1 y=0 (2x + 2y)dydx

 √1−x2
R1 y2
=2 x=−1 xy +
2 y=0
Z  p Z 1
2 2 2 2 1 2
[(2x −y )dx+(x +y )dy] = 2 x 1 − x2 + (1 − x ) dx
C −1 2
(2.20)
To evaluate the integral (I) we note that integrand is an odd
function
R1 √
⇒ x=−1 x 1 − x2 = 0
R1
Given integral I = 2 0 (1 − x2 )dx as the II integrand is even.
 1  
x3 1 4
=2 x− =2 1− = .
3 0 3 3
R 2 − y 2 )dx + (x2 + y 2 )dy]
Hence equation (2.20) becomes C [(2x =
4 4
0+ = .
3 3
R
Example 2.45. Using Green’s theorem evaluate C [x2 ydx +
x2 dy] where C is the boundary described counter clockwise of the
triangle with vertices (0, 0), (1, 0), (1, 1).

Solution: By Green’s theorem,


 
H RR ∂N ∂M
C (M dx + N dy) = R ∂x

∂y
dxdy

R 2 ydx
RR
C [x + x2 dy] = R (2x − x2 )dxdy

130
2 VECTOR CALCULUS

A(1,1)

y=x

O
Fig.10

R1 Rx R1
= 0 (2x − x2 )dx 0 dy = 0 (2x − x2 )[y]dx
 1
R1 x3 x4 2 1 5
= 0 (2x2 − x3 )dx = 2 − = − = .
3 4 0 3 4 12
Example 2.46. Show that the area bounded by the simple closed
1H
curve C is given by (xdy − ydx). Hence find the area of the
2 C
ellipse x = a cos θ, y = b sin θ.

Solution: By Green’s theorem in plane, if HR is a plane region


bounded
 by a simple
 closed curve C, then C (M dx + N dy) =
RR ∂N ∂M
R − dxdy putting M = −y, N = x we get
∂x ∂y
 
H RR ∂ ∂
C (xdy − ydx) = R ∂x
(x) −
∂y
(−y)

131
2 VECTOR CALCULUS

RR
=2 R dxdy = 2A where A is the area bounded by C.

1H
Hence A = (xdy − ydx)
2 C
1H
The area of the ellipse = (xdy − ydx)
2 C
 
1 R 2π dy dx
= a cos θ − b sin θ dθ
2 θ=0 dθ dθ

1 R 2π
= (ab cos2 θ + ab sin2 θ)dθ
2 θ=0
1 R 2π
= ab 0 dθ = πab
2

→ −

Example 2.47. Verify Stoke’s theorem for F = (x2 + y 2 ) i −


2xy j taken round the rectangle bounded by x = ±a, y = 0, y = b


→ −
→ −

i j k


− ∂ ∂ ∂

Solution: We have curl F =
∂x ∂y ∂z
x2 + y 2 −2xy 0


− −

= (2y − 2y) k = −4y k
→ RR
− →
− Rb Ra −
→− →
Also n
b= k ∴ nds = y=0 x=−a (−4y k ) k dxdy
S (curl F ).b

Rb Ra Rb a
= −4 y=0 x=−a ydxdy = −4 0 [xy]−a

Rb
= −4 0 2aydy = −4[ay 2 ]b0 = −4ab2

132
2 VECTOR CALCULUS

C y=b B

x=-a x =a

D y=0 A

Fig.11

H →− − → H 2 2 −
→ −
→ −
→ −

Also C F .d r = C [(x + y ) i − 2xy j ].[dx i + dy j ]
H 2
= C [(x + y 2 )dx − 2xydy]
R 2
R
= DM [(x + y 2 )dx − 2xydy] + AB [(x
2 + y 2 )dx − 2xydy]
R 2
R
+ BE [(x + y 2 )dx − 2xydy] + ED [(x
2 + y 2 )dx − 2xydy]

Along DA, y = 0 and dy = 0, Along AB, x = a and dx = 0

Along BE, y = b and dy = 0, Along ED, x = −a and dx = 0


H →− − → R x=a 2 R y=b
∴ C F .d r = x=−a x dx + y=0 (−2ay)dy

R x=−a R y=0
+ x=a (x2 + b2 )dx + y=b 2aydy

133
2 VECTOR CALCULUS

R x=a 2 dx
R x=a 2
Rb
= x=−a x − x=−a (x + b2 )dx − 4a y=0 ydy

 b
Ra Rb y2
=− b2 dx − 4a 0 ydy = −2ab2 − 4a = −4ab2
−a 2 0

H →− − → R R (curl−

Thus C F .d r = S F ).b
nds

→ −

Example 2.48. Verify Stoke’s theorem for F = (2x − y) i −

− →

yz 2 j −y 2 z k where S is the upper half of the sphere x2 +y 2 +z 2 =
1 and C is its boundary in the xy - plane.

Solution: The boundary C of S is a circle in the xy - plane


of radius unity and centre at origin. Suppose x = cos t, y =
sin t, z = 0, 0 ≤ t ≤ 2φ are parametric equations of C.
H →− − → H →
− 2−
→ 2 −
→ −
→ −

Then C F .d r = C [(2x − y) i − yz j − y z k ].[dx i + dy j +


dz k ]
H
= C [(2x − y)dx − yz 2 dy − y 2 zdz]
H
= C (2x − y)dx, since z = 0 and dz = 0

R 2π dx
= 0 (2 cos t − sin t) dt
dt
R 2π
=− 0 (2 cos t − sin t) sin tdt
 
R 2π 1
=− 0 sin 2t − (1 − cos 2t) dt
2

134
2 VECTOR CALCULUS

 
cos 2t 1 1 sin 2t 2π
=− − − t+
2 2 2 2 0
I   
→ −
− → 1 1 1 1
F .d r = − − + − (2π − 0) + (0 − 0) (2.21)
C 2 2 2 4
− →

→ i


j k


− ∂ ∂ ∂
Also ∇ × F =
∂x ∂y ∂z
2x − y −yz 2 −y 2 z


− →
− −
→ − →
= (−2yz + 2yz) i − (0 − 0) j + (0 + 1) k = k


Also n
b = k , ds = dxdy
RR →
− RR−
→−→
∴ S (∇ × F ).b
nds = k k dxdy

= Area of circle
Z Z


nds = π(l)2 = π
(∇ × F ).b (2.22)
S

Hence from (2.21) and (2.22), the theorem is verified.



→ −

Example 2.49. Verify Stoke’s theorem for F = (y − z + 2) i −

− →

(yz + 4) j − (xz) k over the surface of a cube x = 0, y = 0, z =
0, x = 2, y = 2, z = 2 above the XOY plane.

Solution: Consider the surface of the cube as shown R −in the fi-
→ →
gure. Boundary path OABCO shown by arrows. C F .d− r =
R →
− →
− −
→ −
→ −
→ −

C [(y − z + 2) i − (yz + 4) j − (xz) k ].[dx i + dy j + dz k ]

135
2 VECTOR CALCULUS

F(0, 0, 2)
G
E
D
(x, y, z)

O
C(0, 2, 0)

A(2, 0, 0) B(2, 2, 0)

Fig.12

R
= C (y− z + 2)dx + (yz + 4)dy − xzdz
Z Z Z Z Z
→ −
− → −
− → −
− −
→ − −
→ −
F .d→
r = F .d→r + F .d→
r + F .d→
r + F .d→
r
C OA AB BC CO
(2.23)
Along OA, y = 0, dy = 0, z = 0, dz = 0
R − −
→ R2
OA F .d→
r = 0 2dx = [2x]20 = 4

Along AB, x = 2, dx = 0, z = 0, dz = 0
R − −
→ R2
AB F .d→
r = 0 4dy = 4[y]20 = 8

Along BC, y = 2, dy = 0, z = 0, dz = 0
R − −
→ R0
BC F .d→
r = 2 (2 − 0 + 2)dx = [4x]02 = −8

Along CO, x = 0, dx = 0, z = 0, dz = 0

136
2 VECTOR CALCULUS

R − −
→ R0
CO F .d→
r = 4 2 dy = 4[y]02 = −8

On putting the values of these integrals in (2.23), we get


R →− − →
C F .d r = 4 + 8 − 8 − 8 = −4

To obtain the surface integral



− →
− −

i j k


− ∂ ∂ ∂

∇× F =
∂x ∂y ∂z
2y − z + 2 yz + 4 −xz


− →
− −
→ −
→ → −
− →
= (0 − y) i − (−z + 1) j + (0 − 1) k = −y i + (z − 1) j − k

Here we have to integrate over five surfaces, ABDE, OCGF,


BCGD, OAEF, DEFG.


Over the surface ADBE (x=2), n
b = i , ds = dydz
RR →
− RR −
→ → −
− → − →
∴ S (∇ × F ).b
nds = [−y i + (z − 1) j − k ]. i dydz
RR R2 R2
= (−y)dydz = − 0 ydy 0 dz
 2
y2
=− [z]20 = −4
2 0



Over the surface OCGF (x=0), n
b = − i , ds = dydz
RR →
− RR −
→ → −
− → −

∴ S (∇ × F ).b
nds = [−y i + (z − 1) j − k ].(− i )dydz

137
2 VECTOR CALCULUS

 2
RR R2 R2 y2
= ydydz = 0 ydy 0 dz = 2 =4
2 0



Over the surface BCGD (y=2), n
b = j , ds = dxdz
RR →
− RR −
→ → −
− → − →
∴ S (∇ × F ).b
nds = [−y i + (z − 1) j − k ]. j dxdz
RR R2 R2
= (z − 1)dxdz = 0 dx 0 (z − 1)dz
 2
z2
= [x]20 −z =0
2 0



Over the surface OAEF (y=0), n
b = − j , ds = dxdz
RR →
− RR −
→ → −
− → −

∴ S (∇ × F ).b
nds = [−y i + (z − 1) j − k ].(− j )dxdz
RR R2 R2
=− (z − 1)dxdz = − 0 dx 0 (z − 1)dz
 2
z2
= −[x]20 −z =0
2 0



Over the surface DEFG (z=2), n
b = k , ds = dxdy
RR →
− RR −
→ → −
− → − →
∴ S (∇ × F ).b
nds = [−y i + (z − 1) j − k ]. k dxdz
RR R2 R2
=− dxdy = − 0 dx 0 dy = −[x]20 [y]20 = −4

Total surface integral = −4 + 4 + 0 + 0 − 4 = −4


RR →
− R −→ →
Thus S nds = C F .d−
curl F .b r = −4 which verifies Stoke’s

138
2 VECTOR CALCULUS

theorem.
H
Example 2.50. Evaluate C (xydx + xy 2 dy) by Stoke’s theo-
rem where C is the square in the xy - plane with vertices (1, 0),
(−1, 0), (0, 1), (0, −1).


− −
→ −

Solution: Here F = xy i + xy 2 j

→ −
→ −

i j k


− ∂ ∂ ∂
= (y 2 − x)−

curl F = k
∂x ∂y ∂z
xy xy 2 0



Also n
b= k

− →−
− →
n = (y 2 − x) k . k = y 2 − x
curl F .b
RR 2
R1 R1 2
The given integral = S (y − x)ds = y=−1 x=−1 (y − x)dxdy

 1
R1 x2 R1
= y=−1 y2x − dy = −1 2y
2 dy
2 x=−1

 1
y3 4
=2 =
3 −1 3
RR −

Example 2.51. Use Stoke’s theorem to evaluate (∇× F ).b
nds,

− −
→ →
− −

where F = y i + (x − 2xz) j − xy k and S is the surface of the
sphere x2 + y 2 + z 2 = a2 above the xy plane.

Solution: The boundary C of the surface S is circle x2 + y 2 =


a2 , z = 0. Suppose x = a cos t, y = a sin t and z = 0, 0 ≤ t ≤ 2π
are parametric equations of C. By Stoke’s theorem, we have

139
2 VECTOR CALCULUS

RR →
− R −→ →
nds = C F .d−
(∇ × F ).b r
R −
→ →
− −
→ −
→ −
→ −

= C [y i + (x − 2xz) j − xy k ].[dx i + dy j + dz k ]
R
= C [ydx + (x − 2xz)dy − xydz]
R
= C (ydx + xdy) (∵ on C, z = 0 and dz = 0)
 
R 2π dx dy
= 0 y +x dt
dt dt
R 2π
= 0 [a sin t(−a sin t) + a cos t(a cos t)]dt
R 2π R 2π
= a2 0 (cos2 t − sin2 t)dt = a2 0 cos 2tdt
 2π
sin 2t
= a2 =0
2 0
R − → →
Example 2.52. Evaluate C F .d− r by Stoke’s theorem where

− 2 →
− 2 →
− →

F = y i + x j − (x + z) k and C is the boundary of the
triangle with vertices at (0, 0, 0), (1, 0, 0), (1, 1, 0).


→ −
→ −

i j k



Solution: We have curl F =
∂ ∂ ∂ = 0−

i +
∂x ∂y ∂z

y2 x2 −(x + z)

− →

j + 2(x − y) k

Also we note that z - coordinate of each vertex of the triangle is




zero. Therefore the triangle lies in the xy - plane. So n
b= k.

140
2 VECTOR CALCULUS


− −
→ → −
− →
n = [ j + 2(x − y) k ]. k = 2(x − y) The equation of
∴ curl F .b

Y
B(1,1)

O A(1, 0) X
Fig.13

the line OB is y = x. By Stoke’s theorem


H →− − → R R (curl−
→ R1 Rx
C F .d r = S n)ds = x=0 y=0 2(x − y)dxdy
F .b
 x  
R1 y2 R1 x2
=2 x=0 xy − dx = 2 0 x2 − dx
2 y=0 2

R 1 x2 1
=2 0 2 dx = 3 .

Note: Cartesian Equivalent of Divergence Theorem


 
RRR ∂F1 ∂F2 ∂F3 RR
V + + dxdydz = S (F1 dydz + F2 dzdx +
∂x ∂y ∂z
F3 dxdy)

141
2 VECTOR CALCULUS

Example
RR 2.53. By transforming a triple integral evaluate I =
(x 3 dydz + x2 ydzdx + x2 zdxdy) where S is the closed surface
S
bounded by the planes z = 0, z = b and the cylinder x2 + y 2 = a2 .

Solution: By Divergence theorem, the required surface integral


I is equal to the volume integral
 
RRR ∂ 3 ∂ 2 ∂ 2
V (x ) + (x y) + (x z) dv
∂x ∂y ∂z

Rb Ra R √a2 −y2
= z=0 y=−a
√ (3x2 + x2 + x2 )dxdydz
x=− a2 −y 2


 3  a2 −y2
R b R a R √a2 −y2 R R
b a x
=4×5 0 0 0 x2 dxdydz = 20 0 0 dydz
3 0

20 R b R a 2 20 R a  2 b
= 0 0 (a − y 2 )3/2 dydz == 0 (a − y 2 )3/2 z 0 dy
3 3
20 R a  2 
= 0 (a − y 2 )3/2 b dy
3

Put y = a sin t so that dy = a cos tdt

20 R π/2 3
= b a cos3 t(a cos t)dt
3 0
20 4 R π/2 20 3 π 5
= a b 0 cos4 tdt = a4 b = πa4 b
3 3 4.2 2 4
Example
RR 3 2.54. Apply Gauss divergence theorem to evaluate
[(x −yz)dydz −2x2 ydzdx+zdxdy] over the surface of a cube
bounded by the coordinate planes and the plane x = y = z = a.

142
2 VECTOR CALCULUS

Solution: By divergence theorem, we have


 
RRR ∂F1 ∂F2 ∂F3 RR
V + + dxdydz = S (F1 dydz + F2 dzdx +
∂x ∂y ∂z
F3 dxdy)

Here F1 = x3 − yz, F2 = −2x2 y, F3 = z

∂F1 ∂F2 ∂F3


∴ + + = 3x2 − 2x2 + 1 = x2 + 1
∂x ∂y ∂z

The given surface integral is equal to the volume integral


 a
Ra Ra Ra Ra Ra x3
x=0 y=0 z=0 (x2 + 1)dxdydz = z=0 y=0 +x dydz
3 x=0
   3 
Ra Ra a3 2 a
= z=0 y=0 + a dydz = a +a
3 3
Example
RR 2.55. Apply Gauss divergence theorem to evaluate
[(x + z)dydz + (y + z)dzdx + (x + y)dxdy] where S is the
surface of the sphere x2 + y 2 + z 2 = 4.

Solution: By divergence theorem, the given surface integral is


equal to the volume integral.
 
RRR ∂ ∂ ∂ RRR
V (x + z) + (y + z) + (x + y) dv = v 2dv
∂x ∂y ∂z
RRR 2
= 2 v dv = 2v, where V is the volume of the sphere x +
y2 + z2 = 4
 
4 3 64
= 2 π(2) = π
3 3

143
2 VECTOR CALCULUS


− →
− −
→ −

Example 2.56. If F = x i − y j + (z 2 − 1) k , find the value of
RR − →
nds, where S is the closed surface bounded by the planes
S F .b
z = 0, z = 1 and the cylinder x2 + y 2 = 4.

Solution: By divergence theorem, we have


RR →− RRR −

S F .b
nds = V div F dv


→ ∂ ∂ ∂ 2
Here div F = (x) + (−y) + (z − 1) = 1 − 1 + 2z = 2z
∂x ∂y ∂z

RRR −
→ R1 R2 R x=√4−y2
∴ V div F dv = z=0 y=−2 √ 2zdxdydz
2
x=− 4−y

R1 R2 √ R1 R2 p
x= 4−y 2
= z=0 y=−2 [2zx] √ dydz = z=0 y=−2 4z 4 − y 2 dydz
− 2
4−y

 
R2 z2 p R2 p
= y=−2 4 4 − y 2 dy = 4 0 [ 4 − y 2 ]dy
2
hy p y i2
=4 ( 4 − y 2 ) + 2 sin−1 = 4[2 sin−1 (1)] = 4π
2 2 0
RR 2 2 2
Example 2.57. Evaluate S (ax +by +cz )dS over the sphere
2 2 2
x + y + z = 1 using the divergence theorem.

RR 2 +by 2 +cz 2 )dS.


Solution: Let I = S (ax We put I in the from
RR −→
S F .b
ndS

The normal vector to φ(x, y, z) = x2 + y 2 + z 2 − 1 = 0 is ∇φ =



− −
→ →

2x i + 2y j + 2z k .

144
2 VECTOR CALCULUS


− −
→ →

∇φ 2x i + 2y j + 2z k −
→ −
→ −

n
b= = p = x i + y j + z k as x2 + y 2 +
|∇φ| 2 2 2
4(x + y + z )
z 2 = 1 on S.

− →

n = ax2 +by 2 +cz 2 . Obviously
We have to choose F such that F .b

− →
− −
→ →

F = ax i + by j + cz k
RR →− RRR −

S F .b
nds = V div F dv

RRR −

= V (a + b + c)dV as div F = a + b + c
RRR
= (a + b + c) V dV = (a + b + c)V

4
= (a + b + c) π, (the volume V enclosed by the sphere S of unit
3
4
radius is π)
3

→ −

Example 2.58. Verify divergence theorem for F = (x2 −yz) i +

− →

(y 2 −zx) j +(z 2 −xy) k taken over the rectangular parallelopiped
0 ≤ x ≤ a, 0 ≤ y ≤ b, 0 ≤ z ≤ c.

RR −→ RRR −

Solution: By divergence theorem S F .b
nds = V div F dv


→ ∂ 2 ∂ 2 ∂ 2
Here div F = (x − yz) + (y − zx) + (z − xy) =
∂x ∂y ∂z
2(x + y + z)
RRR −
→ RcRbRa
∴ V div F dv = 0 0 0 2(x + y + z)dxdydz
 a  
R c R b x2 R c R b a2
=2 0 0 + yx + zx dydz = 2 0 0 + ya + za dydz
2 0 2

145
2 VECTOR CALCULUS

 b  
R c a2 y2 R c a2 b2
=2 0 y + a + azy dz = 2 0 b + a + abz dz
2 2 0 2 2
 a
a2 b2 z2
=2 bz + a z + ab = a2 bc + ab2 c + abc2
2 2 2 0
Z Z Z


∴ div F dv = abc(a + b + c) (2.24)
V

Z
C A'
B'
P

O
B Y

X A C'

Fig.14

To evaluate the surface integral on S1 : Face OACB, z = 0, n


b=
→ −
− → →
− →
− −
→ −

n = (x2 i + y 2 j + −xy k ).(− k ) = xy
− k , F .b

RR −
→ RbRa a 2 b2
S1 F .b
n ds = 0 0 xydxdy =
4
→ −
− →
S2 : Face CBPA, z = c, n n = c2 − xy
b = k , F .b

146
2 VECTOR CALCULUS

RR −
→ RbRa 2 2 2
F .b
n ds = (c − xy)dxdy = abc 2− a b
S2 0 0 4
→ −
− →
S3 : Face OBA’C, x = 0, n
b = i , F .b
n = yz

RR −
→ RcRb b2 c 2
S3 F .b
n ds = 0 0 yzdydz =
4
→ −
− →
S4 : Face AC’PB’, x = a, n n = a2 − yz
b = i , F .b

RR −
→ RcRb b2 c 2
S4 nds = 0 0 (a2 − yz)dydz = a2 bc −
F .b
4
→ →
− −
S5 : Face OCB’A, y = 0, n
b = − j , F .b
n = zx

RR −
→ RaRc a2 c 2
S5 F .b
n ds = 0 0 zxdxdz =
4
→ −
− →
S6 : Face BA’PC’, y = b, n n = b2 − zx
b = j , F .b

RR −
→ RaRc 2 2 2
F .b
n ds = (b − zx)dzdx = ab2c − a c
S6 0 0 4

Adding the six surface integrals, we get

RR → − a 2 b2 a 2 b2 b2 c 2 b2 c2 a2 c2
S F .b
nds = + abc2 − + + a2 bc − + +
4 4 4 4 4
2
a c 2
ab2 c −
4 Z Z


F .b
nds = abc(a + b + c) (2.25)
S
The equality of (2.24) and (2.25), verify divergence theorem.


Example 2.59. Verify divergence theorem for the function F =

147
2 VECTOR CALCULUS


− →
− →

4xz i − y 2 j + yz k taken over the cube bounded by the planes
x = 0, x = 1, y = 0, y = 1, z = 0, z = 1.

RR −→ RRR −

Solution: By divergence theorem, S F .b
nds = V div F dv
 

→ →∂
− →∂
− →∂
− −
→ −
→ −

div F = i + j + k .(4xz i − y 2 j + yz k )
∂x ∂y ∂z

∂ ∂ ∂
= (4xz) + (−y 2 ) + (yz) = 4z − 2y + y = 4z − y
∂x ∂y ∂z
RRR −
→ R1R1R1
∴ V div F dv = 0 0 0 (4z − y)dzdydx
R1R1 R1R1
= 0 0 [2z 2 − yz]10 dydx = 0 0 (2 − y)dydx
Z Z Z Z 1 1 Z 1

→ y2 3 3
∴ div F dv = 2y − dx = dx = (2.26)
V 0 2 0 0 2 2

RR − →
To evaluate nds Here S is the surface of the cube bounded
S F .b
by the plane surfaces.


Over the face OABC, z = 0, dz = 0, n
b = − k , ds = dxdy
RR →− R1R1 → −
− →
nds = 0 0 (−y 2 j .(− k )dxdy = 0
S F .b



Over the face BCDE, y = 1, dy = 0, n
b = j , ds = dxdz
RR →− R1R1 → −
− → −
→ − →
nds = 0 0 (4xz i − j + z k ). j dxdz
S F .b

R1R1
= (−1) 0 0 dxdz = −(x)10 (z)10 = −1

148
2 VECTOR CALCULUS

Z
G(0, 0,1)
D

F E
(x, y, z)

O Y
C(0,1, 0)
X A(1, 0, 0) B

Fig.15



Over the face DEFG, z = 1, dz = 0, n
b = k , ds = dxdy
RR →− R1R1 →
− →
− −
→ − →
nds = 0 0 (4x i − y 2 j + y k ). k dxdz
S F .b

 1
R1 R1 y2 1
= 0 dx 0 ydy = (x)10 =
2 0 2



Over the face AOGF, y = 0, dy = 0, n
b = − j , ds = dxdz
RR →− R1R1 →
− →

nds = 0 0 (4xz i ).(− j )dxdz = 0
S F .b



Over the face OCDG, x = 0, dx = 0, n
b = − i , ds = dydz
RR →− R1R1 →
− −
→ −

nds = 0 0 (−y 2 j + yz k ).(− i )dydz = 0
S F .b

149
2 VECTOR CALCULUS



Over the surface ABEF, x = 1, dx = 0, n
b = i , ds = dydz
RR →− R1R1 − → →
− −
→ − →
nds = 0 0 (4z i − y 2 j + yz k ). i dydz
S F .b

R1R1
= 0 0 4zdydz = (y)10 (2z 2 )10 = 2

Adding for the whole surface S, we get


Z Z

− 1 3
F .b
nds = 0 − 1 + + 0 + 0 + 2 = (2.27)
S 2 2

From the equations (2.26) and (2.27), the divergence theorem


stand verified.

EXERCISE
R
1. Evaluate (xdy − ydx), where C is the circle x2 + y 2 = a2
[Ans: 2πa2 ]
RR
2. Evaluate S (xdydz + ydzdx + zdxdy) over the surface of
the sphere x2 + y 2 + z 2 = a2 . [Ans: 4πa3 ]

3. State Gauss divergence theorem.


RR −

4. For any closed surface S, prove that S curl F .b
n.dS = 0
RR →
− n.dS, where S is any closed surface. [Ans:
5. Evaluate S r .b
3V]

6. State Green’s theorem in plane.




7. If F = ∇φ, ∇2 φ = −4πρ, show that

150
2 VECTOR CALCULUS

RR →− RRR
S F .b
n.dS = −4π V ρdV
R
8. Use Green’s theorem to evaluate C [(x2 + xy)dx + (x2 +
y 2 )dy], where C is the square formed by the lines y =
±1, x = ±1. [Ans: 0]
RR → − →
− −
→ −

9. Find n.dS, where F = (2x + 3z) i − (xz + y) j +
S F .b


(y 2 + 2z) k and S is the surface of the sphere having centre
at (3, −1, 2) and radius 3. [Ans: 108π]

− → −
− → −

10. The vector field F = 2x2 i +z j +yz k is defined over the
volume of the cuboid given by 0 ≤ x ≤ a, 0 ≤ y ≤ b, 0 ≤
z ≤ c enclosing the surface
 S, evaluate
 the surface integral
RR − → b
n.dS Ans:abc a +
S F .b 2
R
11. Using Green’s theorem, evaluate C [x2 ydx+x2 dy] where C
is the boundary described counterclockwise  of the triangle
5
with vertices (0, 0), (1, 0), (1, 1). Ans:
12
R →− − →
12. Prove that C r .d r = 0
H
13. Evaluate by Stoke’s theorem C (ex dx + 2ydy − dz), where
C is the curve x2 + y 2 = 4, z = 2 [Ans: 0]

R R R dV RR −
→r .b
n
14. Prove that V r2 = S r 2 dS.

RR →− − →
15. Find S r .d S , where S is the surface of the tetrahedron
whose vertices are (0, 0, 0), (1, 0, 0), (0, 1, 0), (0, 0, 1).

151
2 VECTOR CALCULUS

[Ans: 3]
H
16. Verify Green’s theorem for C (e
−x sin ydx
+ e−x cos
 ydy),
π
where C is the rectangle with vertices (0, 0), (π, 0), π, ,
 π 2
0, . [Ans: Common Value = 2(e−x − 1)]
2
R
17. Verify Green’s theorem for the integral [(x2 + y)dx −
xy 2 dy] taken around the boundary of the square whose
vertices (0, 0), (1, 0), (1, 1) and (0, 1)
 
4
Ans: Common Value = −
3
R
18. Evaluate C [(2xy − x2 )dx + (x + y 2 )dy], where C is the
closed curve 2 2
  of the region bounded y = x and y = x.
1
Ans:
30

19. Find the area of the four cusped hyper cycloid x2/3 +y 2/3 =
a2/3 using Green’s theorem.
 
3πa2
Ans: sq.units
8
R
20. Verify Green’s theorem in the plane for C [(3x2 − 8y 2 )dx +
(4y−6xy)dy], where C is the boundary of the region
 defined
√ 3
on y = x, y = x2 Ans: Common Value =
2


21. Verify Stoke’s theorem for a vector field defined by F =

− −

(x2 − y 2 ) i + 2xy j in the rectangular region in the XOY

152
2 VECTOR CALCULUS

plane bounded by the lines x = 0, x = a, y = 0 and y = b.


[Ans: Common Value = 2ab2 ]

→ −
→ −

22. Verify Stoke’s theorem for F = −y 3 i + x3 j , where S is
the circular disc x2 + y 2 ≤ 1, z = 0
 

Ans: Common Value =
2

23. Apply Stoke’s theorem to evaluate ]intC [(x + y)dx + (2x −


z)dy + (y + z)dz], where C is the boundary of the triangle
with vertices (2, 0, 0), (0, 3, 0) and (0, 0, 6). [Ans: 21]
R → − − → → − −
→ −
→ −

24. Evaluate C F .d r , F =  y i + xZ 3 j − zy 3 k , C is the
19π
circle x2 + y 2 = 4, z = 1.5 Ans :
2
RR 2 x2
25. Evaluate S (a + b2 y 2 + c2 z 2 )−1/2 dS, where
 S is the
2 2 2 4π
surface of the ellipsoid ax + by + cz = 1. Ans : √
abc

→ −
→ −
→ −

26. Verify divergence theorem for F = 4x i − 2y 2 j + z 2 k
taken over the region bounded by x2 + y 2 = 4, z = 0 and
z = 3 [Ans: Common Value = 84π]

27. By converting
R R the surface integral into a volume integral
evaluate (x 3 dydz + y 3 dzdx + z 3 dxdy), where S is the
S  
12π
surface of the sphere x2 + y 2 + z 2 = 1.
5
RR →−
28. Evaluate ndS over the region entire surface of the
S F .b

153
2 VECTOR CALCULUS

region above xy - plane bounded by the curve z 2 = x2 + y 2



− −
→ −
→ −

and the plane z = 4, if F = 4xz i + xyz 2 j + 3z k .

[Ans: 320π]

154
3 LAPLACE TRANSFORM

3.1 Introduction

The Laplace transform plays a vital role in Mathematics required


of Mathematicians, Physicists, Engineers and Scientists. This is
due to the fact that Laplace transform methods provide easy
and effective techniques for the solution of many problems ari-
sing in various fields of Science and Engineering. This chapter
is devoted to the study of Laplace transforms of some elementa-
ry functions, some basic problems, convolution theorem, Inverse
Laplace transforms and finding solutions to linear differential
equations using Laplace transforms.

Definition: Let f (t) be a function of t for t > 0. Then the La-


place transform of f (t), denoted as L[f (t)], is defined by L[f (t)] =
F (s). The Laplace transform of f (t) is also denoted as f (s).

Laplace transform of some elementary functions:

155
3 LAPLACE TRANSFORM

S.No. f(t) L[f(t)]


1
1 1 (s > 0)
s
1
2 t (s > 0)
s2
n!
3 tn (s > 0)
sn+1
1
4 eat (s > a)
s−a
a
5 sin at (s > 0)
s2 + a 2
s
6 cos at (s > 0)
s2 + a 2
a
7 sinh at (s > |a|)
s2 − a 2
s
8 cosh at (s > |a|)
s2 − a 2
1
9 e−at (s > −a)
s+a

Remark 1: It may be noted that not all f (t) are Laplace trans-
formeable.

Remark 2: The sufficient conditions for the existence of Laplace


transform of f (t) are

(i) it is piecewise continuous in every finite interval;

(ii) it is of exponential order ρ, for all s > ρ.

Remark 3: However, the above conditions are not necessary.

1
Illustration: Prove that L[1] = , (s > 0).
s

Proof:

156
3 LAPLACE TRANSFORM

R∞
L[1] = 0 e−st dt (∵ f (t) = 1))
 ∞
e−st 1 1
= = − (0 − 1) =
−s 0 s s

Note: Similarly using the definition we can establish the results


of other functions.

3.2 Some basic operational properties of


Laplace transforms

1. Linearity Property

L[af (t) + bg(t)] = aL[f (t)] + bL[g(t)] where a and b are any two
constants.

Proof:
R∞
L[af (t) + bg(t)] = 0 e−st [af (t) + bg(t)]dt, by definition.
R∞ R∞
=a 0 e−st f (t)dt + b 0 e−st g(t)dt

= aL[f (t)] + bL[g(t)]

This result can be extended to a finite number of functions.

Example: Find: L[2e2t + 3e−3t ]

2 3
L[2e2t + 3e−3t ] = 2L[e2t ] + 3L[e−3t ] = +
s−2 s+3

157
3 LAPLACE TRANSFORM

2. First Shifting Property

If L[f (t)] = F (s), then L[eat f (t)] = F (s − a)


R∞
Proof: We know that L[f (t)] = 0 e−st f (t)dt = F (s)
R∞
∴ L[eat f (t)] = 0 e−st [eat f (t)]dt
R∞
= 0 e−(s−a)t f (t)dt, (s − a) > 0

= F (s − a)

Similarly, L[e−at f (t)] = F (s + a)

Example: Find: L[e−t sin 3t]

3
We know that L[sin 3t] =
s2 + 9

3 3
Then L[e−t sin 3t] = 2
= 2
(s + 1) + 9 s + 2s + 10

3. Second Shifting Property


(
f(t-a), t>a
If L[f (t)] = F (s) and g(t) = then
0, t<a

L[g(t)] = e−as F (s)


R∞
Proof: By definition, L[g(t)] = 0 e−st g(t)dt
Ra R∞ R∞
= 0 e−st g(t)dt + a e−st g(t)dt = 0 + a e−st f (t − a)dt

158
3 LAPLACE TRANSFORM

Put t − a = u, then t = a ⇒ u = 0

dt = du and t = ∞ ⇒ u = ∞
R∞ R∞
L[g(t)] = 0 e−s(u+a) f (u)du = e−sa 0 e−su f (u)du

= e−as L[f (u)] = e−as F (s) (as ’u’ is a dummy variable, L[f(u)=L[f(t)])
R∞
Also L[f (t − a).H(t − a)] = 0 L[f (t − a).H(t − a)]dt

Put t − a = u ⇒ dt = du

t = 0 ⇒ u = −a; t = ∞ ⇒ u = ∞
R∞ −s(u+a) f (u)H(u)du
L[f (t − a).H(t − a)] = −a e

R0 −s(u+a) f (u)H(u)du
R∞
= −a e + 0 e−s(u+a) f (u)H(u)du
R∞ R∞
=0+ 0 e−s(u+a) f (u)du = e−as 0 e−su f (u)du

= e−as L[f (u)] = e−as F (s) (as ’u’ is a dummy variable, L[f(u)=L[f(t)])

It can also be expressed


( as L[f (t − a).H(t − a)] = e−as F (s),
1, if t ≥ a
where H(t − a) = is called the unit step function
0, if t < a
or Heaviside’s unit step function.

3! 6
Example: we have that L[t3 ] = 4
= 4
s s

159
3 LAPLACE TRANSFORM
(
(t − 2)3 , t > 2 6e−2s
Then, L[g(t)], where g(t) = is given by
0, t < 2 s4

4. Change of Scale of Property

1 s
If L[f (t)] = F (s) then If L[f (at)] = F
a a
R∞
Proof: We know that L[f (t)] = 0 e−st f (t)dt = F (s)
R∞
L[f (at)] = 0 e−st f (at)dt

Put at = u ⇒ adt = du when t = 0 ⇒ u = 0; when t = ∞ ⇒


t=∞
R∞ du 1 R ∞ −(s−a)u
∴ L[f (at)] = 0 e−s(u+a) f (u) = e f (u)du
a a 0
1 s
= F
a a

Example: Find L[cos 2t]

s
As L[cos t] =
s2 +1

1 s/2 s
We have L[cos 2t] = . 2
= 2
2 (s/2) + 1 s +4

160
3 LAPLACE TRANSFORM

3.2.1 Laplace transform of Derivatives

I. If L[f (t)] = F (s) then L[f ′ (t)] = sF (s) − f (0) provided f (t)
satisfies the sufficient conditions.
R∞
Proof: Let L[f ′ (t)] = 0 e−st f ′ (t)dt

R∞ df R∞
= 0 e−st dt = 0 e−st df (t)
dt
R∞
= [e−s f (t)]∞
0 − 0 f (t)e
−st (−s)dt

= −f (0) + sL[f (t)] = sF (s) − f (0)

2
Example: For f (t) = sin 2t, we have L[f (t)] = .
s2 +4
 
2 2s
Then we get L[f ′ (t)] = L[2 cos 2t] = s 2
−0=
s +4 s2+4

II. If L[f (t)] = F (s) then L[f ′′ (t)] = s2 F (s) − sf (0) − f ′ (0)

Example: L[f (t)] = cos 3t, find L[f ′′ (t)]

s s
L[f (t)] = (i.e.) F (s) = 2
s2 +9 s +9

We know that L[f ′′ (t)] = s2 F (s) − sf (0) − f ′ (0)

s s3 −9s
= s2 . 2
− s(0) − 0 = 2
−s= 2
s +9 s +9 s +9

Laplace transform of integrals:

161
3 LAPLACE TRANSFORM

hR i F (s)
t
I. If L[f (t)] = F (s) then L 0 f (u)du =
s
Rt
Proof: Let 0 f (u)du = F (t)

∴ F ′ (t) = f (t) and F (0) = 0

We know that L[F ′ (t)] = sL[f (t)] − F (0) = s.L[f (t)]


hR i
t
= s.L 0 f (u)du

hR i 1 1 F (s)
t
(or) L0 f (u)du = L[F ′ (t)] = L[f (t)] =
s s s
 
1
Example: Find L sin 4t
4

s
We know that L[cos 4t] =
s2 + 16
hR i s 1
t
(i.e.) L 0 cos 4udu = = 2 , which can be di-
s(s2 + 16) s + 16
rectly verified.

Rt 1 1
For, cos 4udu = [sin 4u]t0 = sin 4t
0 4 4
 
1 1 4 1
∴L sin 4t = . 2 = 2
4 4 s + 16 s + 16

1
II. If L[f (t)] = F (s) then L[tf (t)] = [F (s)]
s

Proof: We have that L[f (t)] = F (s)

162
3 LAPLACE TRANSFORM

(i.e.) F (s) = L[f (t)] (3.1)

Taking derivatives on both sides of (3.1) w.r.to ’s’, we get

d d d R ∞ −st 
[F (s)] = {L[f (t)]} = e f (t)dt
ds ds ds 0
R ∞ d −st R∞
= 0 ds [e f (t)]dt = 0 (−t)e−st f (t)dt

R∞
= − 0 e−st [tf (t)]dt = −L[tf (t)]

d
(or) L[tf (t)] = − [F (s)]
ds

Example: Find L[tet ]

1
Since L[et ] = , we get
s−1
 
d 1 1
L[tet ] = =
ds s − 1 (s − 1)2
 
f (t) R∞
III. If L[f (t)] = F (s) then L = 0 F (u)du provided
t
f (t)
limt→0 exist.
t

1 sin t
Example: We know that L[sin t] = 2 and limt→0 = 1,
  s + 1 t
sin t R ∞ du 1
then prove that L = s 2
= tan−1
t u +1 s

163
3 LAPLACE TRANSFORM

R∞ du π
∵ s = [tan−1 u]∞
s = − tan−1 s = cot−1 s
u2+1 2
π
(using a theorem, namely cot−1 x + tan−1 x = )
2
1 1
Also using the fact that tan−1 x = cot−1 , cot−1 s = tan−1
x s

IV. Let f (t) be a periodic function with period T (T > 0). Then
its Laplace transform is given by
RT
0 e−st f (t)dt
L[f (t)] =
1 − e−st

3.3 Initial value and Final value theorem

Initial Value Theorem

If L[f (t)] = F (s) then limt→0 f (t) = lims→∞ sF (s), provided the
limits exist.

Proof: We know that

L[f ′ (t)] = sL[f (t)] − f (0) = sF (s) − f (0) (3.2)

Taking limits on both sides of (3.2) as s → ∞, we get

lims→∞ L[f ′ (t)] = lims→∞ [sF (s) − f (0)]


R∞
= lims→∞ 0 e−st f ′ (t)dt = 0

164
3 LAPLACE TRANSFORM

∴ lims→∞ sF (s) = f (0) = limt→0 f (t)

(i.e.) limt→0 f (t) = lims→∞ sF (s)

Final Value Theorem

limt→∞ f (t) = lims→0 sF (s), when L[f (t)] = F (s) such that the
limits exist.

Proof: Again, L[f ′ (t)] = sL[f (t)] − f (0) = sF (s) − f (0)


R∞
(i.e) sF (s) − f (0) = L[f ′ (t)] = 0 e−st f ′ (t)dt
R∞
⇒ lims→0 [sF (s) − f (0)] = lims→0 0 e−st f ′ (t)dt
R∞
= 0 f ′ (t)dt = [f (t)]∞
0

(i.e.) lims→0 sF (s) − f (0) = limt→∞ f (t) − f (0)

(i.e.) limt→∞ f (t) = lims→0 sF (s)

Convolution of the Functions:

Definition: The convolution of the two functions f (t) R t and g(t) is


denoted as f (t) ∗ g(t) and is defined as f (t) ∗ g(t) = 0 f (u).g(t −
u)du

3.4 Convolution Theorem

L[f (t) ∗ g(t)] = L[f (t)].L[g(t)] = F (s).G(s)

165
3 LAPLACE TRANSFORM

(i.e.) The Laplace transform of the convolution of any two func-


tions is the product of their Laplace transforms.
R∞
Proof: Let L[f (t) ∗ g(t)] = 0 e−st [f (t) ∗ g(t)]dt, by definition.

R∞ hR i
t
= 0 e−st 0 f (u)g(t − u)du dt

Z ∞Z t
L[f (t) ∗ g(t)] = e−st f (u)g(t − u)dudt (3.3)
0 0

By changing the order of integration, we get


Z ∞ Z t 
L[f (t) ∗ g(t)] = f (u) −st
e g(t − u)dt du (3.4)
0 0

Let t − u = v ⇒ dt = dv. When t = u ⇒ v = 0

When t = ∞ ⇒ v = ∞

∴ The equation (3.4) becomes,


R∞ R ∞ 
= 0 f (u) 0 e−s(u+v) g(v)dv du
R∞  R∞ 
= 0 f (u) e−su 0 e−sv g(v)dv du
R∞ R∞
= 0 e−su f (u)du. 0 e−sv g(v)dv
R∞ R∞
= 0 e−st f (t)dt. 0 e−st g(t)dt = L[f (t)].L[g(t)]

Thus, L[f (t) ∗ g(t)] = L[f (t)].L[g(t)] = F (s).G(s)


Example 3.1. Find L[2e−3t + 3t2 − 4 sin 2t + 2 cos 3t]

166
3 LAPLACE TRANSFORM

Solution: By the linear property, we have

L[2e−3t +3t2 −4 sin 2t+2 cos 3t] = 2L[e−3t ]+3L[t2 ]−4L[sin 2t]+
2L[cos 3t]
       
1 2! 2 s
=2 +3 −4 +2
s+3 s3 2
s +4 2
s +9

2 6 8 2s
= + 3− 2 + 2
s+3 s s +4 s +9
Example 3.2. Find (i) L[e−2t sin 5t] (ii) L[e3t cosh 4t]

5 5
Solution: (i) Let L[e−2t sin 5t] = 2 2
= 2
(s + 2) + 5 s + 4s + 29

s−3 s−3
(ii) L[e3t cosh 4t] = 2 2
= 2
(s − 3) − 4 s − 6s − 7
     
sin t −1 1 sin 4t
Example 3.3. Given that L = tan , find L
t s t

Solution: By the change of scale property, we have


     
sin 4t 1 sin 4t 1 −1 1
L = L = tan
4t 4 t 4 (s/4)
 
1 4
= tan−1
4 s
   
sin 4t −1 4
∴L = tan
t s
Example 3.4. Find L[sin 3t cos 4t + cos2 2t + 3]

167
3 LAPLACE TRANSFORM

Solution: L[sin 3t cos 4t + cos2 2t + 3]


   
sin(3t + 4t) + sin(3t − 4t) 1 + cos 4t
=L +L + 3L(1)
2 2

1 1
= [L(sin 7t) + L(− sin t)] + [L(1) + L(cos 4t)] + 3L(1)
2 2
   
1 7 1 1 1 s 3
= − + + +
2 s2 + 49 s2 + 1 2 s s2 + 16 s
Example 3.5. Find L[et (cosh t + sinh t)]

Solution: We know that L[eat f (t)] = F (s − a), where F (s) =


L[f (t)]

s 1 s+1
(i.e.) F (s) = L[cosh t] + L[sinh t] = + 2 = 2
s2 −1 s −1 s −1

(s − 1) + 1 s 1
∴ L[et (cosh t + sinh t)] = 2
= 2 =
(s − 1) − 1 s − 2s s−2
Example 3.6. Find L[t3 e−4t ]

Solution: We know that L[e−at f (t)] = F (s + a),

where F (s) = L[f (t)]

3!
Here L[f (t)] = L[t3 ] =
s4
6
(i.e.) F (s) =
s4

168
3 LAPLACE TRANSFORM

6
∴ L[t3 e−4t ] =
(s + 4)4
Example 3.7. Find i) L[t sin 2t] ii) L[t2 cos 3t]

d
Solution: i) We have L[f (t)] = − F (s),
ds
2
where F (s) = L[f (t)] = L[sin 2t] =
s2 +4
 
d 2 2s 4s
∴ L[t sin 2t] = − = 2. 2 = 2
ds s2 + 4 (s + 4)2 (s + 4)2

s
ii) Now, L[cos 3t] =
s2 +9

d2 s
∴ L[t2 cos 3t] = F (s), where F (s) = L[cos 3t] = 2
ds2 s +9
 
d2 s
L[t2 cos 3t] =
ds2 s2 + 9
 
d (s2 + 9)(1) − s.2s
=
ds (s2 + 9)2
 
d 9 − s2 2s3 − 54s
= =
ds (s2 + 9)2 (s2 + 9)3
(
sin t, 0 < t < π
Example 3.8. Find L[f (t)], if f (t) =
0, t > π

R∞
Solution: We know that L[f (t)] = 0 e−st f (t)dt

169
3 LAPLACE TRANSFORM

Rπ R∞
= 0 e−st f (t)dt + π e−st f (t)dt
 π
Rπ e−st
= 0 e−st sin tdt +0= 2 (−s sin t − cos t)
s +1 0

e−sπ (0 − 1)
= 2
(0 − cos π) − 2
s +1 s +1

e−sπ 1 1 + e−sπ
= + =
s2 + 1 s2 + 1 s2 + 1
  
 2π 2π
cos t − ,t>
Example 3.9. Find L[g(t)], if g(t) = 3 3

0, t < 2π
3

Solution: By second
( shifting property, we know if L[f (t)] =
f (t − a), t > a
F (s) and g(t) = , that L[g(t)] = e−as F (s).
0, t < a

s
Here F (s) = L[f (t)] = L[cos t] =
s2 +1
2πs s
∴ L[g(t)] = e 3
s2 +1
 
e−t − e−3t
Example 3.10. Find L
t

Solution: By Laplace transform of f (t)/t, we have that


 
f (t) R∞
L = s F (u)du, if F (s) = L[f (t)]
t

170
3 LAPLACE TRANSFORM

Now L[f (t)] = L[e−t − e−3t ] = L[e−t ] − L[e−3t ]

1 1
= −
s+1 s+3
     
e−t − e−3t R∞ 1 1 s+3
∴L = s − du = ln
t u+1 u+3 s+1
 
cos at − cos bt
Example 3.11. Show that L
t

s s
Solution: L[f (t)] = L[cos at − cos bt] = − 2
+a 2 s2s + b2
   
cos at − cos bt R∞ s s
∴L = s − du
t s 2 + a 2 s 2 + b2
 
1 s 2 + b2
= ln
2 s2 + a 2
Example 3.12. Verify the initial value theorem for the function
2 − cos t.

Solution: Let f (t) = 2 − cos t

lim f (t) = 3 − 2(1) = 1 (3.5)


t→0

3 2s
F (s) = L[f (t)] = 3L(1) − 2L[cos t] = −
s s2 + 1
 
s2
∴ lims→∞ sF (s) = lims→∞ 3−2 2
s +1
lim sF (s) = 3 − 2(1) = 1 (3.6)
s→∞

171
3 LAPLACE TRANSFORM

From (3.5) and (3.6), limt→0 f (t) = lims→∞ sF (s)

Thus the initial value theorem is verified.


Example 3.13. Verify the final value theorem for the function
1 + e−t (sin t + cos t)

Solution: Let f (t) = 1 + e−t (sin t + cos t)


lim f (t) = 1 + 0 = 1 (3.7)
t→∞

Now F (s) = L[f (t)] = L[1] + L[e−t sin t] + L[e−t cos t]

1 1 s
= + +
s (s + 1) + 1 (s + 1)2 + 1
2

1 1 s
= + 2 + 2
s s + 2s + 2 s + 2s + 2
 
s s2
∴ lims→0 sF (s) = lims→0 + +1
s2 + 2s + 2 s2 + 2s + 2
lim sF (s) = 1 + 0 + 0 = 1 (3.8)
s→0

From (3.7) and (3.8), limt→∞ f (t) = lims→0 sF (s)

Thus the final value theorem is verified.



R ∞ e−t sin 3t
Example 3.14. Use Laplace transform to evaluate 0 dt
t

Solution:
Z ∞ −t √ Z ∞  √
e sin 3t sin 3t
dt = −st
e f (t)dt where f (t) =
0 t 0 s=1 t
(3.9)

172
3 LAPLACE TRANSFORM


f (t) R∞
Now since L = s F (u)du, where
t

√ 3
F (s) = L[f (t)] = L[sin 3t] =
s2 + 3
" √ # √
sin3t R∞ 3
∴L = s 2
t u +3
" √ #   ∞  
sin 3t √ 1 u s
L = 3 √ cot −1
√ = cot −1
√ (3.10)
t 3 3 s 3

Using (3.10) in (3.9), we get


√   
R ∞ e−t sin 3t s
0 dt = cot−1 √
t 3 s=1
 
1 π
= cot−1 √ =
3 3

Definition: f (t) is periodic, if f (t + T ) = f (t) for all t, (T > 0)

3.5 Laplace transform of periodic functions

Theorem 3.1. If f (t) has period T > 0, then

1 R T −st
L[f (t)] = 0 e f (t)dt
1−e −sT

R∞
Proof: We know that L[f (t)] = 0 e−st f (t)dt

173
3 LAPLACE TRANSFORM

RT R 2T R 3T
= 0 e−st f (t)dt + T e−st f (t)dt + 2T e−st f (t)dt + · · ·

In the second integral, let t = u + T , in the third integral, let


t = u + 2T , etc.
RT RT
Then L[f (t)] = 0 e−su f (u)du + 0 e−s(u+T ) f (u + T )du
RT
+ 0 e−s(u+2T ) f (u + 2T )du + · · ·
RT
= (1 + e−sT + e−2sT + · · · ) 0 e−su f (u)du

1 R T −su
= 0 e f (u)du
1−e −sT

(∵ f (u + T ) = f (u), f (u + 2T ) = f (u), etc., and using the fact


that s∞ of a G.P. with first term I and common ratio e−sT is
1
)
1 − e−sT

1 R T −st
(or) L[f (t)] = e f (t)dt
1 − e−sT 0
Example 3.15. Find the Laplace transform ( of a periodic func-
1, 0 < t < 1
tion f (t), with period 2, given by f (t) =
−1, 1 < t < 2

Solution: We know that the Laplace transform of a periodic


1 R T −st
function f (t) with period T is L[f (t)] = e f (t)dt
1 − e−sT 0

Here f (t) is a periodic function with period 2.

1 R 2 −st
∴ L[f (t)] = 0 e f (t)dt
1−e −2s

174
3 LAPLACE TRANSFORM

1 hR R 2 −st i
1 −st
= e f (t)dt + e f (t)dt
1 − e−2s 0 1

1 hR R 2 −st i
1 −st
= e (1)dt + e (−1)dt
1 − e−2s 0 1

" 1  −st 2 #
1 e−st e
= −
1 − e−2s −s 0 −s 1
 −s 
1 e 1 e−2s e−s
= + + −
1 − e−2s −s s s s
 
1 1 − 2e−s + e−2s 1 (1 − e−s )2
= = .
1 − e−2s s s (1 + e−s )(1 − e−s )

1 1 − e−s 1 (1 − e−s/2 .e−s/2 )


= . = .
s 1 + e−s s (1 + e−s/2 .e−s/2 )

1 (es/2 − e−s/2 )
= . s/2
s (e + e−s/2 )

x  
1 eθ − e−θ
∴ L[f (t)] = tanh ∵ tanh θ = θ
s 2 e + e−θ
Example 3.16. Find the Laplace transform of the full - sine
wave rectifier f (t) defined as f (t) = | sin ωt|, t ≥ 0

π
Solution: f (t) = | sin ωt| is periodic with period .
ω
1 R ωπ −st
∴ L[f (t)] = e | sin ωt|dt
1 − e−πs/ω 0

175
3 LAPLACE TRANSFORM

1 R ωπ −st
= e sin ωtdt (∵ sin ωt > 0 in [0, π/ω])
1 − e−πs/ω 0

1
= [ωe−πs/ω + ω]
1 − e−πs/ω
" # " #
ω 1 + e−πs/ω ω 1 + e−πs/2ω e−πs/2ω
= 2 = 2
s + ω 2 1 − e−πs/ω s + ω 2 1 − e−πs/ω e−πs/2ω
" #
ω eπs/2ω + e−πs/2ω
= 2
s + ω2 eπs/2ω − e−πs/2ω

 πs   
ω eθ + e−θ
= 2 coth ∵ coth θ = θ
s + ω2 2ω e − e−θ
Example 3.17.(Find the Laplace transform of a function f (t)
t, 0 < t < π
given by f (t) = and that f (t + 2π) = f (t).
0, π < t < 2π

(
t, 0 < t < π
Solution: Given f (t) = such that f (t) is a
0, π < t < 2π
periodic function with period 2π.

1 R 2π −st
∴ L[f (t)] = 0 e f (t)dt
1−e −2πs

1 hR R 2π i
π −st
= e f (t)dt + 0dt
1 − e−2πs 0 π

  −st π  −st π 
1 e e
= t − (1)
1−e −2πs s 0 s2 0

176
3 LAPLACE TRANSFORM
 
1 −πe−sπ (e−sπ − 1)
= −
1 − e−2πs s s2
 
1 1 − e−sπ πe−sπ
= −
1 − e−2πs s2 s
Example 3.18.( Find the Laplace transform of the periodic func-
t, 0 < t < 1
tion f (t) = where f (t + 2) = f (t) for all t
2 − t, 1 < t < 2
where t is positive.

Solution: The given function is a periodic function with period


2.
R2 hR i
0e−st f (t)dt 1 1 −st R 2 −st
∴ L[f (t)] = = e tdt + e (2 − t)dt
1 − e−2s 1 − e−2s 0 1

" 1  −st 2 #
1 e−st t e−st e (2 − t) e−st
= − 1. 2 + − (−1) 2
1 − e−2s s s 0 s s 1

 
1 t e−s e−s 1 e−2s e−s
= + − 2 + 2+ 2 − 2
1 − e−2s s s s s s s
 
1 1 − 2e−s + e−2s (1 − e−s )2
= =
1 − e−2s s2 (1 + e−s )(1 − e−s )s2

1 − e−s 1 1 − e−s 1 es/2 − e−s/2


= = . =
(1 + e−s )s2 s2 1 + e−s s2 es/2 + e−s/2

1 s
= tanh
s2 2

177
3 LAPLACE TRANSFORM

3.6 Inverse Laplace Transform

If L[f (t)] = F (s), then f (t) is called Ïnverse Laplace Transform


of F (s)". This we write as f (t) = L−1 [F (s)]. Here the symbol
L−1 is called the Inverse Laplace transform operator.

Example:

1
We know that L[e2t ] = .
s−2
 
1
Then we can write this as L−1 = e2t .
s−2

Remark: For uniqueness of the inverse Laplace transform, the


sufficient conditions for the existence of L[f (t)] should hold.

From the table of Laplace transform of some elementary functi-


ons, it follows easily the inverse Laplace transforms of F (s), as
given in the following table:

178
3 LAPLACE TRANSFORM

S.No. F (s) L−1 F (s) = f (t)


1
1 1
s
1
2 t
s2
1 tn
3
sn+1 n!
1
4 eat
s−a
1
5 e−at
s+a
a
6 sin at
s + a2
2
s
7 cos at
s + a2
2
a
9 sinh at
s2 − a 2
s
10 cosh at
s − a2
2

As in the case of Laplace tr5ansform, we now write some import-


ant properties of Inverse Laplace transforms.
Property 3.1. Linearity Property:

L−1 [aF (s) + bG(s)] = aL−1 F (s) + bL−1 G(s)af (t) + bg(t)

Example:
     
3 2s 1 s
L−1 − = 3L −1 − 2L −1
s − 1 s2 + 9 s−1 s2 + 9

= 3et − 2 cos 3t
Property 3.2. First shifting property:

If L−1 F (s) = f (t), then L−1 F (s − a) = eat f (t)

179
3 LAPLACE TRANSFORM

Example:
   
s−2 s−2
L−1 =L −1
s2 − 4s + 13 (s − 2)2 + 9
 
s−2
= L−1 = e2t cos 3t
(s − 2)2 + 32
Property 3.3. Second shifting property:

(
f (t − a), t > a
If L−1 F (s) = f (t), then L−1 [e−as F (s)] =
0, t < a

Example:
" #   
e−πs/6 sin t − π , t > π
L−1 = 6 6
s2 + 1 0, t < π
6
Property 3.4. Change of Scale Property:

 
1 t
If L−1 F (s) = f (t), then L−1 [F (as)] = f
a a

Example:
   
3s 1 4t
L−1 2
= cos
(3s) + 16 3 3
Property 3.5. Inverse Laplace transform of derivation:

If L−1 F (s) = f (t), then

180
3 LAPLACE TRANSFORM
 
dn
L−1 [F (n) (s)] = L−1 F (s) = (−1)n tn f (t)
dsn

Example:
  
  
 −1 2
−4s ∵ L = sin 2t
−1 (s2 + 4)
L = −t sin 2t
(s2 + 4)2 
 d 2 −4s
and 2
= 2
ds s + 4 (s + 4)2
Property 3.6. Inverse Laplace transform of integral:

If L−1 F (s) = f (t) and f (0) = 0, then L−1 [sF (s)] = f ′ (t)

Example:
   
s d 1
L−1 = (sin t) = cos t, since L −1 = sin t and
s2 + 1 dt s2 + 1
sin 0 = 0

Property 3.7. Division by s:

 
F (s) Rt Rt
If L−1 F (s) = f (t), then L−1 = 0 f (u)du = 0 L−1 F (s)dt
s

Example:
 
4
Since L−1 = sin 4t, we have
(s2 + 16)
 
4 Rt 1
L−1 2
= 0 sin 4udu = (1 − cos 4t)
s(s + 16) 4

181
3 LAPLACE TRANSFORM

3.7 Convolution Theorem

If L−1 F (s) = f (t) and L−1 G(s) = g(t), then


Rt
L−1 [F (s).G(s)] = 0 f (u)g(t − u)du = f ∗ g = L−1 [F (s)] ∗
L−1 [G(s)] f ∗ g is called the convolution of f (t) and g(t).

Note: f ∗ g = g ∗ f , (i.e.) the convolution is abelian.


 
−1 1
Example 3.19. Find L .
s2 + 9

3
Solution: Since L[sin 3t] =
s2 +9

1 1
L[sin 3t] = 2
3 s +9
 
1 sin 3t
∴ L−1 2 =
s +9 3
 
1
Example 3.20. Find L−1 .
s2 − 3

" √ #
3 √
Solution: Since L √ = sinh 3t
s − ( 3)2
2

1 √ 1
√ L[sinh 3t] = 2
3 s −3
  √
1 sinh 3t
L−1 = √
s2 − 3 3

182
3 LAPLACE TRANSFORM
 
4 4 + 2s
Example 3.21. Find L−1 +
2s − 4 16s2 − 4

Solution:
     
−1 4 4 + 2s −1 4 −1 4
L + 2
= L +L +
2s − 4 16s
 −4 2s − 4 16s2 − 4
s
2L−1 2
16s − 4
   
 
2  4   s 
= L−1 + L−1 
  
 + 2L−1 
  
s−2 4 4 
16 s2 − 16 s2 −
16 16
     
1 1/4 2 −1 s
= 2L−1 +L −1 + L
s−2 s2 − (1/4) 16 s2 − (1/4)
     
1 1 −1 1/2 1 −1 s
= 2L−1 + L + L
s−2 2 s2 − (1/2)2 8 s2 − (1/2)2

1 1 1 1
= 2e2t +sinh t + cosh t
2 2 8 2
 
3s + 7
Example 3.22. Find L−1 2
s − 2s − 3

   
3s + 7 3s + 7
Solution: L−1 =L −1
s2 − 2s − 3 (s − 1)2 − 4
     
3(s − 1) + 10 s−1 2
= L−1 = 3L −1 +5L −1
(s − 1)2 − 4 (s − 1)2 − 22 (s − 1)2 − 22

183
3 LAPLACE TRANSFORM

= 3et cosh 2t + 5et sinh 2t = et [3 cosh 2t + 5 sinh 2t]


Example 3.23. Using Laplace transform of the derivatives, find
L(t cos at).

Solution: We know that

L[f ′′ (t)] = s2 L[f (t)] − sf (0) − f ′ (0) (3.11)

Let f (t) = t cos at ⇒ f (0) = 0

Then f ′ (t) = −at sin at + cos at ∴ f ′ (0) = 1

Also f ′′ (t) = −a[at cos at + sin at] − a sin at

Equation (3.11) becomes

L(−a[at cos at + sin at + a sin at]) = s2 L[t cos at] − 0 − 1

L[−a2 t cos at] − L[2a sin at] = s2 L[t cos at] − 1

1 − L[2a sin at] = (s2 + a2 )L[t cos at]

2a2
⇒ (s2 + a2 )L[t cos at] = 1 − 2aL[sin at] = 1 −
s2+ a2

s2 + a2 − 2a2 s2 − a 2
= =
s2 + a 2 s2 + a 2

s2 − a 2
(or) L(t cos at) =
(s2 + a2 )2
 
−1 s+1
Example 3.24. Find L 2
s +s+1

184
3 LAPLACE TRANSFORM
   
s+1 (s + 1/2) + 1/2
Solution: L−1 =L −1
s2 + s + 1 (s + 1/2)2 + 3/4
  " √ #
(s + 1/2) 1 −1 3/2
= L−1 √ +√ L √
(s + 1/2)2 + ( 3/2)2 3 (s + 1/2)2 + ( 3/2)2
√ √
3 1 −t/2 3
= e−t/2 cos t+ √ e sin t
2 3 2
" √ √ #
e−t/2 √ 3 3
= √ 3 cos t + sin t
3 2 2
 
−1 s
Example 3.25. Find L
(s + 2)2

   
s 1
Solution: L−1 =L −1 s.
(s + 2)2 (s + 2)2
 
d −1 1 d
= L 2
= [e−2t L−1 (1/s2 )]
dt (s + 2) dt

d −2t
= (e t) = e−2t (1) + t.e−2t (−2) = e−2t (1 − 2t)
dt
 
−1 (s + 1)2
Example 3.26. Find L
s2 + 2s + 5

Solution:
   
(s + 1)2 s2
L−1 2 = e−t L−1 , by the first shifting
s + 2s + 5 (s2 + 22 )2
property.

185
3 LAPLACE TRANSFORM
   
s d s
= e−t L−1 s. 2 =e −t L −1
(s + 22 )2 dt (s2 + 22 )2
 
d t
= e−t sin 2t
dt 4
 
1 1
= e−t (sin 2t + 2t cos 2t) = e−t (sin 2t + 2t cos 2t)
4 4
 
s 1
Example 3.27. Given that L−1 2 − a 2 )2
= sinh at, find
  (s 2a
s2
L−1
(s2 − a2 )2

   
s2 s
Solution: L−1 =L −1 s. 2
(s2 − a2 )2 (s − a2 )2
   
d s d t
= = sinh at
dt (s2 − a2 )2 dt 2a

1 1
= [t cosh at.a + sinh at] = [sinh at + at cosh at]
2a 2a
 
−1 1
Example 3.28. Find L
s(s2 + 2s + 2)

Solution:
   
1 1 1
L −1
2
=L −1 F (s) where F (s) = 2
s(s + 2s + 2) s s + 2s + 2

186
3 LAPLACE TRANSFORM

Rt
= 0 L−1 [F (s)]dt
  Z t  
1 1
∴L −1
= L −1
(3.12)
s(s2 + 2s + 2) 0 s2 + 2s + 2
   
1 1
Now, L−1 2 = L−1
s + 2s + 2 (s + 1)2 + 12
   
1 1
L −1 −t −1
=e L = e−t sin t (3.13)
s2 + 2s + 2 s2 + 12

Using (3.13) in (3.12), we get


 
1 Rt
L−1 2
= 0 e−t sin tdt
s + 2s + 2

e−t e−t
= [(1 − sin t + cos t)] = [1 − sin t + cos t]
1+1 2
 
−1 1
Example 3.29. Find L
s(s + 2)3

Solution:
  Z t
1 1
L −1
3
= L−1 F (s)dt, where F (s) =
s(s + 2) 0 (s + 2)3
  (3.14)
1 t 2
L−1 F (s) = e−2t L−1 3 = e−2t (3.15)
s 2
Using (3.15) in (3.14), we get
 
1 R t −2t t2 1 R t 2 −2t
L−1 = 0 e dt = t e dt
s(s + 2) 3 2 2 0

187
3 LAPLACE TRANSFORM

    −2t   −2t t


1 2 e−2t e e
= t − 2t +2
2 2 4 −8 0
  2  
1 −2t t t 1 1
= e − − − +
2 2 2 4 4

1 1
= [e−2t (−2t2 − 2t − 1) + 1] = [1 − e−2t (2t2 + 2t + 1)]
8 8
Example 3.30. Evaluate each of the following using convolution
theorem:
   
−1 s −1 s2
(i) L , (ii) L
(s2 + a2 )2 (s2 + a2 )(s2 + b2 )

Solution:
   
s s
(i) L −1 can be written as L −1
(s2 + a2 )2 (s2 + a2 ).(s2 + a2 )
 
s
L−1 = f ∗ g, by convolution theorem
(s2 + a2 ).(s2 + a2 )
 
Rt s
= f (u)g(t − u)du, where f (t) =
0 L−1 = cos at,
  (s + a2 )
2
1 1
g(t) = L−1 = sin at
(s2 + a2 ) a

Rt 1
= 0 cos au sin a(t − u)du
a
1 Rt
= cos au[sin at cos au − cos at sin au]du
a 0

188
3 LAPLACE TRANSFORM

1 hR t R
2 audu − t cos at sin au cos audu
i
= sin at cos
a 0 0

   
1 R t 1 + cos 2au R t sin 2au
= sin at 0 du − cos at 0 du
a 2 2
"     #
1 1 sin 2au t cos at − cos 2au t
= sin at u+ −
a 2 4a 0 2 2a 0

    
1 t sin 2at cos at cos 2at − 1
= sin at + +
a 2 4a 2 2a
    
1 t sin at cos at 1 − cos 2at
= sin at + − cos at
a 2 2a 4a
    
1 t sin at cos at 2 sin2 at
= sin at + − cos at
a 2 2a 2.2a
 
1 t sin2 at cos at sin at cos at t sin at
= sin at + − =
a 2 2a 2a 2a
 
s2
(ii) L−1 can be written as
(s2 + a2 )(s2 + b2 )
 
s s
L−1 . 2
(s + a ) (s + b2 )
2 2

   
s s
= L−1 ∗L −1 = f ∗ g = cos at ∗ cos bt
(s2 + a2 ) (s2 + b2 )
Rt
= 0 cos au cos b(t − u)du

189
3 LAPLACE TRANSFORM

R t cos(au + bt − bu) + cos(au − bt − bu)


= 0 du
2
1 hR t Rt i
= cos{(a − b)u + u}du + cos cos{(a − b)u − bt}du
2 0 0

"    #
1 sin(bt + (a − b)u) t sin(−bt + (a + b)u) t
= +
2 a−b 0 a+b 0

    
1 sin(bt + at − bt) sin(−bt + at + bt) sin bt sin bt
= + − +
2 a−b a+b a−b a+b
 
1 2a sin at − 2b sin bt a sin at − b sin bt
= =
2 a 2 − b2 a 2 − b2
 
3s + 16
Example 3.31. Using partial fractions, find L−1
s2 − s − 16

3s + 16 3s + 16 A B
Solution: = = +
s2 − s − 16 (s − 3)(s + 2) s−3 s+2

⇒ 3s + 16 = A(s + 2) + B(s − 3)

s = 3 ⇒ 25 = 5A (i.e.) A = 5

s = −2 ⇒ 10 = −5B (i.e.) B = −2
     
3s + 16 5 −2
L−1 =L −1 +L −1 = 5e3t − 2e−2t
s2 − s − 16 s−3 s+2
 
−1 11s2 − 2s + 5
Example 3.32. Find L
(s − 2)(2s − 1)(s + 1)

190
3 LAPLACE TRANSFORM

11s2 − 2s + 5 A B C
Solution: = + +
(s − 2)(2s − 1)(s + 1) s − 2 2s − 1 s + 1

⇒ 11s2 −2s+5 = A(2s−1)(s+1)+B(s−2)(s+1)+C(s−2)(2s−1)

s = 2 ⇒ 45 = 9A ⇒ A = 5

s = −1 ⇒ 18 = 9C ⇒ C = 2

27 9
s = 1/2 ⇒ = − B ⇒ B = −3
4 4
     
−1 11s2 − 2s + 5 −1 5 −1 −3
L = L +L +
 (s − 2)(2s
 − 1)(s + 1) s−2 2s − 1
2
L−1
s+1
     
1 1 1
= 5L−1 − 3L −1 + 2L −1
s−2 2s − 1 s+1
     
1 3 1 1
= 5L−1 − L−1 + 2L−1
s−2 2 s − (1/2) s+1

3 1
= 5e2t − e 2 t + 2e−t
2
 
s2 − 2s + 3
Example 3.33. Find L−1
(s − 1)2 (s + 1)

s2 − 2s + 3 A B C
Solution: = + +
(s − 1)2 (s + 1) (s − 1)2 s − 1 s + 1

⇒ s2 − 2s + 3 = A(s + 1) + B(s − 1)s + 1) + C(s − 1)2

191
3 LAPLACE TRANSFORM

s = −1 ⇒ 4C = 6 ⇒ C = 3/2

s = 1 ⇒ 2A = 2 ⇒ A = 1

s = 0 ⇒ A − B + C = 3 ⇒ B = −1/2
       
s2 − 2s + 3 1 1/2 3/2
∴ L−1 =L −1 −L −1 +L −1
(s − 1)2 (s + 1) (s − 1)2 s−1 s+1
     
1 1 −1 1 3 −1 1
= et L−1 − L + L
s2 2 s−1 2 s+1

1 3 1 3
= tet − et + e−t = et (2t − 1) + e−t
2 2 2 2
 
−1 27 − 2s
Example 3.34. Find L
(s + 4)(s2 + 9)

27 − 2s A Bs + C
Solution: 2
= + 2
(s + 4)(s + 9) s+4 s +9

⇒ 27 − 2s = A(s2 + 9) + (Bs + C)(s + 4)

Comparing like co-efficients of s2 , s and constant terms, on both


sides of the above equation, we get

A+B =0 (3.16)

4B + C = −2 (3.17)
9A + 4C = 27 (3.18)
From (3.16), B = −A

(i.e.) (3.17) becomes −4A + C = −2

192
3 LAPLACE TRANSFORM

Equation (3.18) is 9A + 4C = 27

Solving these two equations, we get −25A = −35

∴ A = 7/5 and B = −7/5

From equation (3.17), C = 2 + 28/5 = 18/5


     
s2 − 2s + 3 7 −1 1 7 −1 s
∴ L−1 2
= L − L +
 (s − 1) (s + 1) 5 s+4 5 s2 + 9
18 −1 s
L
5 s2 + 9

7 7 18
= e−4t − cos 3t + sin 3t
5 5 5
 
s2 − 3
Example 3.35. Evaluate L−1
(s + 2)(s − 3)(s2 + 2s + 5)

s2 − 3 As + B Cs + D
Solution: 2
= 2 + 2
(s + 2)(s − 3)(s + 2s + 5) s − s − 6 s + 2s + 5

⇒ s2 − 3 = (As + B)(s2 + 2s + 5) + (Cs + D)(s2 − s − 6) (3.19)

Comparing the co-efficients of s3 , s2 , s and constant terms on


both sides of above equation, we get

A + C = 20 (3.20)

2A + B − C + D = 1 (3.21)
5A + 2B − 6C − D = 0 (3.22)
5B − 6D = −3 (3.23)

193
3 LAPLACE TRANSFORM

Equation (3.21) + (3.22) gives 7A + 3B − 7C = 1 (3.24)


Equation (3.22) × 6 − (3.23) ⇒ 30A + 7B − 36C = 3 (3.25)
Equation (3.24) × 7 − (3.25) × 3 gives − 41A + 59C = −2 (3.26)
Equation (3.19) × 59 gives 59A + 59C = 0 (3.27)
Equation (3.26) − (3.27) ⇒ −100A = −2 ⇒ A = 1/50

41 59
From (3.26), 59C = −2 + = − ⇒ C = −1/50
50 50
7 7 14 36
Equation (3.24), + 3B + = 1 ⇒ 3B = 1 − =
50 50 50 50

⇒ B = 12/50

60 60 210
From (3.25), − 6D = −3 ⇒ −6D = −3 − =−
50 50 50

⇒ D = 35/50
h s2 − 3 i
L−1
(s + 2)(s − 3)(s2 + 2s + 5)
   
1 12 1 35
s+ − s+
 50   50 
= L−1  50  + L−1  250  → (I)
s2 + 2s + 5 s + 2s + 5

Consider,
 
1 35  
s+
− 1 −1 s+1 35 1
 50 
L−1  250  = − L +
s + 2s + 5 50 (s + 1)2 + 22 50 (s + 1)2 + 22

194
3 LAPLACE TRANSFORM
   
1 −t 1 −1 1 35 −1 1
= − e cos 2t+ L + L
50 50 (s + 1)2 + 22 50 (s + 1)2 + 22

1 −t 1 −t 35 −t
=− e cos 2t + e sin 2t + e sin 2t → (i)
50 100 100
   
1 12 1 12
s+ s+
 50   50 
Now consider L−1  50
2  = L−1  50 
s −s−5 (s + 2)(s − 3)

1 12
s+
⇒ 50 50 = A + B
(s + 2)(s − 3) s+2 s−3

1 3
⇒A=− and B =
25 50
 
1 12    
s+ 1 1 3 1
−1  50 50 
L  2 =− +
s −s−5 25 s + 2 50 s − 3

 
1 35

s+ 1 3
 50 
L−1  250  = − e−2t + e3t → (ii)
s + 2s + 5 25 50

Using (i) and (ii) in (I), we get


 
s2 − 3 3 1 1
L−1 = e3t − e−2t − e−t cos 2t+
(s + 2)(s − 3)(s2 + 2s + 5) 50 25 50
9 −t
e sin 2t
25

195
3 LAPLACE TRANSFORM

3.8 Application of Laplace Transform to


Solve Differential Equations

Laplace transform is useful in solving differential (both ordina-


ry and partial) equations. We confine ourselves to apply this
method to solve only ordinary linear differential equations with
constant co-efficients.

The following steps are to be followed while solving the linear


differential equations.

Step 1: Take Laplace transforms of both sides of the given dif-


ferential equation.

Step 2: Use the formula of Laplace transform of the derivatives


and substitute the initial conditions given.

Step 3: Finally, take Inverse Laplace transforms to get the re-


quired solution.

Solved Problems:

Using Laplace transform, solve the following differential


equations:

Example 3.36. y ′′ − 3y ′ − 4y = 2e−t , y(0) = 1, y ′ (0) = 1.

Solution:

Given that y ′′ − 3y ′ − 4y = 2e−t such that y(0) = y ′ (0) = 1


(3.28)

196
3 LAPLACE TRANSFORM

Taking Laplace transforms of both sides of equation (3.28), we


get

L[y ′′ − 3y ′ − 4y] = 2L[e−t ]

(i.e.) L[y ′′ ] − 3L[y ′ ] − 4L[y] = 2L[e−t ]

1
[s2 y(s) − sy(0) − y ′ (0)] − 3[sy(s) − y(0)] − 4y(s) = 2.
s+1

2
(s2 − 3s − 4)y(s) − s − 1 + 3 = (∵ y(0) = y ′ (0) = 1)
s+1

2
(i.e.) (s2 − 3s − 4)y(s) = +s−2
s+1

2 s−2
∴ y(s) = + 2
(s + 1)(s2− 3s − 4) (s − 3s − 4)
 
1 s−2
=2 2
+ 2
(s + 1) (s − 4) (s − 3s − 4)
   
A B C D E
∴ y(s) = 2 + + + +
s + 1 (s + 1)2 s − 4 s+1 s−4
 
F G
−2 +
s+1 s−4
   
−1/25 1/5 1/25 1/5 4/5
=2 + + + +
s+1 (s + 1)2 s − 4 s+1 s−4
 
1/5 1/5
−2 +
s+1 s−4

197
3 LAPLACE TRANSFORM

Taking the Inverse Laplace transform of both sides, we get

2 −t 2 −t 2 1 4 2 2
y(t) = − e − te + e4t + e−t + e4t + e−t − e4t
25 5 25 5 5 5 5
13 −t 12 4t 2 −t
= e + e − te
25 15 5
1
= (13e−t − 10te−t + 12e4t )
25
Example 3.37. Solve: (D2 + 4D + 13)y = e−t sin t, y = 0 and
d
Dy = 0 at t = 0, where D ≡ .
dt

Solution: Given (D2 + 4D + 13)y = e−t sin t

Taking Laplace transform on both sides, we get

1 1
(s2 + 4s + 13)y(s) = L[e−t sin t] = 2 2
= 2
(s + 1) + 1 s + 2s + 2

1
∴ y(s) =
(s2 + 2s + 2)(s2 + 4s + 13)

As + B Cs + D
= +
s2 + 2s + 2 s2 + 4s + 13

By the usual partial fraction method, the constants can be found


as A = −2/85, B = 7/85, C = 2/85 and D = −3/85.
 
1 −2s + 7 2s − 3
∴ y(s) = +
85 s2 + 2s + 2 s2 + 4s + 13

198
3 LAPLACE TRANSFORM
 
1 −2(s + 1) + 9 2(s + 2) − 7
= +
85 (s + 1)2 + 12 (s + 2)2 + 32

Taking Inverse Laplace transforms on both sides, we get


   
1 h −1 −2(s + 1) + 9 1
L−1 [y(s)] = L + 9L−1
85 (s + 1)2 + 12 (s + 1)2 + 12
   i
s+2 7 3
+2L−1 − L−1
(s + 2)2 + 32 3 (s + 2)2 + 32
  
1 −t −2t 7
∴ y(t) = e (−2 cos t + 9 sin t) + e 2 cos 3t − sin 3t
85 3
Example 3.38. Solve: x′′ +4x′ +3x = 10 sin t, x(0) = x′ (0) = 0.

Solution: Given x′′ + 4x′ + 3x = 10 sin t, x(0) = x′ (0) = 0

Taking Laplace transforms on both sides and using initial con-


ditions, we get

10
(s2 + 4s + 3)x(s) =
s2 +1

10
∴ x(s) =
(s2 + 1)(s2 + 4s + 3)
 
As + B C D
= 10 + + (using partial fractions)
s2 + 1 s+1 s+3
 
(−1/5)s + 1/10 1/4 (−1/20)
= 10 + +
s2 + 1 s+1 s+3

199
3 LAPLACE TRANSFORM

Taking Inverse Laplace transforms on both sides, we get


     
5 −1 1 1 −1 1 s
L−1 [x(s)] = L − L − 2L −1 +
 2 s+1 2 s+1 s2 + 1
1
L−1
s2 +1

5 1
= e−t − e−3t − 2 cos t + sin t
2 2
Example 3.39. Solve: y ′′ +2y ′ −3y = sin t, given y(0) = y ′ (0) =
0.

Solution: Given y ′′ + 2y ′ − 3y = sin t, given y(0) = y ′ (0) = 0

Taking Laplace transforms on both sides, we get

s2 L[y(t) − sy(0) − y ′ (0) + 2(sL(y(t)) − y(0)) − 3L[y(t)]] = L[sin t]

1
(i.e.) (s2 + 2s − 3)y(s) =
s2 +1

1 1
∴ y(s) = . 2
s2 + 1 s + 2s − 3
1
∴ y(s) = (3.29)
(s − 1)(s + 3)(s2 + 1)
Consider
1 A B Cs + D
2
= + + 2 (3.30)
(s − 1)(s + 3)(s + 1) s−1 s+3 s +1
Using usual procedure, we can find the constants A, B, C and D
as A = −1/8, B = −1/40, C = −1/10 and D = −1/5

200
3 LAPLACE TRANSFORM

(i.e.) The equation (3.29) becomes,

1/8 1/40 (1/10)s + 1/5


∴ y(s) = − −
(s − 1) s + 3 s2 + 1

Taking Inverse Laplace transforms on both sides, we get


     
1 −1 1 1 −1 1 1 −1 s
L−1 [y(s)]= L − L − L −
 8 s−1 40 s+3 10 s2 + 1
1 −1 1
L 2
5 s +1

1 1 1 1
(i.e.) y(t) = et − e3t − cos t − sin t
8 40 10 5
Example 3.40. Solve: (D2 + 4)y = cos 2t, y(0) = 3, y ′ (0) = 4

Solution: Given (D2 + 4)y = cos 2t, y(0) = 3, y ′ (0) = 4

Taking Laplace transforms on both sides, we get

L[y ′′ (t)] + 4L[y(t)] = L[cos 2t]

s
⇒ {s2 L[y(t)] − s(3) − 4} + L[y(t)] =
s2 + 4
s
(i.e.) y(s)(s2 + 4) = + 3s + 4
s2 +4

s 3s + 4
(or) y(s) = + 2
(s2 + 4)2 s +4
     
s s 4 −1 2
L−1 [y(s)] = L−1 + 3L −1 + L
(s2 + 4)2 s2 + 4 2 s2 + 4

201
3 LAPLACE TRANSFORM

t
= sin 2t + 3 cos 2t + 2 sin 2t
2×2

t
= sin 2t + 3 cos 2t + 2 sin 2t
4
Example 3.41. Solve: x′′ − 2x′ + x = e−t ,given that x(0) =
2, x′ (0) = 1.

Solution: Given x′′ − 2x′ + x = e−t

Taking Laplace transforms on both sides, we get

L[x′′ ] − 2L[x′ ] + L[x] = L[e−t ]

1
{s2 x(s) − s(2) − 1} − 2{sx(s) − 2} + x(s) =
s+1

1
(i.e.) x(s)(s2 − 2s + 1) = + 2s − 3
s+1

1 2s − 3
x(s) = + 2 (3.31)
(s + 1)(s2− 2s + 1) s − 2s + 1
1 A B C
Consider = + + (3.32)
(s + 1)(s2 − 2s + 1) s + 1 s − 1 (s − 1)2
It can be found that A = 1/4, B = −1/4, C = 1/2.

Using the values of A, B, C (from (3.32)) in equation (3.31), we


get

1 1 1 2s 3
x(s) = − + 2
+ 2

4(s + 1) 4(s − 1) 2(s − 1) (s − 1) (s − 1)2

202
3 LAPLACE TRANSFORM

Taking Inverse Laplace transforms on both sides, we get


     
1 −1 1 1 −1 1 2s − 3 + 1/2
L−1 [x(s)] = L − L +L −1
4 s+1 4 s−1 (s − 1)2
       
1 −1 1 1 −1 1 −1 1 5 −1 1
= L − L +2L − L
4 s+1 4 s−1 (s − 1)2 2 (s − 1)2

1 1 5
(i.e.) x(t) = e−t − et + 2(et + tet ) − tet
4 4 2
   
1 1 5
∴ x(t) = et 2 − + e−t + tet 2 −
4 4 2

7 1 1 1
= et + e−t − tet = (7et + e−t − 2tet )
4 4 2 4
Example 3.42. Solve: (D2 − 4D + 8)y = e2t , given y(0) =
2, y ′ (0) = −2.

Solution: Given (D2 − 4D + 8)y = e2t

Taking Laplace transforms on both sides, we get

1
{s2 y(s) − s(2) + 2} − 4{sy(s) − 2} + 8y(s) =
s−2

1
y(s)[s2 − 4s + 8] = + 2s − 10
s−2

1 2s − 10
∴ y(s) = + 2
(s − 2)(s2− 4s + 8) s − 4s + 8

By the partial fraction method, the above equation can be writ-

203
3 LAPLACE TRANSFORM

ten as
7 19
1/4 s−
y(s) = + 4 2
s − 2 s2 − 4s + 8

7
1 1 (s − 2) − 6
= . + 42
4 s−2 s − 4s + 8

Taking Inverse Laplace transforms on both sides, we get


   
1 −1 1 7 −1 s−2
L−1 [y(s)] = L + L
4 s−2 4 (s − 2)2 + 4
 
6 −1 2
− L
2 (s − 2)2 + 4

1 7
∴ y(t) = e2t + e2t cos 2t − 3e2t sin 2t
4 4
1
= e2t [1 + 7 cos 2t − 12 sin 2t]
4

EXERCISE
 
2s − 12
1. Find L[2 cos 4t − 3 sin 4t] Ans: 2
s + 16
 
2s3 − 6a2 s
2. Find L[t2 cos at] Ans: 2
(s + a2 )3
 
3s2 + 2s − 3
3. Find L[5 − 3t − 2e−t ] Ans: 2
s (s + 1)

204
3 LAPLACE TRANSFORM
 
12s
4. Find L[sin 2t sin 3t] Ans: 2
(s + 1)(s2 + 25)
 
s+2
5. Find L[e−2t cos t] Ans: 2
s + 4s + 5
 
1 − 5s
6. Evaluate L[e−t (3 sinh 2t − 5 cosh 2t)] Ans:
s2 + 2s + 3
(
(t − 1)2 , t > 1
7. Find L[f (t)] if f (t) =
0, 0 < t < 1
 
2e−s
Ans: 3
s

R∞ 3
8. Show that 0 te−3t sin tdt = , by using Laplace trans-
5
form.
 
R ∞ cos 6t − cos 4t 3
9. Evaluate 0 dt using L.T. Ans: ln
t 2

10. Find the Laplace transform of the following periodic func-


tions:
(
t, 0 < t < π
(i) f (t) = given that f (t + 2π) = f (t)
0, π < t < 2π
(
sin t, 0 < t < π
(ii) f (t) =
0, π < t < 2π

205
3 LAPLACE TRANSFORM
 
1 1 −πs π −πs
Ans : (i) (1 − e ) − e
1 − e−2πs s2 s

1
(ii)
(s2 + 1)(1 − e−πs

11. Find the Laplace transform of the following functions:

1 − cos t sin 3t sin t


(i) (ii)
t t
  2   2 
1 s +1 1 s + 16
Ans:(i) log (ii) log
2 s2 4 s2 + 4

12. Using the Laplace transform of the derivatives find the


following:

(i) L[t cos at] (ii) L[t sinh at] (iii) L(sin2 t)
      
s2 − a 2 2as 2
Ans:(i) (ii) (iii)
(s2 + a2 )2 (s − a2 )2
2 2
s(s + 4)
hR i s

t −t
13. Evaluate L 0 te dt Ans:
s(s + 1)2
 
R t 1 − e−t
14. Evaluate L 0 dt
t

15. Using convolution theorem, evaluate the following:


Rt
0 e−u sin(t − u)du

16. Verify the initial value theorem for t + sin 3t.

206
3 LAPLACE TRANSFORM

17. Verify the initial value theorem for 1 + e−t (sin t + cos t)

18. Find the Inverse Laplace transform of the following:


s s
(i) (ii) 2
s2 − 16 s +2

[Ans: (i) cosh 4t (ii) cos 2t]
   
6s − 4 3s + 2
19. Find (i) L−1 (ii) L −1
s2 − 4s + 20 4s2 + 12s + 9
 
3 5
Ans:(i)2e2t (3 cos 4t + sin 4t) (ii) e−3t/2 − te−3t/2
4 8
 
e−3s
20. Find L−1
s2 − 2s + 5
  
 1 (t−3) sin 2(t − 3), t > 3
Ans: 2 e 
0, t < 3

  
2 2 sin t sinh t
21. Find L−1 tan−1 2 Ans:
s t
  
s+2 2 1 1
22. Find L−1 Ans: t + − e−3t
s2 (s + 3) 3 9 9

23. Using convolution theorem, find the Inverse Laplace trans-


forms of the following:

1 1 1
(i) 2
(ii) 2
(iii) 2 2
(s + 1)(s + 1) s(s + 1) s (s + 25)

207
3 LAPLACE TRANSFORM

s2
(iv)
(s2 + 4)2

1
Ans: (i) (sin t − cos t + e−t ) (ii) 1 − cos t
2
1 1 1
(iii) (t cos 4t + sin 4t − sin 5t) (iv) t cos 2t + sin 2t
5 2 4

24. Using partial fractions, find the following:


 
s−1
(i) L−1
(s + 3)(s2 − 2s + 2)
   
s2 − 2s + 3 3s + 1
(ii) L−1 (iii) L −1
(s − 1)2 (s + 1) (s − 1)(s2 + 1)

1 4 1 3
Ans: (i) e−t (4 cos t−3 sin t)− e−3t (ii) (2t−1)et + e−t
5 5 2 2

(iii) 2et − 2 cos t + sin t

25. Using Laplace transforms solve the following differential


equations:

(i) (D2 + D)y = t2 + 2t, given y(0) = 4, y ′ (0) = −2

(ii) y ′′ − 3y ′ + 2y = e2t , given y(0) = −3, y ′ (0) = 5


 
t2
Ans:(i) + 4e−t + 2 (ii)te2t − 10et + 7e2t
3

26. Solve y ′′ + y ′ − 2y = 3 cos 3t − 11 sin 3t, given that y(0) = 0

208
3 LAPLACE TRANSFORM

and y ′ (0) = 6. [Ans: sin 3t − e−2t + et ]

27. Solve: x′′ − 2x′ + x = t2 et , x(0) = 2, x′ (0) = 3.

[Ans: x = (t4 /12 + t + 2)et ]

28. Define Laplace transforms.

29. State the conditions for the existence of Laplace transform


of a function.
 
s3
30. Find L[cosh at cos at] Ans: 4
s + 4a4

31. Define unit step function.


(
sin t, for 0 < t < π
32. Find L[f (t)] if f (t) =
0, for t > π
 
1
Ans: (1 − e−(s−2)
s−2

33. State first shifting property and second shifting property


of Laplace transforms.

14s + 10
34. Find the Inverse Laplace transform of
49s2 + 28s + 13
  
2 (2/7)t 3 3
Ans: e cos t + sin t
7 7 7

t
35. If L[f (t)] = F (s), prove that L f = aF (as).
a

209
3 LAPLACE TRANSFORM

36. State the initial value theorem in Laplace transforms.

37. State the final value theorem in Laplace transforms.


  
1
38. Find L e2t cosh 2t + sinh 2t
2
  r  
1 + 2t π 1
39. Find L √ Ans: 1+
t s s

40. State the formula for the Laplace transform of a periodic


function and hence find the Laplace transform of f (t) = t,
in 0 < t < 1.

1
41. If L[f (t)] = , then find limt→0 f (t) [Ans: − sin 3]
(s − 2)2
 
s
42. Find L−1 [Ans: e−2t (cos t − 2 sin t)]
s2 + 4s + 5

43. State the convolution theorem for Laplace transforms.

44. Prove that f (t) ∗ g(t) = g(t) ∗ f (t)

45. Verify initial value theorem for the function 1 + e−2t


hR i   2 2

46. Find (i) L
t
t cos tdt (ii) L −1 s log s + a
0 s 2 + b2

s2 + 2s
Ans: (i)
(s + 1)(s2 + 2s + 2)

210
3 LAPLACE TRANSFORM
 
1 2(cos bt − cos at)
(ii) 2(a sin at − b sin bt) −
t t

47. Find the Laplace transform of full sine wave rectifier given
below:
 π
E sin ωt, 0 < t <  
ω 2π
f (t) = with f t + = f (t)
0, π < t < 2π ω
ω ω
 
ω
Ans:L[f (t)] =
(1 − e−sπω (s2 + ω 2 )

48. Prove that Laplace transform ( of the triangular wave of


t, 0 ≤ t ≤ π
period 2π defined by f (t) =
2π − t, π ≤ t ≤ 2π

1  πs 
is tanh
s2 2
(
t, 0 < t ≤ 2
49. Find the Laplace transform of f (t) =
4 − t, 2 ≤ t < 4
and satisfy f (t + 4) = f (t).
   2 
s2 + 4 2 s +1
50. Given that L−1 = t cosh t, find L −1
(s2 − 4)2 (s2 − 1)2

211
4 COMPLEX INTEGRATION

Introduction

Let f (z) be a continuous function of the complex variable z =


x + iy defined at every point of a curve ′ C ′ whose end points
are A and B. Let us divide the curve C into ′ n′ parts by points
A = p0 (z0 ), p1 (z1 ), p2 (z2 ), · · · pi (zi ) = B. Let δzi = zi−1 − zi and
let
Pnεi be a point on the arc pi−1 − pi . Then the limit of the sum
i=1 f (εi )δzi and n → ∞ in such a way that each δzi → 0, if it
exits
R is called the line integral of f (z) along C and is denoted by
C f (z)dz.

If C is a closed curve, i.e. if p0 and pn coincide


H the integral is
called the contour integral and is denoted by C f (z)dz

Complex Integration: The integral of a complex function is


defined as the limit of a certain sum in the same manner as
integral of a real function along the x-axis is defined.

Multiple Point: If a curves intersects itself at a point then the


point is said to be a multiple point of the curve.

Simple curve: A continuous curve which does not have a point


of self intersection is called a simple curve. Simple curves are
called Jordan curves.

212
4 COMPLEX INTEGRATION

Positively oriented simple closed curve: A simple closed


curve C encloses a region, if the region lies to the left of a person
when he travels along ′ C ′ , then curve ′ C ′ is called positively
oriented simple closed curve.

Contour Integral: An integral along a simple closed curve is


called a contour integral..

Simply - Connected Region: A region D is said to be sim-


ply - connected if, for every closed curve ′ C ′ in D, Ci is wholly
contained in D. Simply - connectedness of a region is equivalent
to the absence of holes in it or to the situation in which every
closed curve in D can shrink to a point, all the while being in D
itself.

Multiply - Connected Region: A region which is not simply


- connected is called multiply - connected region.

4.1 Evaluation of line integral

The evaluation of a line integral is reduced to the evaluation of


two real line integrals as follows:

Since z = x + iy, dz = dx + idy, f (z) = u + iv


R R R R
C f (z)dz = C (u + iv)(dx + idy) = C (udx − vdy) + i C (vdx +
idu)
R dz
Example 4.1. Evaluate C (z − z0 )n+1

213
4 COMPLEX INTEGRATION

Solution: Let z − z0 = reiθ , so that θ varies from 0 to 2π as z


describes the circle C.

R 2π reiθ i i R 2π
I= 0 iθ n+1
dθ = n 0 e−inθ dθ
(re ) r
R 2π
Case(i): when n = 0, I = i 0 dθ = 2πi

Case(i): when n 6= 0,

1 R 2π 1
I= n 0 (cos nθ − i sin nθ)dθ = n (sin nθ + i cos nθ)2π
0 =0
r nr
R 1+i
Example 4.2. Evaluate 0 (x − y + ix2 )dz
(i) along the straight line between the limits (0, 0) and (1, 1)
(ii) over the path along the lines y = 0, x = 1
(iii) over the path along the lines x = 0 and y = 1
(iv) along the parabola y 2 = x

Solution:

(i) Along the path OP, y = x, dy = dx


R1 R1
∴I= 0 (x − y + ix2 )(dx + idy) = 0 (x − x + ix2 )(1 + i)dx

R1 1
= (i − 1) 0 x2 dx = (i − 1)
3

(ii) Along the path OAP,


Z Z
2
I= (x − y + ix )dz + (x − y + ix2 )dz = I1 + I2 (4.1)
OA AP

Along the line OA, y = 0, dy = 0

214
4 COMPLEX INTEGRATION

R1 R1 3 + 2i
I1 = 0 xdx + i 0 x2 dx =
6

Along the line AP, x = 1, dx = 0

R1 R1 i−2
I2 = 0 (−y)dy + i 0 (1 − y)dy =
2
3 + 2i i − 2 5i − 3
Equation (4.1) becomes I = + =
6 2 6

(iii) Along the path OBP,


I = along the line OB + along the line BP = I1 + I2 (4.2)
Along the line OB, x = 1, dx = 0

R1 i
I1 = i 0 (−y)dy = −
2

Along the line BP, y = 1, dy = 0

R1 −1 R1 i
I2 = 0 (x − 1)dx + i + 0 x2 dx =
2 3
 
i −1 i −3 − i
Equation (4.2) becomes I = − + + =
2 2 3 6

(iv) Along the parabola y 2 = x ⇒ 2ydy = dx


R1 R1
I= 0 (x − y + ix2 )dz = 0 (y 2 − y + iy 4 )(2y + i)dy
 1
y 4 2y 3 y6 y3 y2 y5 −11 i
= − +i +i −i − = +
2 3 3 3 2 5 0 30 6
R
Example 4.3. Evaluate C zdz from A(0, 0) to B(4, 2) along the

215
4 COMPLEX INTEGRATION

curve C and z = t2 + it.

Solution: Let z = x − iy, z = x + iy = t2 + it ⇒ x = t2 , y = t


so that dx = 2tdt, dy = dt and dz = dx + idy = 2tdt + idt =
(2t + i)dt. Also x = 0, 4 ⇒ t = 0, 2 and y = 0, 2 ⇒ t = 0, 2.

R R1 8
Hence I = C zdz = 0 (t2 − it)(2t + i)dt = 10 − i
3
R
Note: The integral udx + vdy is independent of the path if
∂u ∂v
= .
∂y ∂x
R
Example 4.4. Find the value of the integral C (x+y)dx+x2 ydy,
(i) along y = x2 having (0, 0), (3, 9) as end points, (ii) along
y = 3x between the same points. Do the values depend upon the
path?

Solution:
Z Z
2
Let I = (x + y)dx + x ydy = udx + vdy (4.3)

∂u ∂v
Here u = x + y and v = x2 y and we have = 1, = x2 so
∂x ∂y
∂u ∂v
that 6= . hence the integral is dependent on path.
∂x ∂y

(i) Along y = x2 , dy = 2xdx and x varies from 0 to 3. Now


equation (4.3) becomes
R3 R3
I= 0 (x + x2 )dx + x2 .x2 .2xdx = 0 (x + x2 + 2x5 )dx
 3
x2 x3 x6 9 27 729
= + + = + + = 256.5
2 3 6 0 2 3 3

216
4 COMPLEX INTEGRATION

(ii) Along y = 3x, dy = 3dx and x varies from 0 to 3. Now


equation (4.3) becomes
 3
R3 x4
I= 0 (x + 3x)dx + x2 (3x)3dx = 2x2 +9 = 200.25
4 0
R
Example 4.5. Evaluate C 3y 2 dx + 2ydy where C is a circle
x2 + y 2 = 1 counter clockwise from (1, 0) to (0, 1).

R
Solution: Let I = C 3y 2 dx + 2ydy. Given x2 + y 2 = 1.

Differentiate w.r.to x and y, we get

2dx + 2ydy = 0 ⇒ ydy + xdx = 0 ⇒ ydy = −xdx


R0
∴I= 1 3(1 − x2 )dx − 2xdx = [3x − x3 − x2 ]01 = 1

4.2 Cauchy’s Integral Theorem

Statement: If f (z) is analytic and f ′ (z) is continuous


H at all
points inside and on a simple closed curve C, then C f (z)dz = 0.

Proof: Let f (z) = u + iv, z = x + iy and dz = dx + idy.


H H H
Then C f (z)dz = C (u+iv)(dx+idy) = C (udx+iudy +ivdx−
vdy) I I
f (z)dz = (udx − vdy) + i(udy + vdx) (4.4)
C C
We know by Green’s theorem in a plane,

217
4 COMPLEX INTEGRATION
 
H RR ∂u ∂v
C (udx + vdy) =
∂y
+
∂x
dxdy

Equation (4.4) becomes


   
H RR ∂u ∂v ∂u ∂v
C f (z)dz = − dxdy − dxdy + i dxdy − dxdy
∂y ∂x ∂x ∂y
   
RR ∂u ∂v R R ∂u ∂v
= − − dxdy + i − dxdy
∂y ∂x ∂x ∂y
   
RR ∂u ∂u R R ∂u ∂u
− + dxdy + i − dxdy = 0, by C - R
∂y ∂y ∂x ∂x
∂u ∂v ∂u ∂v
equations = , =−
∂x ∂y ∂y ∂x
H
That is C f (z)dz = 0.

4.2.1 Cauchy’s Theorem for Multiply Connected


Region

Statement: If f (z) is analytic and f ′ (z) is continuous at all


pointsH in the regionH bounded by simple closed curves C1 and C2 ,
then C1 f (z)dz = C2 f (z)dz.

Corollary: If f (z) is analytic and f ′ (z) is continuous at all


points in the region between the curves C1 , C2 , C3 , · · · Cn which
lies entirely within a simple closed curve C, then
H H H H
C f (z)dz = C1 f (z)dz + C2 f (z)dz + · · · Cn f (z)dz

218
4 COMPLEX INTEGRATION

Cauchy’s Fundamental Formula (OR) Cauchy’s Integral


Formula

Statement: If f (z) is analytic and f ′ (z) is continuous and if ′ a′


1 H f (z)
is any point inside C then f (a) = dz.
2πi C z − a

f (z)
Proof: The function F (z) = is analytic at all points ex-
z−a
f (z)
cept z = a. Since is analytic with ′ a′ as centre and radius
z−a
′ r ′ draw a small circle C which lies entirely within C.
1
I I
f (z) f (z)
dz = dz (4.5)
C z − a C1 z −a

Let z − a = reiθ ⇒ z = a + reiθ ⇒ dz = 0 + reiθ dθ = rieiθ dθ.

Equation (4.5) becomes


I I
f (z) f (a + reiθ )rieiθ
dz = dθ (4.6)
C z−a C1 reiθ

As r → 0, the circle C1 shrinks to the point ′ a′ , but θ = 0 to 2π.

Equation (4.6) becomes

H f (z) R 2π
C dz = i 0 f (a)dθ = if (a)[θ]2π
0 = if (a)2π = 2πif (a)
z−a
I
1 f (z)
⇒ f (a) = dz (4.7)
2πi C z − a

Corollary: Differentiate (4.7) w.r.to a, we get

219
4 COMPLEX INTEGRATION
 
1 H (−1)
f ′ (a) = (−1) f (z)dz
2πi C (z − a)2

1 H f (z)
⇒ f ′ (a) = C dz
2πi (z − a)2

Differentiate again w.r.to a, we get


 
1 H (−2)
f ′′ (a) = (−1) f (z)dz
2πi C (z − a)3

2! H f (z)
f ′′ (a) = dz
2πi C (z − a)3

Differentiate again w.r.to a, we get


 
2! H (−3) 2! × 3 H f (z)
f ′′′ (a) = (−1) f (z)dz = dz.
2πi C (z − a) 4 2πi C (z − a)4

3! H f (z)
That is f ′′′ (a) = C dz etc.
2πi (z − a)4

n! H f (z)
In general, f n (a) = C dz
2πi (z − a)n+1

Note: We recall that a point z0 at which a function f (z) is not


analytic is known as singular point or a singularity of f (z). To
find the singularity of f (z), equate the denominator of f (z) to
zero and solve it for z. For example the singularity of f (z) =
z+3
is obtained as (z − 1)(z − 2) = 0
(z − 1)(z − 2)

z − 1 = 0, z − 2 = 0 ⇒ z = 1, z = 2.

220
4 COMPLEX INTEGRATION

H e−z
Example 4.6. Evaluate C dz where C is a circle (i) |z| =
z+1
1
2, (ii) |z| =
2

Solution: Singular points are obtained by putting z + 1 = 0 ⇒


z = −1.

(i) The singular point z = −1 lies within the circle |z| = 2.

1 H f (z)
Then by Cauchy’s integral formula f (a) = dz ⇒
2πi C z − a
H f (z)
C z − a dz = 2πif (a), we have

H e−z H e−z
C [z − (−1)] dz = 2πif (−1). Here f (z) = e , a =
dz = −z
C z+1
H e−z
−1 and f (−1) = e. Hence C dz = 2πie.
z+1

1
(ii) The singular point z = −1 lies outside |z| = . Then
2
H f (z)
C z + 1 dz = 0 by Cauchy’s theorem.

H cos πz 2
Example 4.7. Evaluate C dz where C is a circle
(z − 1)(z − 2)
|z| = 3.

Solution: Singular points are obtained by putting (z − 1)(z −


2) = 0, z = 1, 2. Both the singular points lies within |z| = 3.
1 A B
Now = + (4.8)
(z − 1)(z − 2) z−1 z−2

221
4 COMPLEX INTEGRATION

Multiply both sides of (4.8) by (z − 1)(z − 2), we get 1 = A(z −


2) + B(z − 1).

When z = 2, 1 = 0 + B(2 − 1) ⇒ B = 1.

When z = 1, 1 = A(z − 2) = −A ⇒ A = −1.

Hence equation (4.8) becomes

1 −1 1
= +
(z − 1)(z − 2) z−1 z−2
 
H cos πz 2 H 1
Now C dz = C cos πz 2 dz
(z − 1)(z − 2) (z − 1)(z − 2)

H cos πz 2 H cos πz 2
=− C (z − 1) dz + C (z − 2) dz

 
H −1 1
= C + cos πz 2 dz = −I1 + I2
z−1 z−2
I
cos πz 2
I1 = dz (4.9)
C (z − 1)

By Cauchy’s integral formula


I I
1 f (z) f (z)
f (a) = dz ⇒ dz = 2πif (a) (4.10)
2πi C z − a C z−a

Comparing (4.9) and (4.10), we get a = 1, f (z) = cos πz 2 ⇒


f (1) = cos π = −1.

Equation (4.9) implies that

222
4 COMPLEX INTEGRATION

H cos πz 2
I1 = C (z − 1) dz = 2πif (1) = 2πi(−1) = −2πi

I
cos πz 2
Also I2 = dz = 2πif (2) (4.11)
C (z − 2)

We have f (z) = cos πz 2 ⇒ f (2) = cos 4π = 1.

Equation (4.11) becomes

H cos πz 2
I2 = C (z − 2) dz = 2πif (2) = 2πi(1) = 2πi.

H cos πz 2
∴ C dz = −(−2πi) + 2πi = 4πi
(z − 1)(z − 2)
H e2z
Example 4.8. Using Cauchy’s integral formula evaluate C dz
(z + 1)4
where C is a circle |z| = 2.

Solution: Singular points are obtained by putting (z + 1)4 =


0 ⇒ z = −1 which lies inside the circle C. By Cauchy’s integral
formula

3! H f (z) H f (z) 2πi ′′′


f ′′′ (a) = C 4
dz ⇒ C 4
dz = f (a).
2πi (z − a) (z − a) 3!
I I
e2z f (z) 2πi ′′′
Hence 4
dz = 4
dz = f (−1) (4.12)
C (z + 1) C [z − (−1)] 3!
where f (z) = e2z , f ′ (z) = 2e2z , f ′′ (z) = 4e2z , f ′′′ (z) = 8e2z and
8
f ′′′ (−1) = 8e−2 = 2 .
e

Now equation (4.12) becomes

223
4 COMPLEX INTEGRATION
 
H e2z 2πi ′′′ 2πi 8 16πie−2 8πie−2
C 4
dz = f (−1) = = =
(z + 1) 3! 3! e2 6 3
H 3z 2 + z
Example 4.9. Evaluate C z 2 − 1 dz where C is a circle |z −
1| = 1.

Solution: Singular points are obtained by putting z 2 − 1 = 0 ⇒


z 2 = 1.

⇒ z = −1, 1. The singular point z = 1 lies within the circle


|z − 1| = 1 but z = −1 lies outside the circle |z − 1| = 1.

1 1 A B
Now = = + (4.13)
z2 −1 (z + 1)(z − 1) z+1 z−1

Multiply both sides of (4.13) by (z + 1)(z − 1), we get

1 = A(z − 1) + B(z + 1)

When z = 1, 1 = 0 + 2B ⇒ B = 1/2

When z = −1, 1 = A(−2) ⇒ A = −1/2. hence equation (4.13)


becomes
1 1 1
=− +
z2 −1 2(z + 1) 2(z − 1)

H 3z 2 + z H 1 2 + z)dz +
H 1 2
C z2 − 1 dz = C 2(z + 1) (3z C 2(z − 1) (3z + z)dz

1 H 3z 2 + z 1 H 3z 2 + z
=− dz + dz
2 C (z + 1) 2 C (z − 1)

224
4 COMPLEX INTEGRATION

1 1
= − (2πi)f (−1) + (2πi)f (1), where f (z) = 3z 2 + z
2 2

= −π × 0 + πi × 4 = 4πi where f (1) = 4.

H 3z 2 + z
Hence C z 2 − 1 dz = 4πi.

H ze2z
Example 4.10. Evaluate C dz where C is a circle |z +
(z − 1)3
i| = 2.

Solution: Singular points are obtained by putting (z − 1)3 =


0 ⇒ z = 1 is a singular point lies inside the circle |z + i| = 2.

2! H f (z)
By Cauchy’s integral formula f ′′′ (a) = C dz.
2πi (z − a)3
I I
ze2z f (z) 2πi ′′
dz = dz = f (1) where f (z) = ze2z
C (z − 1)3 C (z − a) 3 2!
(4.14)
Let f (z) = ze , f (z) = 2ze +e , f (z) = 2[2ze +e ]+2e2z
2z ′ 2z 2z ′′ 2z 2z

and f ′′ (−1) = 2(2e2 + e2 ) = 8e2 .

Equation (4.14) becomes

H ze2z 2πi ′′ 2πi 2


C dz = f (1) = (8e ) = 8πie2
(z − 1)3 2! 2
H dz
Example 4.11. Evaluate C over the circle |z − 2| = 1.
z2 − 2z

Solution: Singular points are obtained by putting z 2 − 2z = 0

225
4 COMPLEX INTEGRATION

⇒ z(z − 2) = 0 ⇒ z = 0, 2 and |z − 2| = 1 is a circle with centre


(2, 0) and radius 1. Only the singular point z = 2 lies within the
circle |z − 2| = 1. Hence by Cauchy’s integral formula

H dz H dz H z −1 dz H f (z)dz
I= C = C z(z − 2) = C z−2 = C z−2
z 2 − 2z

1 1
= 2πif (2), where f (z) = , f (2) =
z 2
 
1
= 2πi = πi
2
H cos zdz
Example 4.12. Evaluate the integral where C is an
z
ellipse 9x2 + 4y 2 = 1.

Solution: Here z = 0 is a singular point which lies within an


x2 y2
ellipse + = 1. Hence by Cauchy’s integral formula
1/9 1/4

H cos zdz H f (z)


I = = dz = 2πif (0) = 2πi × 1 = 2πi, where
z z
f (z) = cos z
H z2 + 1
Example 4.13. Evaluate C z 2 − 1 dz where C is a circle with
unit radius and centre 1.

Solution: Singular points are z 2 − 1 = 0 ⇒ z = ±1, hence only


z = 1 lies within the circle. Hence by Cauchy’s integral formula

226
4 COMPLEX INTEGRATION

z2 + 1
H z2 + 1 H z2 + 1 H z + 1 dz
I= C z 2 − 1 dz = C (z − 1)(z + 1) dz = C z−1

H f (z) z2 + 1
= C dz = 2πif (z) = 2πi, where f (z) =
z−1 z+1

and f (1) = 1.
H dz
Example 4.14. Evaluate where C is a circle |z| = 2.
z 3 (z + 4)

Solution: Singular points are obtained by putting z 3 (z + 4) =


0 ⇒ z = 0 and z = −4 of which only z = 0 lies inside the given
circle.

By Cauchy’s integral formula

1
H dz H z+4 H f (z) 1
I= = dz = dz, f (z) =
z 3 (z + 4) z3 (z − 0)3 z+4

2πi ′′ 1 2
= f (0), where f ′ (z) = , f ′′ (z) =
2! (z + 4)2 (z + 4)3
 
1 2 1
= πi , where f ′′ (0) = =
32 (0 + 4)3 32

EXERCISE

H sin 3zdz
1. Evaluate if C is a circle |z| = 5.
z + (π/2)

227
4 COMPLEX INTEGRATION

H dz
2. Evaluate where C is a circle |z| = 1.
2z − 3
H dz
3. Evaluate where C is a circle |z| = 1.
zez
H dz
4. Evaluate where C is a circle |z| = 1.
(z − 3)2

H tan(z/2)dz
5. Evaluate , (−2 < a < 2) where C is the boun-
(z − a)2
dary of a square whose sides lie along x = ±2 and y = ±2
described in the positive sense.

H zez
6. Show that dz = πiea (a + 2) where z = a lies
(z − a)3
inside the closed curve C, using Cauchy’s integral formula.

H z−2
7. Show that dz = 2πi where C is a circle |z| = 3.
z(z − 2)

H dz
8. Evaluate where C is |z| = 3.
z2 +4

H (7z − 1)dz
9. Evaluate where C is an ellipse x2 + 4y 2 = 4.
z 2 − 3z − 4

H (z + 1)dz
10. Without using residue theorem, evaluate whe-
z 3 − 2z 2
re C is a unit circle |z| = 1.

H ez dz
11. Evaluate if (i) 0 lies inside C and 1 lies outside
z(1 − z)3
C, (ii) 1 lies inside C and 0 lies outside C and (iii) 0 lies

228
4 COMPLEX INTEGRATION

inside C.

H 2z 2 − z − 2
12. If C is a circle |z| = 3 and if g(z0 ) = dz then
z − z0
find g(2). Hint: g(z0 ) = 2πif (z0 ) = 2πif (2) = 8πi = g(2).

H z2 + 1
13. Evaluate dz where C is the circle |z + 1| = 4.
z2 − 1
H dz
14. Find the value of if C is |z − i| = 2.
(z 2 + 4)2

4.3 Taylor’s Series and Laurent’s Series

Now, we develop Taylor’s series and Laurent’s series to expand


an analytic function f (z) in powers in z.

Taylor’s Theorem(Taylor’s Series)

A function f (z) is analytic at all points inside a circle C, with


its centre at the point ′ a′ and radius R, we can expand

(z − a) ′ (z − a)2 ′′ (z − a)3 ′′′


f (z) = f (a) + f (a) + f (a) + f (a)
1! 2! 3!

(z − a)n n
+··· + f (a) + · · · ∞.
n!

P∞ (z − a)n n
= n=1 f (a), where f n (a) denotes the nth derivative.
n!

229
4 COMPLEX INTEGRATION

Corollary 1. Putting a = 0, we get

z ′ z2 z3
f (z) = f (a) + f (a) + f ′′ (a) + f ′′′ (a)
1! 2! 3!

zn n
+··· + f (a) + · · · ∞.
n!

Corollary 2. Putting z − a = h ⇒ z = a + h, we get

h ′ h2 h3
f (a + h) = f (a) + f (a) + f ′′ (a) + f ′′′ (a)
1! 2! 3!

hn n
+··· + f (a) + · · · ∞.
n!

Laurent’s Theorem(Laurent’s Series)

If f (z) is analytic on two concentric circles C1 and C2 of radii r1


and r2 (r1 > r2 ) with centre at ′ a′ and also on the annular region
R bounded by C1 and C2 then for all z in R.

P∞ P∞ bn
f (z) = n=0 an (z − a)n + n=1 = I1 + I2
(z − a)n

I1 is called the regular part (contain positive powers) and I2 is


called the principal part (contain negative powers) of f(z) and

1 H f (z) 1 H f (z)
an = C n+1
dz, bn = C dz both the
2πi 1 (z − a) 2πi 2 (z − a)1−n
integrals being taken anticlockwise direction.

Note: Note the two formulae:

230
4 COMPLEX INTEGRATION

(1 + x)−1 = 1 − x + x2 − x3 + · · ·

(1 − x)−1 = 1 + x + x2 + x3 + · · ·
z−1
Example 4.15. Obtain the expansion of f (z) = in Tay-
z2
lor’s series in powers of (z − 1) and give the region of validity
and Laurent’s series for |z − 1| > 1

z−1 1 1
Solution: (i) Let f (z) = 2
= − 2 . Now we want f(z) as a
z z z
series in powers of (z − 1). The function is not analytic at z = 0.

Let u = z − 1 ⇒ z = u + 1 |u| < 1.

z−1 u
f (z) = 2
= = u(1 + u)−2
z (u + 1)2

= u(1 − 2u + 3u2 − 4u3 + · · · ) = u − 2u2 + 3u3 − · · ·

= [(z − 1) − 2(z − 1)2 + 3(z − 1)3 − · · · is a Taylor’s series valid


for |z − 1| > 1.

1
(ii) Let u = z − 1 ⇒ z = u + 1 |u| > 1. (i.e.) 1 < |u|. < 1.
|u|
 
z−1 u u 1 1 −2
∴ f (z) = 2
= =  2 = 1+
z (u + 1)2 1 u u
u2 1 +
u

1 2 3
= − 2 + 3 − ···
u u u

231
4 COMPLEX INTEGRATION

1 2 3
= − + −· · · valid expansion for |z−1| < 1.
z − 1 (z − 1) (z − 1)3
2
2
Example 4.16. Expand ze−z in Laurent’s series about z = 0.

 
2 z2 z4 z6
Solution: Let ze−z =z 1− + − + ···
1! 2! 3!

z3 z5 z7
=z− + − + · · · . Here this Laurent’s series does not
1! 2! 3!
contain negative powers of z.
1 − cos z
Example 4.17. Expand in Laurent’s series about z =
z
0.

Solution:
  
1 − cos z 1 z2 z4
Let = 1− 1− + − ···
z z 1! 2!

z z3 z5
= − + − · · · through z = 0 appears to be a singularity
2! 4! 6!
if (1 − cos z)/z, Laurent’s series of (1 − cos z)/z at z = 0 does
not contain negative powers of z.
Example 4.18. Find the residue at z = 0 of (i) f (z) = e1/2

(ii) f (z) = sin z/z 4

1 1
Solution: (i) Here f (z) = 1 + + 2 + ···
z z /2

Residue of f (z) at z = 0 = coefficient of 1/z in the Laurent’s


expansion = 1.

232
4 COMPLEX INTEGRATION
 
1 z3 z5 1 1 z
(ii) Let f (z) = 4 z − + − ··· = 3 − + − ···
z 3! 5! z 3!z 5!

Residue of f (z) at z = 0 = coefficient of 1/z = −1/6


sin z
Example 4.19. Expand about z = π.
z−π

Solution: Put z − π = t.

sin z sin(π + t) sin t


Then = =−
z−π t t
 
1 t3 z 5
=− t− + − ···
t 3! 5!

(z − π)2 (z − π)4
= −1 + − + ···
3! 5!
z
Example 4.20. Expand f (z) = about z = −2.
(z + 1)(z + 2)

Solution: Put z + 2 = t ⇒ z = t − 2.

t−2 2−t
Then f (z) = = (1 − t)−1
t(t − 1) t

1 2
= (2 + t + t2 + · · · ) = + 1 + t + t2 + · · ·
t t
2
= + 1 + (z + 2) + (z + 2)2 + · · ·
z+2
 
1+z
Example 4.21. Expand log at z = 0 using Taylor’s
1−z
series.

233
4 COMPLEX INTEGRATION

Solution:
 
1+z
Let f (z) = log = log(1 + z) − log(1 − z) ⇒ f (0) = 0
1−z

−1 1
Then f ′′ (z) = 2
+ ⇒ f ′′ (0) = 0 etc.
(1 + z) (1 − z)2

The Taylor’s series of f (z) about the point z = a is

(z − a)2 ′′
f (z) = f (a) + (z − a)f ′ (a) + f (a) + · · ·
2!

The Taylor’s series of f (z) about the point z = 0 is

z 2 ′′ 4
f (z) = f (0) + zf ′ (0) + f (0) + · · · = 2z + z 3 + · · ·
2! 3!
1
Example 4.22. Expand f (z) = as Laurent’s series in
z(z − 1)
powers valid in |z| < 1 and |z| > 1.

1 1 1 1
Solution: Let =− − = − − (1 − z)−1
z(z − 1) z 1−z z
 
1 n P∞ n
(i) Let f (z) = − − 0 z is a valid expansion for |z| < 1.
z

1
(ii) Let |z| > 1 ⇒ < 1.
|z|

1 1 1
Then =− +  
z(z − 1) z 1
z 1−
z

234
4 COMPLEX INTEGRATION

       
1 1 1 −1 1 1 P∞ 1 n
= − + 1− = − + is a valid
z z z z z 0 z
expansion for |z| > 1.
1
Example 4.23. If f (z) = about z = 0 valid in
(z − 1)(z − 2)
(i) |z| < 1, (ii) 1 < |z| < 2 and (iii) |z| > 2.

1 1 1
Solution: Let f (z) = =− +
(z − 1)(z − 2) z−1 z−2

1 1
(i) In |z| < 1, f (z) = −  
1−z 2 1− z
2
1 P∞  z  n
= (1+z +z 2 +· · · )− is a valid expansion for |z| < 1.
2 0 2

1 |z|
(ii) Let 1 < |z| < 2 ⇒ < 1, < 1.
|z| 2

1 1 1 1
Then f (z) = − + =   + z 
z−1 z−2 1 2 − 1
z 1− 2
z
 
1 1 −1 1  z −1
=− 1− − 1−
z z 2 2
 n
1 P∞ 1 1 P ∞  z n
=− − is a valid expansion for |z| > 2
z 0 z 2 0 2
1
Example 4.24. Expand f (z) = in the region
(z − 1)(z − 2)
0 < |z − 1| < 1.

235
4 COMPLEX INTEGRATION

1
Solution: Let f (z) =
(z − 1)(z − 2)

1 1
= − = −[1 − (z − 1)]−1 − (z − 1)−1
[(z − 1) − 1] z − 1

= −(z − 1)−1 + [1 + (z − 1) + (z − 1)2 + · · · ]


1
Example 4.25. Find Laurent’s series expansion of
(z + 1)(z + 3)
in powers of (z + 1) for range 0 < |z + 1| < 2.

Solution: Put z + 1 = u, then 0 < |z + 1| < 2 ⇒ 0 < |u| < 2.

1 1 1
Now = =  u
(z + 1)(z + 3) u(u + 2) 2u 1 +
2
 
1  u −1 1 u  u 2
= 1+ = 1− + − ···
2u 2 2u 2 2

1 1 u u2
= − + − + ···
2u 4 8 16

Replace u = z + 1, we have

1 1 1 (z + 1) (z + 1)2
= − + − + ···
(z + 1)(z + 3) 2(z + 1) 4 8 16
Example 4.26. Find the Taylor’s series and Laurent’s series
z2 − 1
expansion to represent the function f (z) = where
(z + 2)(z + 3)
(i) |z| < 2, (ii) 2 < |z| < 3 and (iii) |z| > 3.

236
4 COMPLEX INTEGRATION

z2 − 1 B C
Solution: Let =A+ +
(z + 2)(z + 3) z+2 z+3

⇒ z 2 − 1 = A(z + 2)(z + 3) + B(z + 3) + C(z + 2) (4.15)

Put z = −3 in (4.15), we get 9 − 1 = −C ⇒ C = −8.

Put z = −2 in (4.15), we get 4 − 1 = B ⇒ B = 3 and put z = 0


in (4.15), we get −1 = 6A + 9 − 16 ⇒ A = 1.

z2 − 1 3 8
Then =1+ −
(z + 2)(z + 3) z+2 z+3
z

(i) Let |z| < 2 ⇒ < 1
2

z2 − 1 3 8
=1+  z −  z
(z + 2)(z + 3) 2 1+ 3 1+
2 3
3 z −1 8  z −1
=1+ 1+ − 1+
2 2 3 3
3P  z n 8  z n
= 1+ (−1)n − (−1)n which is the expansion
2 2 3 3
of Taylor’s series valid for |z| < 2.

2 |z|
(ii) Let 2 < |z| < 3 ⇒ |z| > 2 and |z| < 3 ⇒ < 1 and < 1.
|z| 3

z2 − 1 3 8
∴ =1+ −
(z + 2)(z + 3) z+2 z+3

237
4 COMPLEX INTEGRATION

 −1
3 8 3 2 8 z −1
= 1+   − z  = 1+ +1 − 1+
2 3 +1 z z 3 3
z +1 3
z
 n  z n
3P 2 8
= 1+ (−1)n − (−1)n is a suitable expansion
z z 3 3
for 2 < |z| < 3.

|z| 3
(iii) Let |z| > 3 ⇒ >1⇒ < 1.
3 |z|

z2 − 1 3 8
Now =1+ −
(z + 2)(z + 3) z+2 z+3
 −1  
3 8 3 2 8 3 −1
= 1+  −   = 1+ +1 − 1+
2 3 z z z z
z +1 z +1
z z
 n  n
3P 2 8 3
=1+ (−1)n − (−1)n is a suitable expansion
z z z z
for |z| > 3.
4z + 3
Example 4.27. Represent the function in Lau-
z(z − 3)(z + 2)
rent’s series, (i) within |z| = 2, (ii) in the annular region between
|z| = 2 and |z| = 3 and (iii) exterior to |z| = 3.

4z + 3 A B C
Solution: Let = + +
z(z − 3)(z + 2) z z−3 z+2

⇒ 4z + 3 = A(z − 3)(z + 2) + Bz(z + 2) + Cz(z − 3) (4.16)

Put z = 0 in (4.16), we get 3 = A(6) ⇒ A = 1/2.

238
4 COMPLEX INTEGRATION

Put z = −2 in (4.16), we get −5 = C(−2)(−5) ⇒ C = −1/2


and put z = 3 in (4.16), we get 15 = B(3) ⇒ B = 5.

4z + 3 1 5 C
Hence = + −
z(z − 3)(z + 2) 2z z − 3 2(z + 2)

|z| |z|
(i) Within |z| = 2 ⇒ |z| < 2 ⇒ < 1 and <1
2 3
4z + 3 1 5 1
Now = + −
z(z − 3)(z + 2) 2z z − 3 2(z + 2)

z −1 5 1
= +  z −  z
2 −3 1 − 4 1+
3 2

z −1 5  z −1 1  z −1
= − 1− − 1+
2 3 3 4 2

z −1 5 P  z n 1  z n
= − −
2 3 3 4 2

(ii) In the annular region between |z| = 2 and |z| = 3

2 |z|
2 < |z| < 3 ⇒ 2 < |z| and |z| < 3 ⇒ < 1 and <1
|z| 3

4z + 3 1 5 1
Now = +  −  z
z(z − 3)(z + 2) 2z 3 4 1 +
z 1− 2
z
 
z −1 5 3 −1 1  z −1
= + 1− − 1+
2 z z 4 2

239
4 COMPLEX INTEGRATION
 n  z n
z −1 5 P 3 1P
= + − (−1)n
2 z z 4 2

3
(iii) Exterior to |z| = 3 ⇒ |z| > 3 ⇒ <1
|z|

4z + 3 1 5 1
Now = + −
z(z − 3)(z + 2) 2z z − 3 2(z + 2)

z −1 5 1
= −  −  
2 3 2
z 1− 2z 1 +
z z
   
z −1 5 3 −1 1 2 −1
= − 1− − 1+
2 z z 2z z
 n  n
z −1 5 P 3 1 P n 2
= − − (−1)
2 z z 2z z

EXERCISE

π
1. Expand cos z as a Taylor’s series about the point z = .
4
π 
h P∞ fn  π n
Ans:f (z) = 4 z −
n=0
n! 4
π i
the region of convergence is z − < ∞
4
1
2. Find the Taylor’s series for about z = 2, z = i.
z

240
4 COMPLEX INTEGRATION
 
P∞ (−1)n P∞ (−1)n n!
Ans:(ii)f (z) = 0 (ii) f (z) = 0 n+1
1n+1 (z − 1)n i (z − 1)n

3. Find
 the Taylor’s series expansion of sec z in powers of
π
z− .
4
 
√ √  π 3  π 2
Ans:f (z) = 2 + 2 z − + √ z− + ···
4 2 4

4. Find the Taylor’s series expansion about z = 0 of f (z) =


z
.
(z + 1)(z − 3)
   
1 z −1
Ans:f (z) = −1
(1 + z) − 1 −
4 3
 n
1 1 P∞ z−2
5. Show that 2 = (−1)n (n + 1) when |z −
z 4 0 2
2| < 2
 
1+z
6. Expand log at z = 0 using Taylor’s series.
1−z
h  
z3 z5
Ans:f (z) = 2 z + + + ··· which converges for
i 3 3
|z| < 1

1 P n
7. Show that 2
= ∞0 (n + 1)(z + 1) when |z + 1| < 1.
z
1
8. Find the Laurent’s series for f (z) = in the
z2 + 3z + 2

241
4 COMPLEX INTEGRATION

region 1 < |z| < 2.


"  −1 #
1 1 1 z −1
Ans:f (z) = 1+ − 1+
2 z 2 2

z
9. Expand f (z) = as Laurent’s series valid for
(z − 1)(z − 3)
(i) 1 < |z| < 3 (ii) 0 < |z − 1| < 2.

h  
1 1 −1 1  z −1
Ans: (i) − 1− − 1− ,
2z " z 2 3 #
   2 i
−1 3 z−1 z−1
(ii) − 1+ + + ···
2(z − 1) 4 2 2

7z − 2
10. Find the Laurent’s expansion of of f (z) =
z(z − 2)(z + 1)
valid in 1 < |z + 1| < 3.
 
2 P 1 2 P∞ (z + 1)n
Ans: − + ∞2 −
z+1 (z + 1)n 3 0 zn

1
11. Find the Laurent’s series of f (z) = valid in the
z(1 − z)
region (i) |z +1| < 1, (ii) 1 < |z +1| < 2 and (iii) |z +1| > 2.
 
P∞ 1
Ans: (i) f (z) = 0 −1 + (z + 1)n
2n+1

P∞ 1 P 1
(ii) f (z) = 0 n+1
+ ∞0 n+1
(z + 1)n
(z + 1) 2

242
4 COMPLEX INTEGRATION

P∞ 1
(iii) f (z) = 0 (1 − z n )
(z + 1)n+1

z
12. Find the Laurent’s series expansion of f (z) =
(z − 1)(z − 2)
valid in the region (i) |z + 2| < 3, (ii) 3 < |z + 2| < 4 and
(iii) |z + 2| > 4
 
P 1 1
Ans: (i) + (z + 2)n
2.4n 3n+1

1 P (z + 2)n P 3n
(ii) − −
2 4n (z + 2)n

P 1
(iii) (2.4n − 3n )
(z + 2)z n+1

4.4 Poles and Residues

Zeros of an Analytic Function

A zero of an analytic function f (z) is a value of z for which


f (z) = 0. If f (a) = 0 and f (a) 6= 0, the z = a is called a simple
zero or a zero of first order for f (z).

If f (a) = f ′ (a) = f ′′ (a) = · · · f m−1 (a) = 0 and f m (a) 6= 0, then


z = a is called a zero of order m for f (z).

Singular Points

We know that a point z0 at which a function f (z) is not analytic

243
4 COMPLEX INTEGRATION

is known as a singular point or a singularity of f (z).

Types of Singular Points

(i) Isolated Singular Point

A singular point z = z0 is said to be an isolated singular point


of f (z), if there is no other singular point in the neighbourhood
of z0 .

1
Example: The function f (z) = is analytic except at
z(z − 2)
z = 0 and z = 2. There are no other singularities of f (z) in the
neighbourhood of z = 0 and z = 2.

(i.e.) z = 0 and z = 2 are isolated singular points of the function.

(ii) Removable Singular Point

A singular point z = z0 is said to be a removable singular point


of f (z), if limz→z0 f (z) exists and is finite.

sin z
Example: The function f (z) = is analytic except at z = 0.
z
sin z
So z = 0 is a singularity of f (z). Also limz→0 f (z) = limz→0 =
z
1. Hence z = 0 is a removable singular point of f (z).

tan z
Example: The function f (z) = is analytic except at
z
z = 0. So z = 0 is a singularity of f (z). Also limz→0 f (z) =
tan z
limz→0 = 1. Hence z = 0 is a removable singular point of
z
f (z).

244
4 COMPLEX INTEGRATION

z 2 − a2
Example: The function f (z) = is analytic except z =
z+a
−a. So z = −a is a singularity of f (z).

z 2 − a2 (z − a)(z + a)
Also limz→−a f (z) = limz→−a = limz→−a =
z+a z+a
−2a. Hence z = −a is a removable singular point of f (z).

Essential Singular Point

A singular point z = z0 is said to be an essential singular point


of f (z), if it is neither an isolated singularity nor a removable
singularity.
1
Example: The function f (z) = e z+1 is analytic except at z =
−1. So z = −1 is a singularity of f (z). Also z = −1 is neither an
isolated singularity nor a removable singularity. Hence z = −1 is
an essential singular point of f (z).
1
Example: The function f (z) = e z is analytic except at z =
0. So z = 0 is a singularity of f (z). Also z = 0 is neither an
isolated singularity nor a removable singularity. Hence z = 0 is
an essential singularity of f (z).

Poles

Poles are nothing but isolated singular points. Pole of order one is
called a simple pole. For a positive integer n, if limaz (z−a)n f (z) 6=
0, then z = a is called a pole of order n for f (z).

Residue at a Pole

245
4 COMPLEX INTEGRATION

The Laurent’s series expansion of f (z) about z = a is given by

P∞ P∞ bn
f (z) = n=0 an (z − a)n + n=1
(z − a)n

= a0 + a1 (z − a) + a2 (z − a)2 + a3 (z − a)3 + · · ·

b1 b2 b3
+ + + + ···
(z − a) (z − a)2 (z − a)3

1 H f (z) 1 H f (z)
where an = C dz, b n = dz
2πi 1 (z − a)n+1 2πi C2 (z − a)1−n
both the integrals being taken anticlockwise direction.

1
The co-efficient of , i.e., b1 is called the residue of f (z) at
z−a
1 H
z = a. That is b1 = f (z)dz is called the residue of f (z)
2πi C
at z = a.

Evaluation of Residues of f(z)

(i) Residue of f (z) at its simple pole z = z0 is given by

R = Re(z = z0 ) = limz→z0 (z − z0 )f (z).

φ(z)
(ii) If f (z) = , where ψ(a) = 0 but φ(a) = 0, then
ψ(z)

φ(z)
R = Re(z = z0 ) = limz→z0 (z − z0 )f (z) = .
ψ(z)

(iii) Residue of f (z) at its pole z = z0 of order n is given by

246
4 COMPLEX INTEGRATION
 
1 dn−1
R = Re(z = z0 ) = limz→z0 (z − z0 )n f (z)
(n − 1)! dz n−1

4.5 Cauchy’s Residue Theorem

Statement: If f (z) is analytic at all points inside and on a sim-


ple closed curve C except for a finite number ofHisolated singular
points (poles) z1 , z2 , z3 , · · · zn within C, then C f (z)dz = 2πi
(sum of the Residues of f(z) at z1 , z2 , z3 , · · · zn )

Proof: Let us surround each of the poles z1 , z2 , z3 , · · · zn by small


circles C1 , C2 , · · · Cn all within C then the circles together with
′ C ′ form a multiply connected region in which the function is

analytic.

Then by Cauchy’s theorem,


I I I I
f (z)dz = f (z)dz + f (z)dz + · · · + f (z)dz (4.17)
C C1 C2 Cn
1 H
We know that f (z)dz = Residue of f(z) at z.
2πi C
H
⇒ C f (z)dz = 2πi (Residue of f(z) at z) = 2πiR. Hence
H
C1 f (z)dz = 2πi (Residue of f(z) at z = z1 ) = 2πiR1
H
C2 f (z)dz = 2πi (Residue of f(z) at z = z2 ) = 2πiR2
H
C3 f (z)dz = 2πi (Residue of f(z) at z = z3 ) = 2πiR3 and so on.

Now equation (4.17) becomes

247
4 COMPLEX INTEGRATION

H
C f (z)dz = 2πiR1 + 2πiR2 + 2πiR3 + · · · + 2πiRn

= 2πi(R1 + R2 + R3 + · · · + Rn )
P
= 2πi Ri , where Ri is the residue of f (z) at z = zi .

= 2πi (sum of the residues of f(z) at z1 , z2 , z3 , · · · zn )


z2
Example 4.28. Determine the poles of f (z) =
(z − 1)(z − 2)2

Solution: The poles are (z − 1)(z − 2)2 = 0 ⇒ z = 1 is a simple


pole and z = 2 is a pole of order 2.

Example 4.29. Find the poles of cot z.

cos z
Solution: Let f (z) = cot z = .
sin z

Poles are given by sin z = 0 ⇒ z = nπ

⇒ z = 0, ±π, ±2π, · · ·

Example 4.30. Find the poles of (i) tan z (ii) csc z

Solution:

sin z
(i) Let f (z) = tan z =
cos z

The poles are given by cos z = 0


π π
⇒ z = (2n + 1) = nπ + , n = 0, ±1, ±2, · · ·
2 2

248
4 COMPLEX INTEGRATION

1
(ii) Let f (z) = csc z = .
sin z

Poles are given by sin z = 0 ⇒ z = nπ ⇒ z = 0, ±π, ±2π, · · ·


Example 4.31. Determine the poles of the function f (z) =
z2
and also find the residue at each pole.
(z − 1)2 (z + 2)

Solution: Poles are obtained by putting (z − 1)2 (z + 2) = 0 ⇒


z = −2, 1, 1. hence f (z) has a simple pole at z = −2 and pole of
order 2 at z = 1.

(i) Residue of f (z) at the simple pole z = −2 is

(z + 2)2 z 2
R1 = limz→−2 (z + 2)f (z) = limz→−2
(z − 1)2 (z + 2)

z2 4
= limz→−2 2
=
(z − 1) 9

(ii) Residue of f (z) at the pole z = 1 of order 2 is


   
d 2 d (z + 2)2 z 2
R2 = limz→1 (z − 1) f (z) = limz→1
dz dz (z − 1)2 (z + 2)
    
d z2 (z + 2)2z − z 2 5
= limz→1 = limz→1 2
=
dz z+2 (z + 2) 9
e2z
Example 4.32. Find the residues of f (z) = .
(z + 1)2

Solution: The poles are (z + 1)2 = 0 ⇒ z = −1 of order 2.

249
4 COMPLEX INTEGRATION

Residue of f (z) at the pole z = 1 is a pole of order 2 is


 
1 d 2
R = limz→−1 (z + 1) f (z)
(2 − 1)! dz
 
d e2z d
= limz→−1 (z + 1)2 = limz→−1 e2z = 2e−2
dz (z + 1)2 dz
z
Example 4.33. Find the residues at their poles of f (z) = .
(z − 1)2

Solution: The poles are given by (z − 1)2 = 0. So z = 1 is a


pole of order 2.

Residue of f (z) at the pole z = 1 is a pole of order 2 is


 
1 d 2
R = limz→1 (z − 1) f (z)
(2 − 1)! dz
 
d 2 z d
= limz→1 (z − 1) 2
= limz→1 (z) = 1
dz (z − 1) dz
H sin πz + cos πz 2
Example 4.34. Evaluate C dz where C is a
z + z2
circle |z| = 2.

Solution: The poles are given by z(z + 1) = 0 ⇒ z = 0, −1 are


simple poles and both lie within the circle |z| = 2.

R1 = Residue of f (z) at the simple pole z = 0 is

sin πz + cos πz 2
limz→0 (z − 0)f (z) = limz→0 =1
z(z + 1)

250
4 COMPLEX INTEGRATION

R2 = Residue of f (z) at the simple pole z = −1 is

Re(z = −1) = limz→−1 (z + 1)f (z)

sin πz + cos πz 2
= limz→0 (z + 1) =1
z(z + 1)

Now by Cauchy’s residue theorem

H sin πz + cos πz 2 H
C 2
dz = C f (z)dz = 2πi (sum of all residues)
z+z

= 2πi(R1 + R2 ) = 2πi(1 + 1) = 4πi


H cos πz 2
Example 4.35. Evaluate C dz where C is a circle
(z − 1)(z − 2)
|z| = 3.

Solution: Poles are given by (z − 1)(z − 2) = 0 ⇒ z = 1, 2 are


simple poles and both lie within |z| = 3.

R1 = Residue of f (z) at the simple pole z = 1 is

cos πz 2
Re(z = 1) = lim (z − 1)f (z) = lim (z − 1) =1
z→1 z→1 (z − 1)(z − 2)

R2 = Residue of f (z) at the simple pole z = 2 is

cos πz 2
Re(z = 2) = lim (z − 2)f (z) = lim (z − 2) =1
z→2 z→2 (z − 1)(z − 2)

Now by Cauchy’s residue theorem

251
4 COMPLEX INTEGRATION

H cos πz 2 H
C (z − 1)(z − 2) dz = C f (z)dz = 2πi(R1 + R2 ) = 2πi(1 + 1) =
4πi

Example 4.36. Find the residue at the poles of the function


z
2
.
z +1

Solution: Poles are given by z = ±i ⇒ z = i, −i are simple


poles.

R1 = Residue of f (z) at the simple pole z = i is


z 1
Re(z = i) = lim(z − i)f (z) = lim(z − i) =
z→i z→i (z + i)(z − i) 2

R2 = Residue of f (z) at the simple pole z = −i is


z 1
Re(z = −i) = lim (z + i)f (z) = lim (z + i) =−
z→−i z→−i (z + i)(z − i) 2
H cos πz
Example 4.37. Evaluate C z − 1 where C is a circle |z| = 1.5

Solution: The simple pole z = 1 is lies within the circle |z| = 1.5.

R = Residue of f(z) at the pole z = 1 is

cos πz
= Re(z = 1) = limz→1 (z − 1) = −1.
z−1

By Cauchy’s residue theorem


H cos πz H
C z − 1 = C f (z)dz = 2πi(R) = 2πi(−1) = −2πi

252
4 COMPLEX INTEGRATION

H z2
Example 4.38. Evaluate C where C is (i) |z| =
(z − 1)2 (z + 1)
1
, (ii) |z| = 2.
2

Solution: Let (z − 1)2 (z + 1) = 0 implies that f (z) has simple


pole at z = −1 and a pole of order 2 at z = 1.

1
If C is |z| = , both poles lie outside C and hence by Cauchy’s
2
integral theorem

H H z2
C f (z)dz = C (z − 1)2 (z + 1)
dz = 0.

(i) If C is |z| = 2, both poles lie outside C.

z2 1
R1 = lim (z + 1)f (z) = lim (z + 1) 2
=
z→−1 z→−1 (z − 1) (z + 1) 4

 
d 2 z2
R2 = limz→1 (z − 1)
dz (z − 1)2 (z + 1)
 
(z + 1)2z − z 2 3
= limz→1 2
=
(z + 1) 2

By Cauchy’s residue theorem

H z2 H
C = C f (z)dz = 2πi(R1 + R2 )
(z − 1)2 (z + 1)
 
1 3 7πi
= 2πi + =
4 2 2

253
4 COMPLEX INTEGRATION

H e2z
Example 4.39. Evaluate C dz where C is a circle |z| =
cos πz
1.

Solution: Let f(z) has simple pole when cos πz = 0 ⇒ z =


1 3 1
± , ± , · · · of which z = ± lie inside the circle |z| = 1.
2 2 2
    2z
1 1 e 0
R1 = Re z = = limz→ 1 z − = form.
2 2 2 cos πz 0

Using L’hospital rule, we have


 
1
z− 2e2z + e2z
2 e
R1 = limz→ 1 =−
2 −π sin πz π
 
1 e−1
Similarly, R2 = Re z = − =
2 π

By Cauchy’s residue theorem,


 
H e2z H e e−1
C dz = C f (z)dz = 2πi(R1 + R2 ) = 2πi − +
cos πz π π

= −4i sin π
H
Example 4.40. Evaluate C tan zdz where C is a circle |z| = 2.

π 3π
Solution: The poles are given by cos z = 0 ⇒ z = ± , ± , · · ·
2 2
π
of which z = ± lie within |z| = 2.
2

254
4 COMPLEX INTEGRATION
 π
 π z− sin z  0 
R1 = Re z = = limz→ π2 2 , form
2 cos z 0
 π
cos z + sin z
z−
limz→ π2 2 = −1 (Using L’Hospital rule)
− sin z
 π
 π  z + sin z
R2 = Re z = − = limz→− π2 2
2 cos z
 π
z+ cos z + sin z
= limz→− π2 2 = −1
− sin z

By Cauchy’s residue theorem,


H H
C tan zdz = C f (z)dz = 2πi(R1 + R2 ) = 2πi(−1 − 1) = −4πi
z−1H
Example 4.41. Evaluate C where C is the circle
+ 2z + 5 z2
(i) |z| = 1, (ii) |z + 1 + i| = 2 and (iii) |z + 1 − i| = 2

Solution: The poles are given by z 2 + 2z + 5 = 0

⇒ (z + 1 + 2i)(z + 1 − 2i) = 0 ⇒ z = −1 − 2i, z = −1 + 2i are


the simple poles.

(i) Since both poles lie outside the circle |z| = 1, by cauchy’s
integral theorem, we have

H z−1 H
C = C f (z)dz = 0
z2 + 2z + 5

255
4 COMPLEX INTEGRATION

(ii) The centre of the circle |z + 1 + i| = 2 is (−1, −1) and radius


is 2. The simple pole −1 − 2i lies outside the circle.
z−1
R = Re(z = −1−2i) = lim (z+1+2i)
z→−1−2i (z + 1 + 2i)(z + 1 − 2i)
1+i
=
2i

By Cauchy’s residue theorem,


 
H z−1 H 1+i
C 2
= C f (z)dz = 2πi(R) = 2πi = π(1 + i)
z + 2z + 5 2i

(iii) The centre of the circle |z + 1 − i| = 2 is (−1, −1) and radius


is 2. The simple pole −1 + 2i lies within the circle |z + 1 − i| = 2.
z−1
R = Re(z = −1+2i) = lim (z+1−2i)
z→−1+2i (z + 1 + 2i)(z + 1 − 2i)
−1 + i
=
2i

By Cauchy’s residue theorem,


 
H z−1 H −1 + i
C 2
= C f (z)dz = 2πi(R) = 2πi = π(−1+
z + 2z + 5 2i
i)

EXERCISE

z(z − 1) z+3
1. Determine the poles of (i) (ii)
z(z − 2) z(z − 1)(z + 2)
z 1 1 − e2z
(iii) (iv) 2 and (v) .
cos z (z + a2 )2 z4

256
4 COMPLEX INTEGRATION

π
[Ans: (i) 0, 2 (ii) 0, 1, -2 (iii) (2n + 1) (iv) ai, -ia and
2
(v) 0 of order 4]

ez ez
2. Find the residues of (i) at z = ai (ii) (iii)
z 2 + a2 (z − 1)3
1 z2 z+2
2 2 2
at z = ai (iv) 2
(v)
(z + a ) (z − 1) (z + 2) (z + 1)2 (z − 2)
1 z 3
(vi) 2 2
and (vii) 2
(z + 1) (z + 1) (z − 2)(z + 2)
h −i
Ans: (i)2aieai , (ii)e/2, (iii)
, (iv)5/9, 4/9,
4a3
   
−i −i 9 27 i
(v)4/9, 4/9, (vi) , and (vii) , −8,
4 4 4 4

1
3. Find the singular point of f (z) =
z(ez −1

[Ans: z = 0, ±2π, ±4π, · · · ]

1 − e2z
4. Find the residues of f (z) = [Ans: -2]
z3

1 + ez
5. Find the residue at z = 0 of [Ans: 1]
z cos z + sin z

6. Find the residue at z = 0 of the functions (i) f (z) = e1/z


sin z 1
(ii) 4 and (iii) z cos
z z
 
1 1
Ans: (i)1 (ii) − (iii)
6 2

257
4 COMPLEX INTEGRATION

sin z tan z
7. Find the residues of f (z) = or at each of its
z cos z z
poles inside the circle |z| = 2
 
2
Ans:
11
H z sec z 2 2
8. Evaluate C 1 − z 2 where C is the ellipse 4x + 9y = 9

[Ans: −2πi sec 1]

9. Using Cauchy’s residue theorem evaluate the following:

H z 3
(i) C 2
where C is a circle |z| = [Ans: 0]
(z − 1) (z + 1) 2

H z 3
(ii) C over a circle |z| = 3 and |z| =
(z − 1)(z − 3) 2
[Ans: 2πi, −πi]

H e−2z
(iii) C where C is a circle |z| = 2 [Ans: 4πi]
(z + 1)3

H z−2
(iv) C where C is a circle |z| = 3 [Ans: 2πi]
z(z − 1)

H 7z − 1
(v) C 2 − 3z − 4
where C is an ellipse x2 + 4y 2 = 4
 z 
16
Ans: πi
3

H sin πz 2 + cos πz 2
(vi) C (z − 1)(z − 2) where is a circle |z| = 3

258
4 COMPLEX INTEGRATION

[Ans: 4πi, 2πi]


2
H e−z
(vii) C where C is a circle |z| = 1
cos πz

[Ans: 2i(−e1/4 + e−1/4 )]

Application of Residues to evaluate real integrals


R 2π
Integration around the unit circle of the type 0 f (cos θ, sin θ)dθ
where f (cos θ, sin θ)dθ is a rational function sin θ and cos θ.

Here z = eiθ [recall z = reiθ where r = 1 for unit circle] so that

eiθ + e−iθ z + z −1 eiθ − e−iθ z − z −1


cos θ = = and sin θ = =
2 2 2 2i
R 2π dθ
Example 4.42. Evaluate 0
5 + 3 cos θ

dz
Solution: Let z = eiθ ⇒ dz = ieiθ dθ ⇒ dθ = and
iz

eiθ + e−iθ z + z −1 z2 + 1
cos θ = = =
2 2 2z
R 2π dθ H 1 dz
∴I= 0 = C  
5 + 3 cos θ 3 1 z
5+ z+
2 z

2H dz 2H 2
= C 2
= C f (z)dz = [2πi (sum of the resi-
i 3z + 10z + 3 i i
dues of f (z))]

259
4 COMPLEX INTEGRATION

1
= 4π(R1 + R2 + · · · ) where f (z) = and C is a unit
3z 2 + 10z + 3
circle |z| = 1.

Poles of f (z) are z = −1/3, 3 out of which z = −1/3 lies inside


C.
   
1 1 1
Hence R = Re z = − = limz→− 1 z +
3 3 3 (3z + 1)(z + 3)

1 1
= limz→− 1 =
3 3(z + 3) 8
 
R 2π dθ 1 π
∴ 0 = 4π =
5 + 3 cos θ 8 2
R 2π dθ 2π
Example 4.43. Show that 0 =√
1 + a cos θ 1 − a2

dz
Solution: Let z = eiθ ⇒ dz = ieiθ dθ ⇒ dθ = and
z

eiθ + e−iθ z + z −1 z2 + 1
cos θ = = =
2 2 2z
R 2π dθ H 1 dz
∴I= 0 = C  
1 + a cos θ a 1 iz
1+ z+
2 z

2H dz 2H 1
= C 2
= C f (z)dz, where f (z) = 2 .
i az + 2z + a i az + 2z + a

2 −1 ± 1 − a2
Poles of f (z) are given by az + 2z + a = 0 ⇒ z =
a

260
4 COMPLEX INTEGRATION
√ √
−1 + 1 − a2 −1 + 1 − a2
Let α = ,β = it is clear that
a a
r
1 1
β=− − −1>1
a a2

Since a < 1 ⇒ 1/a > 1 and hence |β| > 1. Now αβ = 1

∴ |α| < 1.

z = α is a simple pole inside C.

1
R = Re(z = α) = limz→α (z − α)
a(z − α)(z − β)

1 1
= = √
a(z − α) 2 1 − a2

R 2π dθ 2H
Hence I = 0 = f (z)dz
1 + a cos θ i C

2
= [2πi (sum of the residues of f(z)]
i
 
1 2π
= 4π(R1 + R2 + · · · ) = 4π √ =√
2 1 − a2 1 − a2
R 2π dθ
Example 4.44. Evaluate 0 13 + 5 sin θ

dz
Solution: Let z = eiθ ⇒ dz = ieiθ dθ ⇒ dθ = and
iz

261
4 COMPLEX INTEGRATION

eiθ − e−iθ z − z −1 z2 − 1
sin θ = = =
2 2i 2iz
R 2π dθ H 1 dz
∴I= 0 = C  2 
13 + 5 sin θ z − 1 iz
13 + 5
2iz

2H dz 2H 1
=
5 C 26
=
5 C f (z)dz, where f (z) = 26
z 2 + iz − 1 z 2 + iz − 1
5 5
26 i
Poles of f (z) are given by z 2 + iz − 1 = 0 ⇒ z = − , −5i out
5 5
i
of this z = − lies within |z| = 1.
5
   
i i 1 5i
R = Re z = − = lim z +   =−
5 z→− 5i 5 i 24
(z + 5i) z +
5
R 2π dθ 2H
Hence I = 0 = f (z)dz
13 + 5 sin θ 5 C

2
= [2πi (sum of the residues of f(z)]
5
  
4πi 4πi 5i π
= (R1 + R2 + · · · ) = − =
5 5 24 6
R 2π dθ
Example 4.45. Evaluate 0 2 + cos θ

dz
Solution: Let z = eiθ ⇒ dz = ieiθ dθ ⇒ dθ = and
z

262
4 COMPLEX INTEGRATION

eiθ + e−iθ z + z −1 z2 + 1
cos θ = = =
2 2 2z
R 2π dθ H dz
∴I= 0 = C  
2 + cos θ z2 + 1
iz 2 +
2z

H dz H 1
= −2i C = −2i C f (z)dz, where f (z) = 2 .
z2
+ 4z + 1 z + 4z + 1

2 +4z+1 = 0 ⇒ z = −2+ 3, −2− 3

Poles of f (z) are given
√ by z
of which z = −2 + 3 lies within |z| = 1.
√ 1
R = Re(z = −2 + 3) = lim √ √ √
z→−2+ 3 (z + 2 − 3)(z + 2 + 3)

1
= √
2 3

R 2π dθ H
Hence I = 0 = −2i C f (z)dz
2 + cos θ

= 2i [2πi (sum of the residues of f(z)]


 
1 2π
= 4π(R1 + R2 + · · · ) = 4π √ =√
2 3 3
R 2π sin2 θdθ
Example 4.46. Evaluate 0 5 + 4 cos θ

dz
Solution: Let z = eiθ ⇒ dz = ieiθ dθ ⇒ dθ = and
z

eiθ + e−iθ z + z −1 z2 + 1
cos θ = = =
2 2 2z

263
4 COMPLEX INTEGRATION

R 2π sin2 θdθ R 2π (1 − cos 2θ)dθ


∴I= 0 5 + 4 cos θ = 0 2(5 + 4 cos θ)

1H 1 − z2
= R.P   dz
2 C 4 z2 + 1
5+
2 2

1 1 − z2
I = R.P dz (4.18)
2i 2z 2 + 5z + 2
Poles are given by 2z 2 + 5z + 2 = 0 ⇒ z = −2, −1/2, out of these
z = −1/2 lies within |z| = 1.
 
1 1 − z2
R = Re(z = −1/2) = limz→1/2 z +  
2 2
5
2 z + z+1
2
 
1 1 − z2 1
= limz→−1/2 z +   =
2 1 1 4
z+ (z + 2)
2 2

Hence equation (4.18) becomes

R 2π sin2 θdθ 1 H 1 − z2
I= 0 5 + 4 cos θ = R.P dz
2i C 2z 2 + 5z + 2

1
= R.P [2πi (sum of the residues of f(z)]
2i
 
1 π
= R.P π(R1 + R2 + · · · ) = R.P π =
4 4
R 2π cos 2θdθ
Example 4.47. Evaluate 0 5 − 4 cos θ

264
4 COMPLEX INTEGRATION

dz
Solution: Let z = eiθ ⇒ dz = ieiθ dθ ⇒ dθ = and
z
1
eiθ + e−iθ z+ 2
cos θ = = z = z + 1 and z 2 = (eiθ )2 = cos 2θ +
2 2 2z
i sin 2θ, R.P of e2iθ = cos 2θ.

R 2π cos 2θdθ H z2 dz
Now I = 0 5 − 4 cos θ = R.P C  2 
z + 1 iz
5−4
2z

−1 z2
I = R.P dz (4.19)
i 2z 2 − 5z + 2

Poles are given by 2z 2 −5z+2 = 0 ⇒ z = 2, 1/2 but only z = 1/2


lies within |z| = 1.
 
1 z2 1
R = Re(z = 1/2) = limz→1/2 z − =−
2 (z − 2)(2z − 1) 12

Hence equation (4.19) becomes

R 2π cos 2θdθ −1 H z2
I= 0 5 − 4 cos θ = R.P dz
i C 2z 2 − 5z + 2
 
−1
= R.P (2πi)(R1 + R2 + · · · )
i
   
−1 −1 π
= R.P (2πi) =
i 12 6
R 2π dθ
Example 4.48. Evaluate 0 , a2 < 1
1 − 2a cos θ + a2

265
4 COMPLEX INTEGRATION

dz
Solution: Let z = eiθ ⇒ dz = ieiθ dθ ⇒ dθ = and
z

eiθ + e−iθ z + z −1 z2 + 1
cos θ = = =
2 2 2z
R 2π dθ H (dz/iz)
Now I = 0 2
= C  2 
1 − 2a cos θ + a z +1
1 − 2a + a2
2z

2dz
H (dz/iz)2z H i
= C = C
2z − 2a(z 2 + 1) + a2 .2z 2(z − az 2 − 2a + a2 z

H −idz H −idz
= C = C az(−z + a) + (z − a)
−az 2 + a2 z + z − a
I
dz
I = −i (4.20)
C (z − a)(1 − az)

Poles are given by (z − a)(1 − az) = 0 ⇒ z = a, 1/a. Given


a2 < 1 ⇒ a < 1 and hence the pole z = 1/a lies outside the unit
circle C and only z = a lis within the unit circle.

1 1
R = Re(z = a) = limz→a (z − a) =
(z − a)(1 − az) 1 − a2

Hence equation (4.20) becomes

R 2π dθ H dz
I= 0 2
= −i C
1 − 2a cos θ + a (z − a)(1 − az)
 
1 2π
= −i(2πi) =
1 − a2 1 − a2

266
4 COMPLEX INTEGRATION

Rπ adθ
Example 4.49. Evaluate 0 a2 + cos2 θ

Solution: Put dθ = ϕ so that 2dθ = dϕ. Also when θ = 0, ϕ = 0


and when θ = π, ϕ = 2π.

Rπ adθ Rπ adθ
Now I = 0 = 0
 
a2 + cos2 θ 1 + cos 2θ
a2 +
2


Rπ 2adθ R 2π 2a
= = 0 2
0 2a2 + 1 + cos 2θ 2a2 + 1 + cos 2ϕ
Z 2π

I=a 2
(4.21)
0 2a + 1 + cos 2ϕ
dz
Let z = eiθ dz = ieiθ dϕ ⇒ dϕ = and
iz
1
eiϕ + e−iϕ z+ 2
cos ϕ = = z = z +1
2 2 2z

Now equation (4.21) becomes

Rπ adθ R 2π dϕ
I= 0 =a 0
a2 2
+ cos θ 2
2a + 1 + cos 2ϕ

H (dz/iz) H (dz/iz)2z
=a C  2  =a C 2
z +1 4a z + 2z + z 2 + 1
2a2 + 1 +
2z
I
dz
I = −2ai 2 2
(4.22)
C z + 2z(2a + 1) + 1

267
4 COMPLEX INTEGRATION
p
−2(2a2 + 1) ± 4(2a2 + 1)2 − 4
Poles are given by z =
2

−2(2a2 + 1) ± 2 4a4 + 4a2 + 1 − 1 √
= = −(2a2 +1)±2a a2 + 1
2

Let the√roots be α = −(2a2 + 1) + 2a a2 + 1 and β = −(2a2 +
1) − 2a a2 + 1

Since |β| > 1 and |α||β| = 1 ∴ |α| < 1.

1 1
Then R = Re(z = α) = limz→α (z − α) =
(z − α)(z − β) α−β

Now equation (4.22) becomes

Rπ adθ H dz
I= 0 = −2ai C 2
a2 2
+ cos θ z + 2z ∗ 2a2 + 1) + 1
 
1 π
= −2ai(2πi)(R) = −2ai(2πi) =√
α−β a2+1

EXERCISE

R 2π dθ h πi
1. Evaluate 0 Ans:
13 + 5 sin θ 6
 
R 2π dθ 2π
2. Evaluate 0 Ans:
5 + 4 cos θ 3

R 2π cos 2θdθ h πi
3. Evaluate 0 5 − 4 cos θ Ans:
6

268
4 COMPLEX INTEGRATION

R 2π sin2 θdθ h πi
4. Evaluate 0 5 + 4 cos θ Ans:
4
 
Rπ dθ π
5. Evaluate 0 , a > |b| Ans: √
a + b cos θ a 2 − b2
 
Rπ dθ π
6. Evaluate 0 ,a > 0 Ans: √
a + sin2 θ
2 a a2 − 1
 
Rπ a cos θ a
7. Show that dθ = 2aπ 1 − √
−π a + cos θ a2 − 1

R π (1 + 2 cos θ)dθ
8. Show that 0 =0
5 + 4 cos θ
 
R 2π dθ π
9. Evaluate 0 , a > |b| > 0, Ans: 2
(a + b cos θ)2 (a − b2 )3/2

Type II: Improper integrals of rational functions


R∞
Evaluation of −∞ f (x)dx

If the function f (z) is such that it has no poles on the real axis
and may have some poles in the upper half of theR z - plane and

if xf (x) → 0 as x → ∞ then we can evaluate −∞ f (x)dx by
using Cauchy’s residue formula.
R
Method: Consider I = C f (z)dz

where C is a curve, consisting of the upper half C1 of the circle


|z| = R and part of the real axis from -R to R.

269
4 COMPLEX INTEGRATION

If there are no poles of f (z) on the real axis, then the circle
|z| = R which is arbitrary can be taken such that there is no
singularity of f (z) on the circumference C1 of the upper half of
the plane, but there may be poles of f (z) inside the contour C.

Now, using Cauchy’s theorem of residues, we have,


R
C f (z)dz = 2πi× (sum of residues of f(z) at the poles within
C).
RR R
(i.e.) −R f (x)dx + C1 f (z)dz = 2πi (sum of residues)

(Note that along -R to R, f(z) is real and valued function f(x)).


RR R
∴ −R f (x)dx =− C1 f (z)dz + 2πi (sum of residues within C).

Taking limit as R → ∞ on both sides, we get,


Z R Z
lim f (x)dx = − lim f (z)dz + 2πi (sum of residues)
R→∞ −R R→∞ C1
(4.23)
Let z = reiθ
R
Consider limR→∞ C1 f (z)dz = limR→∞ f (reiθ )rieiθ dθ
R
= limR→∞ C1 f (reiθ )eiθ dθ (R → ∞, r → ∞)

= 0 (∴ xf (x) → 0 as x → 0)

Hence from (4.23)


RR
limR→∞ −R f (x)dx = 2πi (sum of residues of f(z) within C)

270
4 COMPLEX INTEGRATION

R∞
(i.e.) −∞ f (x)dx = 2πi (sum of residues of f(z) within C)

R∞ 1 R∞
∴ 0 f (x)dx = f (x)dx
2 −∞

Cauchy’s Lemma:

If f(z) is continuous function such that |zf (z)| → 0 as |z| → ∞


on the upper semi-circle C1 .
R
|z| = R then C1 f (z)dz → 0 as R → ∞

Type I:

R ∞ P (x)
Integrals of the form −∞ Q(x) dx where P(x) and Q(x) are po-
lynomial in x such that the degree of Q(x) is at least two more
than the degree of P(x) and Q(x) does not vanish for any real x.
R∞ x2
Example 4.50. Evaluate −∞ 2 dx, a > 0, b >
(x + a2 )(x2 + b2 )
0

P (x)
Solution: The integrand is of the form where Q(x) is
Q(x)
two more than degree of P(x) and Q(x) does not vanish. Let
x2
f (x) = 2 (f(x) has no poles on real axis).
(x + a2 )(x2 + b2 )

z2
Hence f (z) =
(z 2 + a2 )(z 2 + b2 )

The poles of f(z) are z = ±ia, z = ±ib. Out of these z = ia and


z = ib lie in the upper half of z - plane.

271
4 COMPLEX INTEGRATION

Let us find residues of f(z) at z = ia and z = ib.

Residue of f(z) at z = ia is

z2
limz→ia (z − ia)f (z) = limz→ia
(z 2 + a2 )(z 2 + b2 )

−a2 a
= = and residue of f(z) at z = ib is
(2ia)(b2 − a2 ) 2i(b2 − a2 )

z2 z2
= limz→ib (z − ib) = limz→ib
(z 2 + a2 )(z 2 + b2 ) (z 2 + a2 )(z + ib)

−b2 −b
= 2 2
=
(2ib)(a − b ) 2i(a2 − b2 )
Rx
∴ −x f (x)dx = 2πi (sum of residues of f(z) in the upper half of
z - plane)
   
a b a−b π
= 2πi 2 2
− 2 2
=π 2 2
=
2i(b − a ) 2i(a − b ) a −b a+b
R∞ x 2
Example 4.51. Evaluate −∞ 2 dx using contour
(x + 1)(x2 + 4)
integration.

x2
Solution: Let f (x) =
(x2 + 1)(x2 + 4)

z2
Let f (z) =
(z 2 + 1)(z 2 + 4)

The poles of f(z) are given by (z 2 + 1)(z 2 + 4) = 0

272
4 COMPLEX INTEGRATION

⇒ z = ±i, ±2i which are simple poles.

But only z = i, 2i lie in the upper half of the z - plane.

Let us find the residue of f(z) at z = i z = 2i.

z2 1
Re(z = i) = limz→i (z − i) 2
=−
(z + i)(z − i)(z + 4) 6i

z2 1
Re(z = 2i) = limz→2i (z − 2i) 2
=
(z + 2i)(z − 2i)(z + 4) 3i
Rx
∴ −x f (x)dx = 2πi (sum of residues of f(z) in the upper half of
z - plane)
 
1 1 π
= 2πi − + =
6i 3i 3
R∞ dx
Example 4.52. Evaluate 0 x4 +1

1
Solution: Let f (x) = (Note that here limits are 0 to ∞)
x4 +1

1
Let f (z) = . We find poles of f(z).
z4 +1

(i.e.) z 4 = −1 = eiπ = ei(2nπ+π)


π
∴ z = ei(2n+1) 4 where n = 0, 1, 2, 3.

π 1 1
When (i) n = 0, z = ei 4 = √ + i √
2 2

273
4 COMPLEX INTEGRATION

3π 1 1
(ii) n = 1, z = ei 4 = √ + i√
2 2

5π 5π 5π 1 1
(iii) n = 2, z = ei 4 = cos + i sin = −√ − i√
4 4 2 2

7π 1 1
(iv) n = 3, z = ei 4 = √ − i√
2 2
π 3π
Of these only z = ei 4 and z = ei 4 lie above the upper half of z
- plane.
π 3π
We find residues of f(z) at z = ei 4 and z = ei 4

π
(i) Residue at z = ei 4

π 1
= limz→ei π4 (z − ei 4 )
z4 +1

Applying L’Hospital’s rule

π 1 1 1 3π
= limz→ei π4 (z − ei 4 ) = = e−i 4
4z 3 i 3π
4e 4 4

(ii) Residue of z = ei 4

3π 1
= lim 3π (z − ei 4 )
z→ei 4 z4 + 1

Applying L’Hospital’s rule

3π 1 1 1 −i 9π
= lim 3π (z − ei 4 ) = 9π = e 4
z→ei 4 4z 3 i 4
4e 4

274
4 COMPLEX INTEGRATION

R∞ dx 1 R∞ dx Ra Ra
Now 0 = (Recall −a f (x)dx = 2 0 f (x)dx,
x4 +1 2 −∞ 4
x +1
if f(x) is even)

1
= × 2πi (sum of residues)
2
 
1 −i 3π 1 −i 9π
= πi e 4 + e 4
4 4
 
πi 3πi 3πi 9πi 9πi
= cos − i sin + cos − i sin
4 4 4 4 4
  √
πi 1 1 1 1 πi √ π 2
= − √ − i √ + √ − i √ = (−i 2) =
4 2 2 2 2 4 4
R∞ dx
Example 4.53. Evaluate 0 (x2 + 1)3

R∞ dx
Solution: Let I = 0 (x2 + 1)3

1 R∞ dx
=
2 −∞ (x + 1)3
2

dx
f (x) =
(x2 + 1)3

dz R dz
Let f (z) = and consider C 2
(z 2 + 1) 3 (z + 1)3

Here f(z) has poles at z = ±i of order 3 each. Out of these z = i,


a multiple pole of order 3 lies above x-axis.

275
4 COMPLEX INTEGRATION

We find residue at z = i
 
1 d2 3 1 1 d2 1
= limz→i 2 (z − i) 2 = limz→i 2
2i dz (z + 1)3 2 dz (z + i)3

1 d 1
= limz→i [−3(z + i)−4 ] = limz→i [12(z + i)−5 ]
2 dz 2
6 1 3 1 −3i
= 6(2i)−5 = 5
. 5 = . =
2 i 16 i 16
 
R∞ −3i 3π
∴ −∞ f (x)dx = 2πi =
16 8
 
R∞ R∞ dx 1 3π 3π
∴ 0 f (x)dx = 0 = =
(x2 + 1)3 2 8 16

Type II:

R ∞ P (x) sin mx R ∞ P (x) cos mx


Integrals of the form dx and −∞ dx,
−∞ Q(x) Q(x)

m > 0, where P(x) and Q(x) are polynomials in x such that the
degree of Q(x) greater than the degree of P(x) and Q(x) does
not vanish for any real x.

Jacobi’s Lemma:

If f(x) is a continuous
R function such that |f (z)| → 0 uniformly
as |z| → 0 then C1 f (z)eimz dz → 0 as R → ∞ where C1 is the
upper semi-circular |z| = R.
R ∞ cos ax
Example 4.54. Evaluate 0 dx
(x2 + 1)

276
4 COMPLEX INTEGRATION

R ∞ cos ax 1 R ∞ cos ax
Solution: Let I = 0 x2 + 1 dx = 2 −∞ x2 + 1 dx

R ∞ cos ax R ∞ eiax
Consider 0 x2 + 1 dx = R.P. 0 x2 + 1 dx

eiax eiaz
Let f (x) = . We write f (z) =
x2 + 1 z2 + 1

This has poles at z = ±i, out of these z = i lies in the upper


half of z - plane.

Residue of f(z) at z = i is

eiaz eiaz e−a


limz→i (z − i) = lim z→i (z − i) =
z2 + 1 (z − i)z + i) 2i
 
R eiaz R ∞ eiax e−a
∴ C dz = −∞ x2 + 1 dx = 2πi = πe−a
z2 + 1 2i

R ∞ (cos ax + i sin ax)


∴ dx = πe−a
−∞ x2 + 1
R ∞ cos ax R ∞ sin ax 2
∴ −∞ x2 + 1 dx + x + 1dx = πe−a
−∞ =
R ∞ cos ax
equating real part, we get −∞ x2 + 1 dx = πe
−a

R ∞ cos ax 1 R ∞ cos ax π
0 (x2 + 1) dx = 2
dx = e−a
2 −∞ x +1 2
R∞ cos 2xdx
Example 4.55. Evaluate 0 (x2 + 9)2 (x2 + 16)

277
4 COMPLEX INTEGRATION

R∞ cos 2xdx
Solution: Let I = 0 (x2 + 9)2 (x2 + 16)

1 R∞ cos 2xdx
=
2 −∞ (x + 9)2 (x2 + 16)
2

cos 2xdx
f (x) =
(x2 + 9)2 (x2 + 16)

R cos 2zdz
Consider C f (z)dz where f (z) = .
(z 2 + 9)2 (z 2 + 16)

f(z) has double poles at z = ±3i and simple poles at z = ±4i.


Out of these z = 3i and z = 4i lie above real axis.

To find residues

(i) Residue at double pole z = 3i.

1 d
= [(z − 3i)2 f (z)]z=3i
1! dz
 
d 2 cos 2zdz
= (z − 3i)
dz (z − 3i)2 (z + 3i)2 (z 2 + 16) z=3i
 
d cos 2zdz 31ie−6
= = −
dz (z + 3i)2 (z 2 + 16) z=3i 108 × 49

(ii) Residue at a simple pole z = 4i is given by

(z − 4i) cos 2zdz


= limz→4i (z − 4i)f (z) = limz→4i
(z 2 + 9)2 (z 2 + 16)

278
4 COMPLEX INTEGRATION

cos 2zdz ie−8


= limz→4i = −
(z 2 + 9)2 (z + 4i) 392

By Residue theorem,
 
R∞ cos 2xdx 31ie−6 ie−8
= 2πi − −
−∞ (x2 + 9)2 (x2 + 16) 108 × 49 392
 
2π 31 −6 e−8
= e +
196 27 2
 
R∞ cos 2xdx π 31 −6 e−8
∴ 0 = e +
(x2 + 9)2 (x2 + 16) 196 27 2

EXERCISE

Evaluate the following integrals:

R∞ dx h πi
1. Ans:
−∞ x6 + 1 3
 
R∞ dx π
2. Ans:
−∞ (x + a )(x2 + b2 )
2 2 ab(a + b)
 
R ∞ (x2 − x + 2)dx 5π
3. −∞ 4 Ans:
x + 10x2 + 9 12
 
R ∞ x sin xdx πe−a
4. 0 ,a > 0 Ans:
x2 + a2 2

R∞ x2 dx h π i
5. 0 Ans:
(x2 + 9)(x2 + 4)2 200

279
4 COMPLEX INTEGRATION

R∞ cos xdx
6. ,a > b > 0
−∞ (x2 + a2 )(x2 + b2 )
  
π e−b e−a
Ans: 2 −
a − b2 b a
  
R∞ cos 3xdx π e−3 e−6
7. Ans: −
−∞ (x2 + 1)((x2 + 4) 3 1 2

R∞ x2 dx h πi
8. 0 Ans:
(x2 + 1)2 4
 
R ∞ x sin mxdx πe−ma
9. 0 , m > 0, a > b > 0 Ans:
x2 + a2 2
 
R∞ x cos mxdx π(be−mb − ae−ma )
10. 0 Ans:
(x2 + a2 )(x2 + b2 ) 2(b2 − a2 )
 
R ∞ cos mxdx π(m + 1)e−m
11. 0 ,m > 0 Ans:
(1 + x2 )2 4
 
R∞ sin xdx −π sin 2
12. 0 2
Ans:
x + 4x + 5 e

280

You might also like